Haryana Board Class 6 Maths Solutions For Chapter 10 Mensuration

Haryana Board Class 6 Maths Solutions For Chapter 10 Mensuration

  • Perimeter is the distance covered along the boundary forming a closed figure when you go round the figure once.
  • The length of the boundary of a closed figure is called its ‘Perimeter’ (Or) Sum of all the lengths of a polygon is called its ‘Perimeter’.
  • The perimeter of a rectangle = 2 [length + breadth]
  • Perimeter of a square = 4 x [length of its side]
  • Perimeter of a equilateral triangle = 3 x length of the side.
  • Circumference [Perimeter] of a circle = πd (Or) 2πr
  • The place occupied by an object is called area of the object. Area is measured in square units.
  • Area of a rectangle = length x breadth
  • Area of a square = side x side

Question 1. Meera went to a park 150 m long and 80m wide. She took one complete round on its boundary. What is the distance covered by her?

Solution.

Class 6 Maths Chapter 10 Mensuration Question 3

Distance covered by Meera

= AB + BC + DC + DA

= (150 + 80 + 150 + 80) m

= 460 m

Haryana Board Class 6 Maths Mensuration solutions

Question 2. Find the perimeter of the following figures:

Solution. a)

Class 6 Maths Chapter 10 Mensuration Question 4.1

Perimeter

= AB + BC + CD + DA

= 40 m + 10 m + 40 m + 10m

= 100 m

b)

Class 6 Maths Chapter 10 Mensuration Question 4.2

Perimeter

= AB + BC + CD + AD

= 5m + 5m + 5m + 5m

= 20 m

c)

Class 6 Maths Chapter 10 Mensuration Question 4.3

Perimeter

= AB + BC + CD + DE + EF + FG + GH + HI + IJ + JK + KL + LA

= 1 cm + 3 cm + 3 cm + 1 cm + 3 cm + 3cm + 1 cm + 3 cm + 3 cm + 1 cm + 3 cm + 3 cm

= 28 cm

Perimeter and area formulas for Class 6 HBSE Maths

d)

Class 6 Maths Chapter 10 Mensuration Question 4.4

Perimeter

= AB + BC + CD + DE + EF + FA

= 100 m + 120 m + 90m + 45m + 60 m + 80 m

= 495 m

Question 3. Find the perimeter of the following rectangles:

Class 6 Maths Chapter 10 Mensuration Question 5

Solution. 1) Perimeter = 2(1 + b) = 2 (0.5 m + 0.25 m)

= 2(0.75) m = 1.5 m

Perimeter by adding all the sides

= 0.5 + 0.25 + 0.5 + 0.25

= 1.5 m

2) Perimeter = 2(1 + b)= 2(18 cm + 15 cm) = 66 cm.

Perimeter by adding all the sides

= 18 + 15 + 18 + 15 = 66 cm.

3) Perimeter = 2(1 + b)

= 2 (10.5 cm + 8.5 cm)

= 2 (19 cm) = 38 cm.

Perimeter by adding all the sides

= 10.5 + 8.5 + 10.5 + 8.5

= 38 cm.

Volume of cube and cuboid Class 6 HBSE Maths

Question 4. Find various objects from your surroundings which have regular shapes and find their perimeters.

1) Square:

Solution. 1) A square has four equal sides.

2) Let’s assume the length of one side is 5 cm.

3) The perimeter of a square is calculated by multiplying the length of one side by 4.

4) Therefore, the perimeter of the square would be 5 cm x 4 = 20 cm.

2) Rectangle:

Solution. 1) A rectangle has two pairs of equal sides (opposite sides)

2) Let’s assume the length of one side is 6 cm and the length of the adjacent side is 8 cm.

3) The perimeter of a rectangle is calculated by adding the lengths of all four sides.

4) Therefore, the perimeter of the rectangle would be (6 cm + 8 cm) x 2 = 28 cm..

3) Circle:

Solution. 1) A circle is a curved shape with a constant radius.

2) Let’s assume the radius of a circle is 3 cm.

3) The perimeter of a circle is called its circumference and is calculated using the formula: 2 × π × radius.

4) Therefore, the perimeter (circumference) of the circle would be

2 x 3.14 x 3 cm = 18.84 cm.

Please note that these are just examples, and the actual objects around you may have different dimensions and perimeters.

Haryana Board Class 6 Maths Solutions For Chapter 10 Mensuration Exercise 10.1

Question 1. Find the perimeter of each of the following figures:

Class 6 Maths Chapter 10 Mensuration Exercise 10.1 Question 1

Solution. a) Perimeter

5 cm + 1 cm + 2 cm + 4 cm

= 12 cm

b) Perimeter

= 40 cm + 35 cm + 23 cm + 35 cm

= 133 cm

c) Perimeter =

= 15 cm + 15 cm + 15 cm + 15 cm

= 60 cm

d) Perimeter = 4 cm + 4 cm + 4 cm + 4 cm + 4 cm = 20 cm

e) Perimeter = 1 cm + 4 cm + 0.5 cm + 2.5 cm + 2.5 cm + 0.5cm + 4cm = 15cm

f) Perimeter = 1 cm + 3 cm + 2 cm + 3 cm + 4 cm + 1 cm + 3 cm + 2 cm + 3 cm + 4 cm + 1 cm, + 3 cm + 2 cm + 3 cm + 4 cm + 1 cm + 3 cm + 2 cm + 3 cm + 4 cm = 52 cm

Circumference and area of a circle Class 6 Haryana Board

Question 2. The lid of a rectangular box of sides 40 cm by 10 cm is sealed all round with tape. What is the length of the tape required?

Solution. Length of rectangular box = 40 cm

Breadth of rectangular box = 10 cm

Length of the tape required

= 2 (length + breadth)

= 2(40 cm + 10 cm)

= 2 × 50 cm

= 100 cm

= 1 m

Question 3. A table top measures 2 m 25 cm by 1 m 50 cm. What is the perimeter of the table-top?

Solution. Length = 2 m 25 cm = 2.25 m

Breadth = 1 m 50 cm = 1.5 m

Perimeter of top of the table

= 2(length + breadth)

= 2(2.25 + 1.5) m.

= 2(3.75 m)

= 7.5 m.

Question 4. What is the length of the wooden strip required to frame a photograph of length and breadth 32 cm and 21 cm respectively?

Solution. Length = 32 cm

Breadth = 21 cm

Length of wooden strip required

= 2(length + breadth)

= 2(32 + 21) cm

= 2 x 53 cm

= 106 cm.

Question 5. A rectangular piece of land measures 0.7 km by 0.5 km. Each side is to be fenced with 4 rows of wires. What is the length of the wire needed?

Solution. Length = 0.7 km

Breadth = 0.5 km

Perimeter of a rectangle

= 2 (length + breadth)

= 2 (0.7 km + 0.5 km)

= 2.4 km.

Length of wire needed

= 4 × perimeter

= 4 × 2.4 km

= 9.6 km.

Mensuration Class 6 HBSE important questions

Question 6. Find the perimeter of each of the following shapes.

a) A triangle of sides 3 cm, 4 cm, and 5 cm.

Solution. Perimeter of the triangle

= 3 cm + 4 cm + 5 cm

= 12 cm

b) An equilateral triangle of side 9 cm.

Solution. Perimeter = 9 cm + 9 cm + 9 cm

= 27 cm

c) An isosceles triangle with equal sides 8 cm each and third side 6 cm.

Solution. Perimeter of an isosceles triangle

= 8 cm + 8 cm + 6 cm

= 22 cm

Question 7. Find the perimeter of a triangle with sides measuring 10 cm, 14 cm and 15 cm.

Solution. Perimeter of the triangle

= Sum of the lengths of sides

= 10 cm + 14 cm + 15 cm

= 39 cm.

Question 8. Find the perimeter of a regular hexagon with each side measuring 8 m.

Solution. Perimeter of regular hexagon

= 6 x length of side

= 6 × 8 m

= 48 m.

Question 9. Find the side of the square whose perimeter is 20 m.

Solution. Perimeter of square = 20 m

4 x length of the side = 20 m

Length of the side = \(\frac{20 \mathrm{~m}}{4}=5 \mathrm{~m}\)

Question 10. The perimeter of a regular pentagon is 100 cm. How long is its each side?

Solution. Perimeter of regular pentagon = 100 cm

5 x length of the side = 100 cm

Length of the side = \(\frac{100 \mathrm{~cm}}{5}=20 \mathrm{~cm} .\)

Question 11. A piece of string is 30 cm long. What will be the length of each side if the string is used to form:

a) a square?

b) an equilateral triangle ?

c) a regular hexagon?

Solution. a) Perimeter of a square = 30 cm

4 × length of side = 30 cm

length of side = \(\frac{30 \mathrm{~cm}}{4}=7.5 \mathrm{~cm}\)

b) Perimeter of equilateral triangle = 30 cm.

3 x length of side = 30 cm

Length of side = \(\frac{30 \mathrm{~cm}^{\prime}}{3}=10 \mathrm{~cm} .\)

c) Perimeter of regular hexagon = 30 cm

6 x length of side = 30 cm

Length of side = \(\frac{30 \mathrm{~cm}}{6}=5 \mathrm{~cm} .\)

Question 12. Two sides of a triangle are 12 cm and 14 cm. The perimeter of the triangle is 36 cm. What is the third side?

Solution. Perimeter of a triangle = Sum of the lengths of 3 sides

36 cm = 12 cm + 14 cm + length of third side

36cm = 26 cm + length of third side

∴ length of third side

= 36 cm – 26 cm = 10 cm

Question 13. Find the cost of fencing a square park of side 250 m at the rate of ₹ 20 per metre.

Solution. Perimeter of square park

= 4 x length of one side.

= 4 x 250 m

= 1000 m

∴ Cost of fencing at the rate of ₹ 20 per metre.

= ₹ 1000 x ₹ 20

= ₹ 20,000

Question 14. Find the cost of fencing a rectangular park of length 175 m and breadth 125 m at the rate of ₹ 12 per metre.

Solution. Perimeter of a rectangular park

= 2(length + breadth)

= 2(175 + 125)m

= 2(300)m

= 600 m

∴ Cost of fencing the rectangular park at the rate of ₹ 12 per metre.

= ₹ 12 × ₹ 600

= ₹ 7200

Question 15. Sweety runs around a square park of side 75 m. Bulbul runs around a rectangular park with length 60 m and breadth 45m. Who covers less distance?

Solution. Perimeter of a square park

= 4 x length of one side

= 4 x 75 m.

= 300 m.

Perimeter of a rectangular park

= 2(1 + b)

= 2 (60 m + 45 m)

= 2 (105 m)

= 210 m

here 300 m > 210 m

∴ Bulbul covers less distance.

Question 16. What is the perimeter of each of the following figure? What do you infer from the answers?

Class 6 Maths Chapter 10 Mensuration Exercise 10.1 Question 16.1

Solution. a) Perimeter = 25 cm + 25 cm + 25 cm +25 cm 100 cm.

b) Perimeter = 30 cm + 20 cm + 30 cm + 20 cm = 100 cm

c) Perimeter = 40 cm + 10 cm + 40 cm +10 cm = 100 cm

d) Perimeter = 40 cm + 30 cm + 30 cm = 100 cm

Inference: The perimeter of all the fig- ures is 100 cm.

Question 17. Avneet buys 9 square paving slabs, each with a side of \(\frac{1}{2}\) m. He lays them in the form of a square.

Class 6 Maths Chapter 10 Mensuration Exercise 10.1 Question 17

a) What is the perimeter of his arrangement in the above figure (1)?

b) Shari does not like his arrangement. She gets him to lay them out like a cross. What is the perimeter of her arrangement in the above figure (2)?

c) Which has greater perimeter?

d) Avneet wonders if there is a way of getting an even greater perimeter. Can you find a way of doing this? (The paving slabs must meet along complete edges i.e. they can- not be broken.)

Solution. a) Perimeter of the figure(1) = 4 x side

Length of the side of the square

= 3 x square

Slabs x length of each side

= 3 x 0.5 m = 1.5 m(each side = \(\frac{1}{2}\)m)

Perimeter of Avneet’s arrangement

= 4 x 1.5m = 6m.

b) Peimeter of the figure (2)

= 5 x length of each side

= 5 x 0.5 = 2.5 m (each side = \(\frac{1}{2}\)m)

Perimeter of the cross = 4 x figure

= 4 x 2.5 = 10 m

c) Shari’s arrangement has greater perimeter.

d) No, but we can achieve the same perimeter as shari’s arrangement by arranging all the squares in a row.

Haryana Board Class 6 Maths Solutions For Chapter 10 Mensuration Exercise 10.2

Question 1. Find the areas of the following figures by counting squares.

Class 6 Maths Chapter 10 Mensuration Exercise 10.2 Question 1.1

Solution. a)

Class 6 Maths Chapter 10 Mensuration Exercise 10.2 Question 1.2

b)

Class 6 Maths Chapter 10 Mensuration Exercise 10.2 Question 1.3

∴ Total area of the figure = 5 sq.cm

c) Fully-filled squares = 2

Half-filled squares = 4

Area covered by fully squares = 2 × 1 = 2 sq. cm

Area covered by half squared = 4 x \(\frac{1}{2}\) = 2 sq. cm

Total area = 4 sq.cm.

d) Fully-filled squares = 8

∴ Total area = Area covered by full squares

=8 × 1 sq.cm

= 8 sq. cm

e) Fully filled squares = 10

∴ Total area = Area covered by full squares

= 10 × 1 sq cm.

= 10 sq.cm.

f) Fully-filled squares = 2

Half-filled squares = 4

Area covered by fully squares = 2 x 1 sq.cm

= 2 sq.cm

Area covered by half squares = 4 x \(\frac{1}{2}\) sq.cm

= 2 sq. cm.

∴ Total area = 2 sq.cm + 2 sq.cm

= 4 sq.cm

g) Fully-filled squares = 4

Half-filled squares = 4

Area covered by full squares

= 4 x 1 sq. cm = 4 sq. cm

Area covered by half squares

= 4 x \(\frac{1}{2}\) sq. cm = 2 sq. cm

∴ Total area 4 sq. cm + 2 sq. cm

= 6 sq. cm.

h) Fully-filled squares = 5

∴ Total area = Area covered by full squares

= 8 x 1 sq. cm = 5 sq. cm

i) Fully-filled squares = 9

∴ Total area = Area covered by full squares

= 9 × 1 sq. cm

= 9 sq. cm

j) Fully-filled squares = 2

Half filled squares = 4

Area covered by full squares

= 2 x 1 sq. cm

= 2 sq. cm

Area covered by half squares

= 4 x \(\frac{1}{2}\) sq.cm = 2 sq.cm

∴ Total area 2 sq. cm + 2 sq. cm

= 4 sq. cm

k) Fully filled squares = 4

Half filled squares = 2

Area covered by full squares

= 4 × 1 sq. cm

= 4 sq. cm

Area covered by half squares

= 2 x \(\frac{1}{2}\) sq.cm

= 1 sq. cm

∴ Total area = 4 sq. cm + 1 sq. cm

= 5 sq. cm

1) Fully-filled squares = 4 = 4 sq.cm

Half-filled squares

= 2 = 2 x \(\frac{1}{2}\) = 1 sq.cm

More than half filled squares

= 3 = 3 x 1 sq cm = 3 sq.cm

∴ Total area

= 4 sq. cm + 3 sq. cm

= 7 sq.cm

m) Fully filled squares = 7 x 1 sq.cm

= 7 sq.cm

Half-filled squares = More than half filled squares

= 7 x 1 sq.cm = 7 sq.cm

Less than half filled squares = 4

Total area = 7 sq. cm + 7sq.cm

= 14 cm.

n) Fully filled squares = 9 x 1 = 9 sq.cm

Half-filled squares

More than half filled squares

= 9 x 1 = 9 sq.cm

Total area = 9 sq. cm + 9 sq.cm

= 18 sq.cm

Question 2. Find the area of the floor of your classroom.

Solution. Let’s assume the length of the classroom is 10 meters and the width is 8 meters.

Area = 10 meters x 8 meters = 80 sq.m.

So, the area of the floor of the classroom would be 80 sq m.

Question 3. Find the area of any one door in your house.

Solution. The most common standard door size is 80 inches (6.67) feet) in height and 36 inches (3 feet) in width area of door

= Height x Width

Area of door = 80 inches x 36 inches = 2880 sq. inches.

To convert square inches to square feet, divide the area by 144 (since there are 144 square inches in a square foot):

Area of door (in square feet) = 2880 square inches/144 = 20 square feet.

There fore, the approximate area of standard -size door in a house is around 20 square feet.

Haryana Board Class 6 Maths Solutions For Chapter 10 Mensuration Exercise 10.3

Question 1. Find the areas of the rectangles whose sides are:

a) 3 cm and 4 cm

b) 12 m and 21 m

c) 2 km and 3 km

d) 2 m and 70 cm

Solution. a) 1 = 4 cm, b = 3 cm

Area of rectangle = 1 x b

= 4 x 3

= 12 sq. cm.

b) 1 = 21 m, b = 12 m

Area of rectangle = 1 x b

= 21 × 12

= 252 sq.m.

c) 1 = 3 km, b = 2 km

Area of rectangle = 1x b

= 3 × 2

= 16 sq.km.

d) 1 = 2 m, b = 70 cm = 0.7 m

Area of rectangle = 1 x b

= 2 × 0.7

= 1.4 sq.m.

Question 2. Find the areas of the squares whose sides are:

a) 10 cm

b) 14 cm

c) 5 m

Solution. a) a = 10 cm

Area of the square a2 sq. cm

= 102 sq. cm

= 100 sq. cm

b) a = 14 cm

Area of the square = a2 sq. cm

= 142 sq. cm

= 196 sq. cm

c) a = 5 m

Area of the square = a2

= 52 sq.cm

= 25 sq.cm

Question 3. The length and breadth of three rectangles are as given below:

a)9 m and 6 m

b) 17 m and 3 m

c) 4 m and 14 m

Which one has the largest area and which one has the smallest ?

Solution. a) 1 = 9 m, b = 6m

Area of rectangle = l x b

= 9 x 6

= 54 sq.m.

b) 1 = 17 m, b = 3 m

Area of rectangle = 1 x b

= 17 × 3

= 51 sq. m.

c) 1 = 14 m, b = 4m

Area of rectangle

= 1 x b

= 14 × 4

= 56 sq.m

The rectangle(c) has the largest area and the rectangle (b) has the smallest area.

Word problems on Mensuration for Class 6 HBSE

Question 4. The area of a rectangular garden 50 m long is 300 sq.m. Find the width of the garden.

Solution. Area of a rectangular garden = 300 sq.m.

length = 50 m.

1 x b = 300

50 x b = 300

b = \(\frac{300}{50}\) m

b = 6m

∴ Width of the garden = 6m

Question 5. What is the cost of tiling a rectangular plot of land 500 m long and 200 m wide at the rate of 8 per hundred sq. m.?

Solution. Length of a rectangular piece of land = 500 m

Breadth of a rectangular piece of land = 200 m

Area = length x breadth

= 500 m × 200 m

= 1,00,000 sq.m.

Cost of tiling per 100sq. m = ₹ 8

Cost of tiling 1,00,000 sq. m

= \(₹ \frac{8}{100} \times 100000\)

= ₹ 8000

Question 6. A table-top measures 2 m by 1 m 50 cm. What is its area in square metres?

Solution. Length of the table = 2 m

Breadth of the table = 1 m 50 cm = 1.5 m

Area of the table = length x breadth

= 2 × 15 sq.m

= 3 sq.m.

Question 7. A room is 4m long and 3 m 50 cm wide. How many square metres of carpet is needed to cover the floor of the room?

Solution. Length of the room = 4 m = 4 m

Breadth of the room = 3 m 50 cm

= 3.50 m

∴ Area of the room

= length x breadth

= 4m x 3.50 m

= 14 sq.m.

∴ 14 sq. m of carpet is needed to cover the floor.

Question 8. A floor is 5m long and 4 m wide. A square carpet of sides 3 m is laid on the floor. Find the area of the floor that is not carpeted.

Solution. Length of the floor = 5 m

Breadth of the floor = 4 m

Area of the floor = length x breadth

= 5 x 4 sq. m

= 20 sq.m

Area of the square carpet = a2

= (3)2 sq. m

= 9 sq. m.

Area of the floor that is not carpeted

= 20 sq.m. – 9 sq.m

= 11 sq.m.

Question 9. Five square flower beds each of sides 1 m are dug on a piece of land 5 m long and 4 m wide. What is the area of the remaining part of land?

Solution. Area of square flower bed = a2 sq. m

=(1)2sq. m

= 1 sq. m

∴ Area of 5 square flower beds

= 1 x 5 sq. m

= 5 sq. m

Length of the piece of land = 5 m

Breadth of the piece of land

= length x breadth

= 4 x 5 sq. cm

= 20 sq. m

Area of remaining part of land

= 20 sq. m – 5 sq. m

= 15 sq. m

Question 10. By splitting the following figures into rectangles find their areas.

(The measures are given in centimetres)

Class 6 Maths Chapter 10 Mensuration Exercise 10.2 Question 10.1

Solution. a) Area of the figure

= (3 x 3) + (1 x 2) + (3 x 3) + (4 × 2) sq. cm

= 9 + 2 + 9 + 8 sq.cm

= 28 sq. cm

b) Area of the figure

= (3 x 1 + 3 x 1 + 3 x 1) sq. m

Class 6 Maths Chapter 10 Mensuration Exercise 10.2 Question 10.2

= 3 + 3 + 3 sq. cm

= 9 sq. cm

Question 11. Split the following shapes into rectangles and find their areas. (The measures are given in centimetres)

Solution. a)

Class 6 Maths Chapter 10 Mensuration Exercise 10.2 Question 11.1

Area of the shape

= (8 x 2 + 12 x 2) sq. cm

= 16 + 24 sq. cm

= 40 sq. cm

b)

Class 6 Maths Chapter 10 Mensuration Exercise 10.2 Question 11.2

Area of the shape

= (7×7) + (7×7) + (7×7) + (7×7) + (7×7) sq.m

=(49 + 49 + 49 + 49 + 49) sq. m

= 245 sq. cm

c)

Class 6 Maths Chapter 10 Mensuration Exercise 10.2 Question 11.3

Area of the shape

= (5 x 1)+(4 x 1) sq. cm

= (5 + 4) sq. cm

= 9 sq. cm

Question 12. How many tiles whose length and breadth are 12 cm and 15 cm respectively will be needed to fit in a rectangular region whose length and breadth are respectively.

a) 100 cm and 144 cm

Solution. 1 = 144 cm, b = 100 cm

Area of rectangle = 1 x b

= 144 × 100

= 14400 sq.cm

Length of the tile = 5 cm

Breadth of the tile = 12 cm

Area of rectangle = 1 x b

= 5 x 12 sq.cm

= 60 sq.cm

∴ Number of tiles needed to fit the

region = \(\frac{\text { Area of theregion }}{\text { Area of tile }}\)

= \(\frac{14400}{60}\) = 240.

b) 70 cm and 36 cm

Solution. 1 = 70 cm, b = 36 cm

Area of the region = 70 × 36 sq.cm = 2520 sq.cm

Area of the tile = 60 sq.cm

Number of tiles needed = \(\frac{2520}{60}\)

= 42 sq.cm

A challenge!

On a centimetre squared paper, make as many rectangles as you can, such that the area of the rectangle is 16 sq cm (consider only natural number lengths).

a) Which rectangle has the greatest perimeter?

b) Which rectangle has the least perimeter? If you take a rectangle of area 24 sq cm, what will be your answers? Given any area, is it possible to predict the shape of the rectangle with thegreatest perimeter? With the least perimeter? Give example and reason.

Solution. Three rectangles can be made as follows:

1) Sides 16 cm and 1 cm

Perimeter = 2(1 + b)

= 2(16 + 1) = 34 cm

2) Sides 8 cm and 2 cm

Perimeter = 2(l + b)

= 2(8 + 2) = 20 cm

3) Sides 4 cm and 4 cm

Perimeter = 2(l + b)

= 2(4 + 4) = 16 cm

a) The rectangle (1) has the greatest perimeter.

b) The rectangle (3) has the least perimeter. If the area is 24 sq. cm. Four rectangles can be made as follows.

1) Sides 24 cm and 1 cm

Perimeter = 2(1 + b)

= 2(24 + 1) = 50 cm

2) Sides 4 cm and 6 cm

Perimeter = 2(1 + b)

= 2(4 + 6) = 20 cm

3) Sides 8 cm and 3 cm

Perimeter = 2 (1 + b)

= 2(8 + 3) = 22 cm

4) Sides 12 cm and 2 cm

Perimeter = 2(l + b)

= 2(12 + 2) = 28 cm

Yes! it is possible to predict the shape of the rectangle with the (1) greatest perimeter. 2) least perimeter.

Difference between perimeter and area Class 6

Haryana Board Class 6 Maths Solutions For Chapter 10 Mensuration Very Short Answer Questions

Question 1. Find the perimeters of the given figures.

Class 6 Maths Chapter 10 Mensuration Very Short Answer Question 1

In the above figures (1) and (2), find the perimeter of ΔKLM, ΔKMN and ▢KLMN.

Solution. 1) Perimeter of ΔKLM

= KL + LM + KM

= 2 + 2.6 + 3.8 = 8.4 cm

Perimeter of ΔKMN

= KM + MN + KN

= 3.8 + 2 + 2.6 = 8.4 cm

2) Perimeter of ▢KLMN

= KL + LM + MN + KN

= 2 + 2.6 + 2 + 2.6 = 9.2 cm

Question 2. Find the perimeters of the following figures.

Class 6 Maths Chapter 10 Mensuration Very Short Answer Question 2

Solution. Perimeter of AXYZ

= XY + YZ + XZ

= 2 + 2 + 2 = 6 cm

2) Perimeter of a square ABCD

= AB + BC + CD + AD

= 3 + 3 + 3 + 3 = 12 cm

3) Perimeter of ▢PQRS

= PQ + QR + RS + PS

= 2 + 2 + 2 + 2 = 8 cm

Question 3. Find the area of a square with side 16 cm.

Solution.

Class 6 Maths Chapter 10 Mensuration Very Short Answer Question 3

Length of the side of a square = 16 cm

Area of a square = side x side

= 16 x 16 = 256 cm2

Question 4. Length and breadth of a rectangle are 16 cm and 12 cm respectively. Find its

Solution.

Class 6 Maths Chapter 10 Mensuration Very Short Answer Question 4

Length of a rectangle l = 16 cm

Its breadth = b = 12 cm

Area of a rectangle

= 1 x b

= 16 × 12

= 192 cm2

Question 5. Find the area of the rectangle of measurements 15cm and 8cm as length and breadth respectively.

Solution.

Class 6 Maths Chapter 10 Mensuration Very Short Answer Question 5

Length of a rectangle = 1 = 15 cm

Its breadth = b = 8 cm

Area of a rectangle = l x b

= 15 x 8

= 120 cm2

Question 6. Find the area of a square whose perimeter is 64m.

Solution. Perimeter of a square = 64 m

Perimeter of a square

= 4 x side(s)

= 4 x s = 64

s = \(\frac{64}{4}\) = 16

Class 6 Maths Chapter 10 Mensuration Very Short Answer Question 6

Side of a square(s) = 16 m

∴ Area of a square = s x s

= 16 x 16 = 256 m2

Question 7. Ritu went to a park 130 m long and 90 m wide. She took one complete round of it. What distance did she cover?

Solution.

 

Class 6 Maths Chapter 10 Mensuration Very Short Answer Question 7

Total distance covered by Ritu = Perimeter of the park ABCD

= AB + BC + CD + DA

= 130 + 90 + 130 + 90

= 440 m.

Question 8. Find the perimeter of given shape.

Solution.

Class 6 Maths Chapter 10 Mensuration Very Short Answer Question 8

IJ = DC = 3 cm; EF = HG = 2 cm

AB = LK = 4 cm; FG = KJ = CB = 1 cm

AL = BC + DE + FG + HI + JK

= 1 + 2 + 1 + 2 + 1 = 7 cm.

Perimeter

= AB + BC + CD + DE + EF + FG + GH + HI + IJ + JK + KL + LA

= 4 + 1 + 3 + 2 + 2 + 1 + 2 + 2 + 3 + 1 + 4 + 7 = 32 cm.

Question 9. Length and breadth of a rectangle are 15 cm. and 10 cm. Find the perimeter.

Solution. Length of the rectangle = 15 cm.

Breadth = 10 cm.

Perimeter of the rectangle

= 2 (length + breadth)

= 2 (15 + 10)

= 2 x 25 = 50 cm.

Question 10. Find the perimeter of a rectangular field which is 36 m. long and 24 m. wide.

Solution. Length of the rectangle field (1) = 36 m.

Breadth of the field (b) = 24 m.

∴ Perimeter of the field = 2(1 + b)

= 2(36 + 24)

= 2 x 60 = 120 m.

Haryana Board Class 6 Maths Solutions For Chapter 10 Mensuration Short Answer Questions

Question 11. Find the perimeter of

1) An equilateral triangle whose side is 3.5 cm.

Solution.

Class 6 Maths Chapter 10 Mensuration Short Answer Question 11.1

The length of the side of an equilateral triangle = 3.5 cm

Perimeter of an equilateral triangle

= 3 x length of the side

= 3 x 3.5 – 10.5 cm

2) A square whose side is 4.8 cm.

Solution.

Class 6 Maths Chapter 10 Mensuration Short Answer Question 11.2

The length of the side of a square = s = 4.8 cm

Perimeter of square

= 4 x length of the side

= 4 x 4.8 = 19.2 cm

Question 12. Length and breadth of top of one table is 160cm and 90cm respectively. Find how much length of beading is required for each table.

Solution.

Class 6 Maths Chapter 10 Mensuration Short Answer Question 12

A table is in the shape of a rectangle.

Length of a table = l = 160 cm

Its breadth = b = 90 cm

∴ The length of beading is required for each table

= Perimeter of the rectangular table.

= 2[1 + b]

= 2[160 + 90] = 2 × 250 = 500 cm

Question 13. Find the perimeter and area of a square of side 4 cm. Are these same? Give some examples to support your answer.

Solution. Length of the side of a square = 4 cm Perimeter of a square

= 4 x side = 4 x 4 = 16 cm

Area of a square = s2 = 42 = 16 cm2

In this condition perimeter and area of a square are equal.

For example length of the side be 6 cm.

Perimeter of a square

= 4 x s = 4 x 6 = 24 cm

Area of a square = s x s

= 6 × 6 = 36 cm

∴ Perimeter and area of a square need not be equal for all conditions.

Question 14. Find the area of the square whose perimeter is 48cm.

Solution. Perimeter of a square = 48 cm

Perimeter of a square = 4 × side

∴ 4 × side = 48

Side = \(\frac{48}{4}\) = 12

∴ Side of the square (s) = 12 cm

∴ Area of the square = s x s

= 12 x 12 = 144cm2

Haryana Board Class 6 Maths Solutions For Chapter 10 Mensuration Long Answer Questions

Question 15. Measurements of two rectangular fields are 50m x 30m and 60m x 40m. Find their perimeters. Check whether the perimeters are 2 x length + 2 x breadth.

Solution. Measurements of rectangle ABCD:

Length (l) Breadth (b)

Breadth (b)= 30 m

Class 6 Maths Chapter 10 Mensuration Long Answer Question 15.1

Perimeter of ▢ABCD

= AB + BC + CD + AD

= 50 + 30 + 50 + 30

= 160 m

Perimeter of ▢ABCD

=2 x length + 2 x breadth

= 2 (50) + 2 (30)

= 100 + 60

= 160 m

Yes, it is true.

Class 6 Maths Chapter 10 Mensuration Long Answer Question 15.2

Length of rectangle PQRS = l = 60 m

Breadth (b) = 40 m

Perimeter of ▢PQRS

= PQ + QR + RS + PS

= 60 + 40 + 60 + 40

= 200 m

Perimeter of ▢PQRS

= 2 x length + 2 x breadth

=2 x 60 + 2 x 40

= 120 + 80.

= 200 m

Yes, it is true.

Question 16. Manasa has 24 cm of metallic wire with her. She wanted to make some polygons with equal sides whose sides are integral without milling into pieces values. Find how many such polygons she can make with the metallic wire?

Solution. The length of the wire that Manasa has = 24 cm

She has to make some polygons with equal sides i.e., [squares] whose sides are integral without milling into pieces values.

∴ The smallest length of the side of a polygon [square],

she wants to make is (s) = 1 cm

Perimeter of one polygon [square]

= 4 x side

= 4 x 1 = 4 cm

∴ Number of polygons [square] that she can make with the length of 24 cm metallic wire

= \(\frac{\text { Length of the wire }}{\text { Perimeter of } 1 \text { square }}=\frac{24}{4}=6\)

Question 17. Find the perimeter of the following figures (1) and (2).

Class 6 Maths Chapter 10 Mensuration Long Answer Question 17.1

Solution. The perimeter of figure (1) ABCDEFGHIJKL

= AB + BC + CD + DE + EF + FG + GH + HI + IJ + JK + KL + AL

= 5 + 3 + 1 + 2 + 1 + 1 + 1 + 1 + 1 + 2 + 1 + 3 = 22 cm

Class 6 Maths Chapter 10 Mensuration Long Answer Question 17.2

 

The perimeter of figure (2) MNOPQRSTUVWX

= MN + NO + OP + PQ + QR + RS + ST + TU + UV + VW + WX + MX

= 1 + 2 + 1 + 1 + 1 + 1 + 1 + 2 + 1 + 1 + 5 + 1 = 18 cm

Question 18. If the length of a rectangle is 14cm and its perimeter is 3 times of its length. Find its area.

Solution. Length of a rectangle = 1 = 14cm

Its perimeter = 3 x length of a rectangle

= 3 x 14 = 42 cm

∴ Perimeter of a rectangle = 2 [1 + b]

= 2[14 + b] cm.

∴ By the problem 2[14 + b] = 42

14 + b = \(\frac{42}{2}\)

14+ b = 21

b = 21 – 14 = 7

∴ Breadth of a rectangle = b = 7 cm

∴ Area of a rectangle = 1 x b

= 14 x 7 = 98 cm2

Question 19. 14cm and 12cm are the length and breadth of a rectangle. If the breadth is increased by 6cm and length is decreased by 6cm, find the difference in areas.

Solution. Length of a rectangle = l = 14cm

Its breadth = b = 12 cm

Area of a rectangle = l x b

= 14 x 12 = 168 cm2

If the breadth is increased by 6 cm and length is decreased by 6 cm then so formed.

Length = l = (l – 6) cm;

breadth = b = (b + 6) cm = 14 – 6 = 8 cm

∴ Area of so formed rectangle

= l x b

12 + 6 = 18 cm

Area of new rectangle = 1 x b

= 8 × 18

= 144 cm2

∴ Difference in areas

= 168 cm2 – 144 cm2

= 24 cm2

Haryana Board Class 6 Maths Solutions For Chapter 10 Mensuration Objective Type Questions

Choose the correct answer:

Question 1. Perimeter of an equilateral triangle of side ‘s’ is

  1. 4s
  2. 2s
  3. 3s
  4. 6s

Answer. 3. 3s

Question 2. Perimeter of a regular hexagon of side 2 cm is

  1. 12 cm
  2. 10 cm
  3. 8 cm
  4. 12 sq.cm.

Answer. 1. 12 cm

Question 3. The space occupied by a closed figure is called its

  1. area
  2. perimeter
  3. segment
  4. point

Answer. 1. area

Question 4. The perimeter of triangle with sides \(\overline{A B}\), \(\overline{B C}\), \(\overline{C A}\)

  1. \(\overline{A B}\) + \(\overline{B C}\)
  2. \(\overline{A B}\) + \(\overline{B C}\) + \(\overline{C A}\)
  3. \(\overline{B C}\) + \(\overline{C A}\)
  4. A + B + C

Answer. 2. \(\overline{A B}\) + \(\overline{B C}\) + \(\overline{C A}\)

Question 5. The number of sides of a octagon is

  1. 6
  2. 7
  3. 10
  4. 8

Answer. 4. 8

Question 6. If length and breadth of a rectangle is 12 cm and 8 cm respectively, then its perimeter is ….

  1. 20 cm
  2. 96 cm
  3. 40 sq.m.
  4. 40 cm

Answer. 4. 40 cm

Question 7. Area of a rectangle

  1. l x b
  2. 2(l + b)
  3. 4s
  4. s2

Answer. 1. l x b

Question 8. A = s2 is the formula for

  1. Area of a rectangle
  2. Area of a square
  3. Perimeter of a square
  4. Perimeter of a rectangle

Answer. 2. Area of a square

Question 9. The sum of all the lengths of a polygon is called ………..

  1. area
  2. perimeter
  3. volume
  4. none

Answer. 2. perimeter

Question 10. How many sides are there in a pentagon?

  1. 2
  2. 9
  3. 6
  4. 5

Answer. 4. 5

Question 11. In a rectangle l = 3 cm, b = 0.9 cm then A = …… cm2

  1. 9
  2. 27
  3. 2.7
  4. 3

Answer. 3. 2.7

Question 12. Side of an equilateral triangle is 3 cm then its perimeter is ………. cm.

  1. 3.5
  2. 11
  3. 12
  4. 9

Answer. 4. 9

Question 13. Perimeter of rectangle is …..

  1. \(\frac{2}{3}\)(l + b)
  2. \(\frac{l+b}{2}\)
  3. 2(l + b)
  4. \(l+\frac{\mathrm{b}}{2}\)

Answer. 3. 2(l + b)

Question 14. Perimeter of a square is 20 cm then its side is ………. cm.

  1. 10
  2. 5
  3. 4
  4. 7

Answer. 2. 5

Question 15. Identify the odd one from the following

  1. length
  2. breadth
  3. area
  4. perimeter

Answer. 3. area

Question 16. P = 2(l + b) then l = ……

  1. \(\frac{\mathrm{P}}{2}+\mathrm{b}\)
  2. \(\frac{P}{2}-l\)
  3. \(\frac{P}{2}-b\)
  4. \(\frac{P}{2}-2b\)

Answer. 3. \(\frac{P}{2}-b\)

Question 17. Formula to find area of square of side ‘a’ units is ………unit2.

  1. a
  2. 2a
  3. \(\frac{a}{2}\)
  4. a2

Answer. 4. a2

Question 18. The perimeter of regular hexagon of side 6 cm is ……….. cm.

  1. 20
  2. 36
  3. 15
  4. 16

Answer. 2. 36

Question 19. 1 m = ………. cm.

  1. 50
  2. 60
  3. 70
  4. 100

Answer. 4. 100

Question 20. The cost of fencing a square park of side 150 m at the rate of ₹ 10 per meter is

  1. 6000
  2. 5000
  3. 1100
  4. 1700

Answer. 1. 6000

Question 21. In a rectangle l = 36 m, b = 24 m, then P = …..m.

  1. 114
  2. 103
  3. 120
  4. 110

Answer. 3. 120

Question 22. Side of a square is 17 cm then its area is ……… cm2.

  1. 423
  2. 324
  3. 189
  4. 289

Answer. 4. 2889

Question 23. The area of a rectangle is 1,125 cm2. If its breadth is 25 cm then length = ….. cm.

  1. 25
  2. 40
  3. 15
  4. 45

Answer. 4. 45

Question 24. Perimeter of a regular pentagon of side 8 cm is ….. cm.

  1. 60
  2. 40
  3. 80
  4. 70

Answer. 2. 40

Question 25. ……… are the simple closed plane figures bounded by line segments.

  1. Area
  2. Perimeter
  3. Square
  4. Polygons

Answer. 4. Polygons

Question 26. The perimeter of a triangle is 30 cm. Sum of its two sides is 21 cm, then the length of 3rd side is …….. cm.

  1. 13
  2. 12
  3. 9
  4. 11

Answer. 3. 9

Question 27. Area measured in ……….units.

  1. square
  2. cubic
  3. Both A & B
  4. none

Answer. 1. square

Question 28. If the perimeter of a square is 48 cm then its side

  1. 12 cm
  2. 24 cm
  3. 6 cm
  4. 144 cm

Answer. 1. 12 cm

Question 29. If the area of a square is 144 cm2 then its perimeter is

  1. 12 cm
  2. 48 cm
  3. 24 cm
  4. 36 cm

Answer. 2. 48 cm

Question 30. If the side of a square is doubled then its area will be increased by

  1. doubled
  2. 3 times
  3. 4 times
  4. None

Answer. 3. 4 times

Question 31. If the side of a square is halved then its area will be

  1. same as original area
  2. halved
  3. 4 times increases
  4. \(\frac{1}{4}\) of original area

Answer. 4. \(\frac{1}{4}\) of original area

Question 32. If the length and breadth of rectangle are doubled then its area will be

  1. same as original area
  2. halved
  3. doubled
  4. increased by 4 times

Answer. 4. increased by 4 times

Question 33. If the length is doubled and breadth is tripled then its area will be

  1. doubled
  2. increased by 3 times
  3. increased by 6 times
  4. same as original area

Answer. 3. increased by 6 times

Question 34. A rectangular plot of land is 240 m by 200 m. The cost of fencing per meter is ₹ 10. What is the cost of fencing the entire field?

  1. ₹ 8800
  2. ₹ 8000
  3. ₹ 4800
  4. ₹ 8880

Answer. 1. ₹ 8800

Question 35. A piece of wire is 60 cm long. What will be the length of each side if the string is used to form and equilateral triangle

  1. 10 cm
  2. 15 cm
  3. 20 cm
  4. 30 cm

Answer. 3. 20 cm

Question 36. The perimeter of a rectangle whose lengths is ‘l’ meters and breadth is ‘b’ meters is ………… meters.

  1. l + b
  2. lb
  3. 2(l + b)
  4. 2l + b

Answer. 3. 2(l + b)

Question 37. The perimeter of a rectangle whose sides are 18 cm and 15 cm is ….. cm.

  1. 38
  2. 60
  3. 66
  4. 36

Answer. 3. 66

Question 38. A farmer has to cover three times the perimeter of field. The length is 10 m and the breadth is 8 m then total distance travelled by him is …… m.

  1. 108
  2. 100
  3. 18
  4. 36

Answer. 1. 108

Question 39. Perimeter of a square whose side is ‘a’ meters is ………… meters.

  1. a4
  2. 4 + a
  3. 2a
  4. 4a

Answer. 4. 4a

Question 40. The perimeter of a regular pentagon whose side is 5 cm is …… cm.

  1. 28
  2. 25
  3. 20
  4. 29

Answer. 2. 25

Question 41. The area of rectangle whose sides are 3 m and 4 m is …….. sq.m.

  1. 12
  2. 13
  3. 34
  4. 7

Answer. 1. 12

Question 42. The area of a square whose side is 4 cm is …….. sq.cm.

  1. 4
  2. 16
  3. 24
  4. 8

Answer. 2. 16

Question 43. The area of rectangle is 20 sq.cm and the length is 4 cm then the breadth is ….. cm.

  1. 10
  2. 4
  3. 3
  4. 5

Answer. 4. 5

Question 44. The area of rectangle whose measurements are given by length = 3 m and breadth = 40 cm is ……. sq.m.

  1. 340
  2. 120
  3. 1.2
  4. 12

Answer. 3. 1.2

Chapter 10 Mensuration Fill in the blanks:

Question 45. The perimeter of a square whose length is 4.5 cm ………..cm

Answer. 18

Question 46. The perimeter of an equilateral triangle is 30 cm then its side is ……..cm

Answer. 10

Question 47. The area of square is 12544 cm2 then its side is……cm

Answer. 112

Question 48. The perimeter of a rectangle is 48 cm, its length is 20 cm then its breadth = …………..cm.

Answer. 4

Question 49. The area of adjacent rectangle PQRS is ……………cm2.

Class 6 Maths Chapter 10 Mensuration Fill in the Blanks Question 51

Answer. 36

Question 50. The cost of fencing a square park of side 250 m. at the rate of 20 per meter is.

Answer. 20,000

Question 51. The area of a square is 144 cm2 then its perimeter is ………..cm

Answer. 48

Question 52. The perimeter of given figure is………cm

Class 6 Maths Chapter 10 Mensuration Fill in the Blanks Question 54

Answer. 8.4

Question 53. The perimeter of a square is 40 cm., then its side is.

Answer. 10 cm.

Question 54. The length of the boundary of closed figure is called its

Answer. perimeter

Question 55. Area of a rectangle is 100 cm2 its length and breadth are equal then its perimeter is………..

Answer. 40 cm.

Question 56. The perimeter of a square is 24 cm., then its area is …….

Answer. 36 cm2.

Question 57. The perimeter of a regular hexagon of side 8 cm., is……..

Answer. 48 cm.

Question 58. Which is different among 15 cm., 16 cm., 19 cm.2, 18 cm.

Answer. 19 cm2.

Question 59. The side of a square is 6 cm. The length and breadth of rectangle are 7 cm. and 5 cm. respectively. Which has less area?………..

Answer. rectangle

Question 60. Which is different among “length, breadth, side, perimeter, area”………

Answer. area

Question 61. 2(1+b) P is the formula for …………….

Answer. perimeter of a rectangle

Question 62. Which is different among 16 cm2, 17 cm2, 18 cm2, 20 cm2 ……

Answer. 18 cm.

Question 63. The sum of the lengths of line segments of the polygon is ………..

Answer. perimeter of the polygon

Question 64. If the area of a rectangular field is 1350 m2 and its length is 50 m then its breadth is ………

Answer. 27 m.

Question 65. If the length of the side of a square is 10 m. Then 100 square mts is its …….

Answer. area

Question 66.

Class 6 Maths Chapter 10 Mensuration Fill in the Blanks Question 68

What is the relationship between these shapes?

Answer. perimeters are equal.

Question 67. The amount of surface enclosed by a closed figure is called its ……

Answer. area

Question 68. The perimeter of the regular hexagon is 18 cm then side is ……. cm.

Answer. 3

Question 69. The area of square whose side is 4 cm. is…………………….cm.

Answer. 16

Question 70. The area in square metre of a piece of cloth 1 m 25 cm wide and 2.m long is …….cm2.

Answer. 25

Question 71. The perimeter of an equilateral triangle whose side is 4 cm is…….cm.

Answer. 12

Question 72. The perimeter of a triangle whose sides are 3 cm, 4 m and 6 cm is ………….. cm.

Answer. 13

Question 73. The perimeter of the rectangle given in the figure is ………. cm

Class 6 Maths Chapter 10 Mensuration Fill in the Blanks Question 75

Answer. 12

Haryana Board Class 6 Maths Solutions For Chapter 10 Mensuration Match The Following:

Question 74.

Class 6 Maths Chapter 10 Mensuration Match the following Question 76

Answer. 1 – D, 2 – A, 3 – B, 4 – F, 5 – E, 6 – C.

Haryana Board Class 6 Maths Solutions For Chapter 9 Data Handling

Haryana Board Class 6 Maths Solutions For Chapter 9 Data Handling

  • ‘Data’ is a collection of numbers gathered to give some information.
  • ‘Information’ is either in the form of numbers or words.
  • To get a particular information from the given data quickly, the data can be arranged in a tabular form using tally marks.
  • Data that has been organised and presented in ‘frequency distribution tables’ can also be represented using ‘pictographs’ and ‘bar graphs’.
  • Representation of the data in the form of pictures, objects or parts of objects is called ‘pictograph’ or ‘pictogram’.
  • A pictograph uses pictures or symbols to represent the frequency of the data.
  • In pictograph the size of all pictures should be the same and should represent the same number of items of the data.
  • We use tally marks for recording and organisation of data.
  • Representation of data can be done in the following ways (1) Pictographs (2) Bar graphs.
  • The pictographs represents the representation of the data through pictures.
  • “P.C. Mahalanobis” is known as father of Indian Statistics.

Haryana Board Class 6 Maths Solutions For Chapter 9 Data Handling Exercise 9.1

Question 1. In a Mathematics test, the following marks were obtained by 40 students. Arrange these marks in a table using tally marks.

Class 6 Maths Chapter 9 Data Handling Exercise 9.1 Question 1.1

a) Find how many students obtained marks equal to or more-than 7.

b) How many students obtained marks below 4?

Solution.

Class 6 Maths Chapter 9 Data Handling Exercise 9.1 Question 1.2

a) 5 + 4 + 3 = 12 students obtained marks equal to or more than

7 (7 marks – 5 students, 8 marks – 4 students; 9 marks – 3 students)

b) 3 + 3 + 2 = 8 students obtained marks below 4.

Haryana Board Class 6 Maths Data Handling solutions

Question 2. Following is the choice of sweets of 30 students of Class VI: Ladoo, Barfi, Ladoo, Jalebi, Ladoo, Rasgulla, Jalebi, Ladoo, Barfi, Rasgulla, Ladoo, Jalebi, Jalebi, Rasgulla, Ladoo, Rasgulla, Jalebi, Ladoo, Rasgulla, Ladoo, Ladoo, Barfi, Rasgulla, Rasgulla, Jalebi, Rasgulla, Ladoo, Rasgulla, Jalebi, Ladoo.

a) Arrange the names of sweets in a table using tally marks.

b) Which sweet is preferred by most of the students?

Solution. a)

Class 6 Maths Chapter 9 Data Handling Exercise 9.1 Question 2

b) Ladoo is preferred by most of the students.

Question 3. Catherine threw a dice 40 times and noted the number appearing each time as shown below:

Class 6 Maths Chapter 9 Data Handling Exercise 9.1 Question 3.1

Make a table and enter the data using tally marks. Find the numbers that appeared:

a) The minimum number of times.

b) The maximum number of times.

c) Find those numbers that appear an equal number of times.

Solution.

Class 6 Maths Chapter 9 Data Handling Exercise 9.1 Question 3.2

a) Number 4 appeared minimum number of times.

b) Number 5 appeared maximum number of times.

c) Number 1.and 6 appeared equal number of times.

Question 4. Following pictograph shows the number of tractors in five villages.

Class 6 Maths Chapter 9 Data Handling Exercise 9.1 Question 4

Observe the pictograph and answer the following questions.

1) Which village has the minimum number of tractors?

Solution. Village D has the minimum number of tractors – 3.

2) Which village has the maximum number of tractors?

Solution. Village C has the maximum number of tractors – 8.

3) How many more tractors village C has as compared to village B ?

Solution. Village C has 8 – 5 = 3 more tractors as compared to village B.

4) What is the total number of tractors in all the five villages?

Solution. Total number of tractors in all the five villages = 6 + 5 + 8 + 3 + 6 = 28.

Question 5. The number of girl students in each class of a co-educational middle school is depicted by the pictograph.

Class 6 Maths Chapter 9 Data Handling Exercise 9.1 Question 5

Observe this pictograph and answer the following questions:

a) Which class has the minimum number of girl students?

Solution. Class VIII has the minimum no. of girl students (6).

b) Is the number of girls in class VI less than the number of girls in Class V ?

Solution. No, the number of girls (16) in class VI is not less than the number of girls (10) in class V.

c) How many girls are there in class VII?

Solution. Number of girls in class VII = 3 x 4 = 12.

How to draw a pictograph Class 6 HBSE

Question 6. The sale of electric bulbs on different days of a week is shown below:

Class 6 Maths Chapter 9 Data Handling Exercise 9.1 Question 6

Observe the pictograph and answer the following questions:

a) How many bulbs were sold on Friday.

Solution. 14 bulbs were sold on Friday.

b) On which day were maximum number of bulbs sold?

Solution. Maximum bulbs were sold on Sunday.

c) On which of the days same number of bulbs were sold?

Solution. Equal no. of bulbs were sold on Wednesday and Saturday.

d) On which of the days minimum number of bulbs were sold?

Solution. Minimum bulbs were sold on Wednesday and Saturday.

e) If one big carton can hold 9 bulbs. How many cartons were needed in the givenweek?

Solution. No. of bulbs can hold in one big carton = 9

No.of cartons were need = 86 ÷ 9

= 10 (Approx)

Question 7. In a village six fruit merchants sold the following number of fruit baskets in a particular season:

Class 6 Maths Chapter 9 Data Handling Exercise 9.1 Question 7

Observe this pictograph and answer the following questions:

a) Which merchant sold the maximum number of baskets?

Solution. Martin sold the maximum number of baskets – 950.

b) How many fruit baskets were sold by Anwar?

Solution. 7 x 100 = 700 fruit baskets were sold by Anwar.

c) The merchants who have sold 600 or more number of baskets are planning to buy a godown for the next season. Can you name them?

Solution. Anwar, Martin and Ranjit Singh are planning to buy a godown for the next season.

Haryana Board Class 6 Maths Solutions For Chapter 9 Data Handling Very Short Answer Questions

Question 1. Give two examples of data in numerical figures.

Solution. Example: 1

The number of students in each class in a school:

Class 6 Maths Chapter 9 Data Handling Very Short Answer Question 1.1

Example: 2

The details of number of cricket bats sold in a week:

Class 6 Maths Chapter 9 Data Handling Very Short Answer Question 1.2

Question 2. Give two examples of data in words.

Solution. Example: 1

Days of a week

MON, TUE, WED, THU, FRI, SAT, SUN.

Example: 2

Important cities in Andhra Pradesh.

Vijayawada, Guntur, Tirupathi, Visakhapatnam, Machilipatnam, Nellore, Kakinada.

Question 3. Vehicles that crossed a checkpost between 10 AM and 11 AM are as follows:

car, lorry, bus, lorry, auto, lorry, lorry, bus, auto, bike, bus, lorry, lorry, zeep, lorry, bus, zeep, car, bike, bus, car, lorry, bus, lorry, bus, bike, car, zeep, bus, lorry, lorry, bus, car, car, bike, auto.

Represent the data in a frequency distribution table using tally marks.

Solution.

Class 6 Maths Chapter 9 Data Handling Very Short Answer Question 3

Frequency table and data interpretation Class 6

Question 4. The following pictograph shows the number of students use cycles, in five classes of a school.

Class 6 Maths Chapter 9 Data Handling Very Short Answer Question 4

Answer the following questions based on the pictograph given above –

1) Which class students have the maximum number of cycles?

2) Which class students have the minimum number of cycles?

3) Which class students have 9 cycles?

4) What is the total number of cycles in all the five classes?

Solution. 1) IX class students have the maximum number of cycles.

2) VI class students have the minimum number of cycles.

3) VIII class students have 9 cycles.

4) Total number of cycles in all the five classes:

VI      5

VII     10

VIII     9

IX       12

X         7

Total   43

Question 5. A book-shelf has books of different subjects. The number of books of each subject is represented as a pictograph given below. Observe them.

Class 6 Maths Chapter 9 Data Handling Very Short Answer Question 5

Answer the following questions:

1) Which books are more in number?

Solution. Maths

2) Which books are least in number?

Solution. English

3) How many total books are there ?

Solution.

Telugu      4

English     3

Hindi        4

Maths       6

Science     5

Social        4

Total         26

Question 6. 26 students in a class got the following marks in an assignment – 5, 6, 7, 5, 4, 2, 2, 9, 10, 2, 4, 7, 4, 6, 9, 5, 5, 4, 3, 7, 9, 5, 2, 4, 5, 7. The assignment was for 10 marks.

1) Organise the data and represent in the form of a frequency distribution table using tally marks.

2) Find out the marks obtained by maximum number of students.

3) Find out how many students received least marks.

4) How many students got 8 marks?

Solution. 1)

Class 6 Maths Chapter 9 Data Handling Very Short Answer Question 6

2) Maximum number of students (6) got 5 marks.,

3) Least mark (2) was obtained by 4 students.

4) No student in the class got 8 marks.

How to create a bar graph Class 6 Haryana Board

Question 7. In a class of 25, students like various games. The details are shown in the following pictograph. (No student plays more than one game).

Class 6 Maths Chapter 9 Data Handling Very Short Answer Question 7

1) How many students play badminton?

2) Which game is played by most number of students ?

3) What is the game in which least number of students are interested?

4) How many students do not play any game?

Solution. 1) 5 students play badminton.

2) Kabaddi is played by most number of students i.e., 7.

3) Tennikoit is played by least number of students i.e., 4.

4) Total number of players = 7 + 4 + 5 + 6 = 22

Number of students in the classroom = 25.

Thus, the number of students who do not play any game = 25 – 22 = 3

Question 8. Give two examples of data in numerical figures.

Solution. 1) The population of Andhra Pradesh in (2011) – 4,96,34,314

2) The cost of I phone 11 pro-1,21,889.

Question 9. Give two examples of data in words.

Solution. 1) The cost of Maruti Brezza [Z x 1] price on road in India – Nine lakh ten thousand rupees only.

2) The population of India according to census 2011 – One hundred and twenty one crore eight lakhs fifty four thousand nine hundred and seventy seven.

Haryana Board Class 6 Maths Solutions For Chapter 9 Data Handling Short Answer Questions

Question 10. A child’s Kiddy bank is opened and the coins collected are in the following denomination.

Class 6 Maths Chapter 9 Data Handling Short Answer Question 10.1

Represent the data in a frequency distribution table using tally marks.

Solution.

Class 6 Maths Chapter 9 Data Handling Short Answer Question 10.2

Question 11. The favourite colours of 25 students in a class are given below:

Blue, Red, Green, White, Blue, Green, White, Red, Orange, Green, Blue, White, Blue, Orange, Blue, Blue, White, Red, White, White, Red, Green, Blue, Blue, White.

Write a frequency distribution table using tally marks for the data. Which is the least favourite colour for the students ?

Solution.

Class 6 Maths Chapter 9 Data Handling Short Answer Question 11

colour for the students = 2 (Orange)

Question 12. A TV channel invited a SMS poll on ‘Ban of Liquor’ giving options:-

A – Complete ban

B – Partial ban

C – Continue sales

They received the following SMS, in the first hour-

Class 6 Maths Chapter 9 Data Handling Short Answer Question 12.1

Represent the data in a frequency distribution table using tally marks.

Solution.

Class 6 Maths Chapter 9 Data Handling Short Answer Question 12.2

Question 13. The number of wrist watches manufactured by a factory in a week are as follows:

Class 6 Maths Chapter 9 Data Handling Short Answer Question 13.1

Represent the data using a pictograph. Choose a suitable scale.

Solution. = 50 watches.

Class 6 Maths Chapter 9 Data Handling Short Answer Question 13.2

Question 14. Votes polled for various candidates in a sarpanch election are shown below, against their symbols in the following table.

Class 6 Maths Chapter 9 Data Handling Short Answer Question 14.1

Represent the data using a pictograph. Choose a suitable scale.

Answer the following questions:

1) Which symbol got least votes?

2) Which symbol candidate won in the election?

Solution. Each represents 50 votes

Class 6 Maths Chapter 9 Data Handling Short Answer Question 14.2

1) Watch symbol got least votes.

2) Pot symbol candidate won in the election.

Question 15. The sale of television sets of different companies on a day is shown in the pictograph given below. Scale: = 5 televisions

Class 6 Maths Chapter 9 Data Handling Short Answer Question 15

Answer the following questions:

1) How many TVs of company A were sold?

2) Which company’s TVs are people more crazy about?

3) Which company sold 15 TV sets?

4) Which company had the least sale?

Solution. 1) Number of TV’s sold by company A = 5 x 5 = 25

2) People more crazy about the TV company ‘C’.

3) Company ‘E’ sold 15 TV sets.

4) Company ‘D’ sold least sale.

Haryana Board Class 6 Maths Solutions For Chapter 9 Data Handling Long Answer Questions

Question 16. Given below are the ages of 20 Students of Class VI in a School.

13, 10, 11, 12, 10, 11, 11, 13, 12, 11, 10, 11, 12, 11, 13, 11, 10, 13, 10, 12

1) Organise the data and represent in the form of a frequency distribution table using tally marks.

Solution.

Class 6 Maths Chapter 9 Data Handling Long Answer Question 17

2) Find out the age having more number of students

Solution. More number of students having the age is 11 years.

3) How many students are there in 10 Years age?

Solution. 5 students

4) Find out number of students who are having more age?

Solution. Number of students who are having more age 13 years is 4.

Question 17. A die was thrown 30 times and following scores were obtained

5 3 4 6 2 3

6 2 2 3 15

2 5 4 6 2 1

4 5 1 6 2 1

3 1 3 3 4 6

Word problems on pictographs and bar graphs Class 6

1) Prepare a frequency table of the scores.

Solution.

Class 6 Maths Chapter 9 Data Handling Long Answer Question 18

2) Which number obtained more times?

Solution. The numbers 2 and 3 obtained more times ie., 6.

3) How many times was a score greater than 4 obtained?

Solution. The number of times a score greater than 4 obtained was 4 + 5 = 9

4) Find the total number of times an odd number obtained.

Solution. Total number of times an odd number obtained is = 5 + 6 + 4

= 15 [1, 3 and 5 are faces of a dice]

Haryana Board Class 6 Maths Solutions For Chapter 9 Data Handling Objective Type Questions

Choose the correct answer:

Question 1. The information collected in terms of numbers

theorem

data

mean

range

Answer. 2. data

Question 2. Representing the data in terms of pictures is

Bar graphs

Bar charts

Pictographs

Pie charts

Answer. 3. Pictographs

Question 3. While drawing pictures in a pictograph all ………… must be equal.

lengths

sizes

breadths

bases

Answer. 2. sizes

Question 4. five || indicates ….

7

6

3

5

Answer. 1. 7

Question 5. If a * represents 3 cars, how many * are required to indicate 18 cars?

3

6

8

12

Answer. 2. 6

Question 6. If 6 cm represent 9 lakh people then the scale is ………..

1 cm = 1,50,000

2 cm = 1,00,000

1 cm = 2,00,000

1 cm = 1,20,000

Answer. 1. 1 cm = 1,50,000

Question 7. In a data maximum value = 30 and the minimum value is 10. Then the range is

31

12

20

16

Answer. 3. 20

Question 8. 13 can be represented by tally marks as

fivefive

fivefive|||

five||

fivefivefive|

Answer. 2. fivefive|||

Question 9. According to the scale \(\frac{1}{2}\) cm = 15 students then 45 students = …… cm

8

4

7

\(\frac{3}{2}\)

Answer. 4. \(\frac{3}{2}\)

Question 10. ………. are used in preparing frequency table

Symbols

Tally marks

Data

Scale

Answer. 2. Tally marks

Data Handling examples for Class 6 HBSE Maths

Question 11. Which of the following indicates ‘5’

|||||

five

|||

fivefive

Answer. 2. Diagram

Question 12. Average of 1, 2, 3 is

  1. 2
  2. 3
  3. 8
  4. 3.5

Answer. 1. 2

Question 13. Statement – a: A table showing the frequency or count of various items is called a frequency distribution.

Statement – b: To get a particular information from the given data quickly, the data can be arranged in a tabular form using tally marks.

  1. both a & b are true
  2. a is true b is false
  3. a is false, b is true
  4. both a & b are false

Answer. 1. both a & b are true

Question 14. Father of Indian Statistics

  1. Ramanujan
  2. Aryabhata
  3. P.C.Mahalanobis
  4. Bhaskaracharya

Answer. 3. P.C.Mahalanobis

Question 15. If represents = 10 books then represent = …….books.

  1. 10
  2. 20
  3. 30
  4. 40

Answer. 4. 40

Answer the questions [16-19] from the information given below.

Class 6 Maths Chapter 9 Data Handling Choose the correct answer 15

Question 16. How many marks obtained by maximum number of students?

  1. 5
  2. 6
  3. 2
  4. 10

Answer. 1. 5

Question 17. How many students received least marks?

  1. 1
  2. 2
  3. 4
  4. 10

Answer. 3. 4

Question 18. How many students got 8 marks?

  1. 1
  2. 4
  3. 21
  4. 0

Answer. 4. 0

Question 19. How many students got more than 8 marks?

  1. 3
  2. 4
  3. 8
  4. 21

Answer. 2. 4

Question 20. If represents 5 eggs, then how many eggs do represent?

  1. 20
  2. 25
  3. 100
  4. 30

Answer. 2. 25

Question 21. If represents 5 balloons, then the number of symbols to be drawn to represent 70 balloons is

  1. 12
  2. 10
  3. 15
  4. 14

Answer. 4. 14

Question 22. The number of members in 20 families in a village is given as 6, 8, 6, 3, 2, 5, 7, 8, 6, 5, 5, 7, 7, 8, 6, 6, 7, 7, 6, 5.

How many families are of the largest size?

  1. 4
  2. 3
  3. 5
  4. 6

Answer. 2. 3

Question 23. The number of times an observation occurs is called

  1. frequency
  2. data
  3. tallymarks
  4. none

Answer. 1. frequency

Haryana Board Class 6 Maths Solutions For Chapter 9 Data Handling Fill in the blanks:

Question 24. Diagram represents ……

Answer. 5

Question 25. 1 + 3 + 4 + 5 + 6 + 7 + 8 + 10 = ……….

Answer. 44

Question 26. ……………… collection of numbers gathered to give some information.

Answer. Data

Question 27. Average of -2, -1, 0, 1, 2, … ………….

Answer. 0

Question 28. According to a scale \(\frac{1}{2}\) cm = 20 students then 60 students

Answer. 3/2

Question 29. Range of first 10 whole numbers is ………

Answer. 9

Question 30. Range of first 100 even natural numbers is = …….

Answer. 98

Question 31. Show 12 as tally mark…………….

Answer. fivefive||

Question 32. Collection of certain information is called …….

Answer. data

Question 33. Representing the data in the form of pictures is called ………

Answer. pictograph

Question 34. If Diagram represents 150 then Diagram represent …….

Answer. 750

Question 35. If 100 books = Diagram then 150 books are represented by ……….

Answer. ▢▢ ▢

Question 36. The tally marks fivefive ||| represent ………..

Answer. 13

Question 37. Which are used in preparing frequency table are ……..

Answer. Tally marks

Question 38. The tally marks to represent 16……………..

Answer. fivefivefive|

Haryana Board Class 6 Maths Solutions For Chapter 6 Integers

Haryana Board Class 6 Maths Solutions For  Chapter 6 Integers

  • The collection of all positive numbers, zero and negative numbers together are called ‘Integers’.
    The set of Integers is denoted by the symbol Z. Z = {………….. -4, -3, -2, -1, 0, 1, 2, 3, 4 …}
  • The number without any sign is considered positive number only.
    Eg: 3 is considered + 3
  • Brahmagupta (598 AD-670 AD) first used a special sign (-) for negative numbers and stated the rules for dealing with positive and negative quantities.
  • The letter “Z” was first used by the Germans because the word for Integers in the Ger- man language is “Zehlen”, which means “Number”.
  • Natural numbers, (1,2,3,4,…) are also called positive integers and Whole numbers. {0,1,2,3,4,…) are also called non-negative integers.
    Negative numbers are important to represent just opposite numbers for positive numbers.
  • Every integer can be represented on a number line horizontally or vertically.
  • Addition and subtraction of integers can be done on number line.
  • Every negative integer is always lesser than zero.
  • The smallest and the biggest positive integers are +1 and positive infinite (+∞).
  • The smallest and the biggest negative integers are negative infinite (-∞) and -1
  • The sum of two negative integers is always a negative integer. Eg: -2 + (-3) = -5
  • The sum of two positive integers is always a positive integer. Eg: 5 + 7 = 12
  • The sum of two integers, one of which is positive and the other is negative, then the sum may be either positive, negative or zero.
    Eg: 12 + (-9) = 3;-12 + 9 = -3;-3 + 3 = 0
  • For any integer a, -(-a) = a
  • We add the opposite of an integer (additive inverse) to an integer in subtraction.
  • Integers are widely used in our daily life i.e., business, engineering, games, temperatures, medicine etc……
  • Integers are closed under addition and sub- traction.
  • Integers are commutative and associative under addition.
  • Integers are not commutative and associative under subtraction.
  • We use negative numbers to represent debit, temperature below the 0°C, past periods of time depth below sea level.
  • Number Line:

Class 6 Maths Chapter 6 Integers Number Line

  • The numbers on the left side of zero (i.e., less than zero) are called ‘negative numbers’.
  • The numbers on the right side of zero (i.e., greater than zero) are called ‘positive numbers’.
  • We can show the addition and subtraction of integers on the number line.
  • ‘O’ is neither positive nor negative.
  • Any two distinct numbers that give zero when added to each other are additive inverse of each other.
    Eg. : 3 and -3
    3 + (-3) = 0
  • The subtraction of integers is the same as the addition of their additive inverse.
    Eg. The additive inverse of 7 is -7
    The additive inverse of -8 is 8
  • Numbers with a negtative sign are called negative numbers. They are less than zero.
  • If we add ‘1’ (one) to a given number we get successor.
  • If we subtract ‘one’ from a given number we get predecessor.
  • If we move right side of a given number on a number line the value of the number increases and to the left the value decreases.
  • If we add a positive number and a negative number we must do subtraction and put the sign of the bigger number.
  • The addition of two integers is zero then each one is called additive inverse of the other.

Suppose David and Mohan have started walking from zero position in opposite directions. Let the steps to the right of zero be represented by ‘+’ sign and to the left of zero represented by “” sign. If Mohan moves 5 steps to the right of zero it can be represented as +5 and if David moves 5 steps to the left of zero it can be represented as -5. Now represent the following positions with +or – sign:

a) 8 steps to the left of zero.

Answer. -8

b) 7 steps to the right of zero.

Answer. +7

c) 11 steps to the right of zero.

Answer. +11

d) 6 steps to the left of zero.

Answer. -6

Write the succeeding number of the following:

Class 6 Maths Chapter 6 Integers Successor

Write the preceeding number of the following:

Class 6 Maths Chapter 6 Integers Predecessor

Write the following numbers with appropriate signs:

a. 100 m below sea level.

Solution. -100 m

b. 25°C above 0°C temperature.

Solution. + 25°C

c. 15°C below 0°C temperature.

Solution. -15°C.

d. Five numbers less than 0.

Solution. -1, -2, -3, -4, -5

Mark 3, 7,-4,-8,- 1 and 3 on the number line.

Solution.

Class 6 Maths Chapter 6 Integers Mark Number LIne

Compare the following pairs of numbers using > or <.

0 _ 8

-1 _ -15

5 _ -5

11 _ 15

0 _ 6

– 20 _ 2

Solution.

0 > 8

-1 > -15

5 > -5

11 < 15

0 < 6

-20 < 2

Haryana Board Class 6 Maths Solutions For  Chapter 6 Integers Exercise – 6.1

Question 1. Write the opposites of the following:

a) Increase in weight

b) 30 km north

c) 80 m east

d) Loss of Rs. 700

e) 100 m above sea level

Solution. a) decrease in weight

b) 30 km south

c) 80 m west

d) Profit of Rs. 700

e) 100 m below sea level

Haryana Board Class 6 Maths Integers solutions

Question 2. Represent the following numbers as integers with appropriate signs.

a) An aeroplane is flying at a height, two thousand metre above the ground.

Solution. +2000 metre

b) A submarine is moving at a depth, eight hundred metre below the sea level.

Solution. -800 metere

c) A deposit of rupees two hundred.

Solution. +Rs. 200

d) Withdrawal of rupees seven hundred.

Solution. -Rs. 700

Question 3. Represent the following numbers on a number line.

a) +5

Class 6 Maths Chapter 6 Integers +5

b) -10

Class 6 Maths Chapter 6 Integers -10

c) +8

Class 6 Maths Chapter 6 Integers +8

d) -1

Class 6 Maths Chapter 6 Integers -1

e) -6

 

Class 6 Maths Chapter 6 Integers -6

Addition and subtraction of integers Class 6 HBSE

Question 4. Adjacent figure is a vertical number line, representing integers. Observe it and locate the following points:

Class 6 Maths Chapter 6 Integers Exercise 6.1 Question 4

a. If the point D is + 8, then which point is -8?

Solution. Point F.

b. Is point G is a negative integer or a positive integer?

Solution. It is a negative integer.

c. Write integers for points B and E.

Solution. Integer for point B = +4

Integer for point E = -10.

d. Which point marked on this number line has the least value?

Solution. Point E.

e. Arrange all the points in decreasing order of value.

Solution. D, C, B, A, O, H, G, F, E.

Question 5. Following is the list of temperatures of five places in India, on a particular day of the

Class 6 Maths Chapter 6 Integers Exercise 6.1 Question 5

a) Write the temperatures of these places in the form of integers in blank column.

Solution.

 

Class 6 Maths Chapter 6 Integers Exercise 6.1 Question 5.1

b) Following is the number line representing the temperature in degree celsius.

Plot the name of the city against its temperature.

Class 6 Maths Chapter 6 Integers Exercise 6.1 Question 5.2

c) Which is the coolest place?

Solution. Siachin is the coolest place.

d) Write the names of the places where temperatures are above 10° C.

Solution. Delhi and Ahmedabad.

Question 6. In each of the following pairs, which number is to the right of the other on the number line?

a) 2,9

Solution. The number 9 is to the right of the number 2.

b) -3, -8

Solution. The number -3 is to the right of the number -8.

c) 0,-1

Solution. The number ‘0’(zero) is to the right of the number -1.

d) -11, 10

Solution. The number 10 is to the right of the number -11.

e) -6,6

Solution. The number 6 is to the right of the number -6.

f) 1, -100

Solution. The number 1 is to the right of the

Multiplication and division of integers Class 6

Question 7. Write all the integers between the given pairs (write them in the increasing or

a. 0 and -7.

Solution. -6,-5,-4,-3,-2 and -1.

b. -4 and 4.

Solution. -3, -2, -1, 0, 1, 2 and 3.

C. -8 and -15.

Solution. -14,13,12,-11,-10 and -9.

d. 30 and -23.

Solution. -29,-28,-27,-26,-25 and -24.

Question 8. a. Write four negative integers greater than 20.

Solution. 19, -18,-17 and – 16..

b. Write four negative integers less than -10.

Solution. 11, 12, 13, and – 14.

Question 9. For the following statements, write True (T) or False (F). If the statement is false, correct the statement.

a. -8 is to the right of -10 on a number line.

Solution. True (T)

b. -100 is to the right of -50 on a number line.

Solution. False (F)

c. Smallest negative integer is -1.

Solution. False (F)

d. -26 is greater than -25.

Solution. False (F)

Question 10. Draw a number line and answer the following:

a. Which number will we reach if we move 4 numbers to the right of – 2?

Solution. We reach the number ‘2’.

b. Which number will we reach if we move 5 numbers to the left of 1?

Solution. We reach the number – 4.

c. If we are at 8 on the number line, in which direction should we move to reach – 13?

Solution. We should move in the left direction.

d. If we are at 6 on the number line, in which direction should we move to reach – 1?

Solution. We should move in the right direction.

Question 11. Draw a figure on the ground in the form of a horizontal number line as shown below. Frame questions as given in the said example and ask your friends.

Class 6 Maths Chapter 6 Integers Question 1

1) Which is greater (-2) or (-1)?

Solution. 1.

2) Which is smaller among – 6 and 3?

Solution. -6.

3) Which is the nearest positive integer to zero ?

Solution. 1.

4) Which integer is neither positive nor negative ?

Solution. ‘O’ is neither positive nor negative.

5) How many negative numbers you will find on left side of zero?

Solution. Infinite

6) Write three integers greater than -2.

Solution. -1,0,1.

7) Write five integers lesser than -1.

Solution. -2,-3,-4,-5, -6.

8) Which is the nearest positive integer to -1?

Solution. +1.

Integer number line Class 6 Haryana Board

Question 12. Take two different coloured buttons like white and black. Let us denote one white button by (+1) and one black button by (-1). A pair of one white button (+1) and black button (-1) will denoted zero i.e. [1 + (-1) = 0]

In the following table, integers are shown with the help of coloured buttons.

Class 6 Maths Chapter 6 Integers Question 2

Let us perform additions with the help of the coloured buttons. Observe the following table and complete it.

Class 6 Maths Chapter 6 Integers Question 2.1

Question 13. Find the answers of the following additions:

a. (-11)+(-12)

Solution. (-11) + (-12)

= -11 – 12 = -23.

b. (+10) + (+4)

Solution. 10 + 4 = 14.

c. (32) + (-25)

Solution. -32 + (-25)

= -32 – 25 = -57.

d. (+23) + (+40)

Solution. 23 + 40 = 63.

Question 14. Find the solution of the following:

a. (-7) + (+8)

Solution. -7 + (+8)

= -7 + 7 + 1 = 1.

b. (-9) + (+13)

Solution. (-9) + (+13)

= -9 + 9 + 4 = 4.

c. (+7) + (-10)

Solution. +7 + (-10) = +7 – 10

[∴ -10 = -7 – 3]

= 7 – 7 – 3 = -3.

d. (+12) + (-7)

Solution. (+12) + (-7) = 12 – 7

[∴ 12 = 5 + 7]

= 5 + 7 – 7 = 5.

Question 15. Find the solution of the following additions using a number line:

a) (-2) + 6

Solution.

Class 6 Maths Chapter 6 Integers (-2) + 6

∴ 2 + 6 = 4

b. (-6) + 2

Solution.

Class 6 Maths Chapter 6 Integers (-6) + 2

∴ -6 + 2 = -4

Question 16. Make two such questions and solve them using the number line.

Solution. 1) (-5) + 3

Class 6 Maths Chapter 6 Integers (-5) + 3

∴ 5 + 3 = -2

2) -3 + 5

Class 6 Maths Chapter 6 Integers -3 + 5

Question 17. Find the solution of the following without using number line.

a) (+7) + (-11)

Solution. 7 – 11 = -4

b) (-13) + (+10)

Solution. 13 + 10 = -3.

c) (-7)+(+9)

Solution. -7 + 9 = 2.

d) (+10)+(-5)

Solution. 10 – 5 = 5.

Make five such questions and solve them.

Question 18. Find the solution of (-7) + (+3) without using the number line.

Solution. (-7) + (+3) = (-4)

Question 2. Find the solution of (+15) + (-9) without using the number line.

Solution. (+15) + (-9) = (+6)

Question 3. Find the solution of (-20) + (+20) without using the number line.

Solution. (-20) + (+20) = 0

Question 4. Find the solution of (+11) + (-11) without using the number line.

Solution. (11) + (-11) = 0

Question 5. Find the solution of (-5) + (+7) without using the number line.

Solution. (-5) + (+7) = (+2)

Haryana Board Class 6 Maths Solutions For  Chapter 6 Integers Exercise 6.2

Question 1. Using the number line write the integer which is:

a) 3 more than 5.

Solution.

Class 6 Maths Chapter 6 Integers Exercise 6.2 Question 1.1

∴ 3 more than 5 is 8.

Word problems on integers for Class 6 HBSE

b) 5 more than -5.

Solution.

Class 6 Maths Chapter 6 Integers Exercise 6.2 Question 1.2

∴ 5 more than -5 is ‘0’.

c) 6 less than 2.

Solution.

Class 6 Maths Chapter 6 Integers Exercise 6.2 Question 1.3

∴ 6 less than 2 is -4.

d) 3 less than -2.

Solution.

Class 6 Maths Chapter 6 Integers Exercise 6.2 Question 1.4

∴ 3 less than 2 is -5.

Question 2. Use number line and add the following integers :

a) 9 + (-6)

Solution.

Class 6 Maths Chapter 6 Integers Exercise 6.2 Question 2.1

∴ 9 – 6 = 3

b) 5 + (-11)

Solution.

Class 6 Maths Chapter 6 Integers Exercise 6.2 Question 2.2

∴ 5 – 11 = -6.

c) (-1) + (-7)

Solution.

Class 6 Maths Chapter 6 Integers Exercise 6.2 Question 2.3

∴ -1 – 7 = -8.

d) -5 + 10

Solution.

Class 6 Maths Chapter 6 Integers Exercise 6.2 Question 2.4

∴ -5 + 10 =5 .

e) (-1) + (-2) + (-3)

Solution.

Class 6 Maths Chapter 6 Integers Exercise 6.2 Question 2.5

∴ -1 – 2 – 3 = -6.

f) (-2) + 8 + (-4)

Solution.

Class 6 Maths Chapter 6 Integers Exercise 6.2 Question 2.6

∴ – 2 + 8 – 4 = 8 – 6 = 2.

Question 3. Add without using number line:

a) 11 + (-7).

Solution. 11 + (-7)

= 11 – 7

= 4 + 7 – 7 [∴ 11 = 4 + 7]

= 4 + 0 = 4

b) (-13) + (+18)

Solution. (-13) + (+18)

= -13 + 18

= -13 + 13 + 5

[∴ 18 = 13 + 5]

= 0 + 5

= 5

c) (-10) + (+19)

Solution. (-10) + (+19) = -10 + 19

= -10 + 10 + 9

= 0 + 9

= 9

d) (-250) + (+150)

Solution. -250 + 150 = -150 – 100 + 150

[∴ – 250 = -150 – 100]

= -150 + 150 – 100

= 0 – 100

= -100

e) (-380) + (-270)

Solution. (-380) + (-270)

= -380 – 270

= -650

f) (-217)+(-100)

Solution. (-217) + (-100)

= -217 – 100

= -317

Question 4. Find the sum of:

a) 137 and -354

Solution. 137 + (-354) = 137 – 137 – 217

[∴ -354 = -137 – 127]

= 0 – 217

= – 217

b) -52 and 52

Solution. -52 + 52 = 0

c) -312, 39 and 192.

Solution. -312, 39 and 192.

= -300 – 12 + 39 + 192

[∴ -312 – 300 – 12]

= -300 – 12 + 39 + 180 + 12

= -300 + 219 – 81.

d) -50 – 200 + 300

Solution. -50 – 200 + 300

= -250 + 300

= -250 + 250 + 50

[∴ 300 = 250 + 50]

= +50

= 50

Question 5. Find the sum?

a) (-7) + (-9) + 4 + 16

Solution. (-7) + (-9) + 4 + 16

= -16 + 20

= -16 + 16 + 4 (20 = 16 + 4)

= 0 + 4 – 4

b) (37) + (-2) + (-65) + (-8)

Solution. 37 + (-2) + (-65) + (-8) = 37 – 75

= 37 – 75

= -38

Chapter 6 Integers Exercise 6.3

Question 1. Find:

a) 35-(20)

Solution. 35 – 20 = 15 + 20 – 20 (35 = 15 + 20)

= 15 + 0

= 15

b) 72 – (90)

Solution. 72 – 90

= 72 – 72 – 18 [∴ -90 = -72 – 18]

=0 – 18

= -18

c) (-15) – (-18)

Solution. -15 – (-18)

= -15 + 18

= -15 + 15 + 3 [∴ 18 = 15 + 3]

= 0 + 3

= 3

d) (-20)-(13)

Solution. -20 – 13

= -33

e) 23 (12)

Solution. 23 – (-12)

= 23 + 12 = 35 [∴ – x – = +]

f)(-32) – (-40)

Solution. (-32) – (-40)

= -32 + 40

= -32 + 32 + 8

[∴ 40 = 32 + 8]

= 0 + 8

= 8.

Question 2. Fill in the blanks with >, < or = sign.

a) (-3)+(-6) ____ (-3)-(-6)

Solution. L.H.S. = -3 – 6 = -9

R.H.S. = -3 – (-6)

= -3 + 6 = 3

∴ (-3) + (-6) < (-3) – (-6)

b) (-21) – (-10) ____ (31) + (-11)

Solution. L.H.S. = -21 – (-10) = -21 + 10 = -11

R.H.S. = -31 + (-11) = -31 – 11 = -42

= (-21) – (-10) > (-31) + (-11)

c) (45)-(-11) ____ 57 + (-4).

Solution. L.H.S. = 45 – (-11) = 45 + 11 = 56

R.H.S. = 57 + (-4) = 57 – 4 = 53

= 45 – (-11) > 57 + (-4)

d) (-25)-(-42) ____ (-42)-(-25)

Solution. L.H.S. = -25 – (-42) = -25 + 42.

= -25 + 25 + 17

= 0 + 17 = 17

R.H.S. = (-42)-(-25) = -42 + 25

= -17 – 25 + 25

= -17 – 0 = -17

∴ (-25) – (-42) > (-42) – (-25)

Question 3. Fill in the blanks:

a) (-8) + ____ = 0

Solution. (-8) + 8= 0

b) 13 + ____ = 0

Solution. 13 + (-13) = 0

c) 12 + (-12) = ____

Solution. 12 + (-12) = 0

d) (-4) + ____ = -12

Solution. (-4) + (-8) = -12.

e) ____ -15 = -10

Solution. (+5) – 15 – 10

Question 4. Find:

a) (-7) – 8 – (-25)

Solution. (-7) -8 -(-25) = 7 – 8 + 25

= -7 – 8 + 7 + 18 [∴ 25 = 7 + 18]

= -8 + 18

= -8 + 8 + 10

= 0 + 10 = 10

b) (-13) + 32 – 8 – 1

Solution. (-13) + 32 – 8 – 1

= -13 + 32 – 9

= -13 + 23 + 9 – 9 [∴ 32 = 23 + 9]

= -13 + 23

= -13 + 13 + 10

= 0 + 10

= 10

c) (-7)+(-8)+(-90)

Solution. (-7) + (-8) + (-90)

= -7 – 8 – 90

= -15 – 90

= -105

d) 50 (-40)-(-2).

Solution. 50 – (-40) – (-2)

50 + 40 + 2 = 90 + 2

= 92

Haryana Board Class 6 Maths Solutions For  Chapter 6 Integers Very Short Answer Questions

Question 1. Put appropriate symbol > or < in the boxes given between the two integers.

1) – 1………. 0

Solution. -1 < 0

2) -3 …….. -7

Solution. -3 > -7

3)-10 ……. +10

Solution. -10 < +10

Question 2. The temperature recorded in Shimla is -4° c and in Kufri is -6°c on the same day. Which place is colder on that day? Why?

Solution. Temperature recorded in Shimla = -4°C;

Kufri = -6°C

Class 6 Maths Chapter 6 Integers Very Short Answer Question 2

Comparing these: -6 < -4

∴ Kufri is colder than Shimla.

Question 3. Find the sum of:

1) 120 and -274

Solution. The sum of 120 and -274 is

= 120 + (-274)

= -154

2) -68 and 28

Solution. The sum of -68 and 28 is

= -68 + 28

= -40

Question 4. Simplify

1) (-6) + (-10) + 5 + 17

Solution. -6 + (-10) + 5 + 17 = -16 + 22

= 6

2) 30 + (-30) = (-60) + (-18)

Solution. 30 + (-30) + (-60) + (-18)

= [30+(-30)] + [(-60)+(-18)]

= 0 + (-78)

= -78

Question 5. Fill in boxes with <, > or = sign.

1) (-4) + (-5) ____ (-5) – (-4)

Solution.-9 < -1

2) (-16)-(-23) ____ (-6) + (-12)

Solution. 7 > -18

3) 44-(-10) ____ 47+ (-3)

Solution. 54 > 44

Question 6. Fill in the blanks:

1) (-13) + …… = 0

Solution. (-13) + 13 = 0

2) (-16) + 16 =

Solution. (-16) + 16 = 0

3) (-5) + …………. = -14

Solution. (-5) + (-9) = -14

4) ……..+ (2 – 16) = -22

Solution. -8 + (2 – 16) = -22 [∴ 2 – 16 = -14]

Question 7. Arrange the following integers in ascending order and descending order. -1000, 10,-1,-100, 0, 1000, 1, -10

Solution. Ascending order: -1000, -100, -10, -1, 0, 10, 1000 (From smallest to greatest)

Descending order: 1000, 10, 0, 1-, -10, – 100,-1000 (From greatest to smallest)

Question 8. The temperature in Nainital is -5° C and in Shimla is -10° C. In which place is the temperature higher ?

Solution.

Class 6 Maths Chapter 6 Integers Very Short Answer Question 8

The temperature in Nainital is -5°C and temperature in Shimla is -10°C – 5°C is greaterthan -10°C.

∴ The temperature in Nainital is higher than the temperature in Shimla.

Question 9. Simplify (-7)+(-12)+5+12

Solution. (-7) + (-12) + 5 + 12 = [-7 – 12] + [5 + 12]

= -19 + 17

= -2

Question 10. Find 25 + (-21) + (-20) + 17 + (-1).

Solution. 25 + (-21) + (-20) + 17 + (-1)

= 25 + 17 + [-21 – 20 – 1]

= 42 + [-42]

= 42 – 42 = 0

Haryana Board Class 6 Maths Solutions For  Chapter 6 Integers Short Answer Questions

Question 11. Represent the integers on a number line as given below.

1) Integers lie between -7 and -2.

2) Integers lie between -2 and 5.

Solution. 1) The integers lie between -7 and -2 are -6, -5, -4, -3

Class 6 Maths Chapter 6 Integers Short Answer Question 11.1

2) The integers lie between -2 and 5 are -1, 0, 1, 2, 3, 4

Class 6 Maths Chapter 6 Integers Short Answer Question 11.2

Question 12. Write the following integers in increasing and decreasing order:

1) -7,5,-3

Solution. Increasing order: -7,-3,5

Decreasing order: 5, -3, -7

2) -1,3,0

Solution. Increasing order: -1,0,3

Decreasing order: 3, 0, -1

3) 1,3, -6

Solution. Increasing order: -6, 1, 3

Decreasing order: 3, 1, -6

4) -5, -3, -1

Solution. Increasing order: -5, -3, -1

Decreasing order: -1, -3, -5

Question 13. Write True or False, Correct those that are false:

1) Zero is on the right of -3

Solution. True

2) -12 and +12 represent the same integer on the number line.

Solution. False; Correct answer : -12 is a negative integer and +12 is a positive integer.

3) Every positive integer is greater than zero.

Solution. True

4) (-100) > (+100)

Solution. False; Correct answer: -100 < +100.

Question 14. Find the value of the following using a numberline.

1) (-3) + 5

2) (-5) + 3

Make your own two new questions and solve them using the number line.

Solution. 1) (-3) + 5

Class 6 Maths Chapter 6 Integers Short Answer Question 14.1

First move 3 steps to the left of 0 reach – 3 and then from that point we move 5 steps to the right. We reach the number.

∴ (-3) + 5 = 2

Solution. 2) (-5) + 3

Class 6 Maths Chapter 6 Integers Short Answer Question 14.2

First move 5 steps to the left of 0 reach – 5 and then from that point we move 3 steps to the right. We reach the number – 2.

∴ (-5) + 3 = -2.

Question 15. Add without using number line.

1) 10 + (-3)

2) (-10) + (+16)

3) (-8) + (+8)

Solution.

1) 10+ (-3)

= (7 + 3) + (-3)

= 7 + [3+(-3)]

= 7 + 0 = 7

2) (-10) + (+16)

= (-10) + 10 + 6

= [-10 + 10] + 6

= 0 + 6 = 6

3) (-8) + (+8) = 0

Chapter 6 Integers Long Answer Questions

Question 16. Find all integers which lie between the given two integers. Represent them on number line.

1) -1 and 1

2) -5 and 0

3) -6 and -8

4) 0 and -3

1) -1 and 1

Solution. The integer between -1 and 1 is 0.

Class 6 Maths Chapter 6 Integers Long Answer Question 16.1

2) -5 and 0

Solution. The integers between -5 and 0 are -4, -3, -2, -1.

Class 6 Maths Chapter 6 Integers Long Answer Question 16.2

3) -6 and -8

Solution. The integer between -6 and -8 is -7.

Class 6 Maths Chapter 6 Integers Long Answer Question 16.3

4) 0 and -3

Solution. The integers between 0 and -3 are -1, -2

Class 6 Maths Chapter 6 Integers Long Answer Question 16.4

Properties of integers Class 6 HBSE Maths

Question 17. Add the following integers using number line.

(1) 7 + (-6)

(2) (-8) + (-2)

(3) (-6)+(-5)+(+2)

1) 7 + (-6)

Solution.

Class 6 Maths Chapter 6 Integers Long Answer Question 17.1

First move to the right of 0 by 7 steps to each 7, then move 6 steps to the left of 7 we reach 1.

∴ 7 + (-6) = 1

2) (-8) + (-2)

Solution.

Class 6 Maths Chapter 6 Integers Long Answer Question 17.2

First move to the left of 0 by 8 steps to reach -8, then again move 2 steps to the left of -8 we reach -10.

3) (-6)+(-5) + (+2)

Solution.

Class 6 Maths Chapter 6 Integers Long Answer Question 17.3

First move to the left of 0 by 6 steps to reach – 6.

Again move 5 steps to the left of -6 we reach -11.

Now move 2 steps to the right of -11 we reach -9.

∴ (-6) + (-5) + (+2) = -9.

Question 18. In a quiz competition,where negative score for wrong answer is taken,Team A scored +10, -10, 0, -10, 10,-10 and Team B scored 10,10, -10,0,0,10 in 6 rounds successively. Which team wins the competition? How?

Solution. In a quiz, the score made by team A are +10, -10, 0, -10, 10, -10-

The total score made by team A

= 10 + (-10) + 0 + (-10) + 10 + (-10) = -10

The score made by team B are 10, 10, -10, 0, 0, 10

The total score made by team B

= 10 + 10 + (-10) + 0 + 0 + 10 = 20
Team B scored more than team A.

∴ Team B wins the quiz competition.

Question 19. Find the values of (1) (-3)+(-8)+(-5); (2) (-1)+7+ (-3) using number line.

1) (-3) + (-8) + (-5)

Solution.

Class 6 Maths Chapter 6 Integers Long Answer Question 19.1

First move to the left of 0 by 3 steps to reach – 3.

Again move 8 steps to the left of -3 we reach -11.

Now move 5 steps to the left of -11 we reach -16.

∴ (-3) + (-8) + (-5) = -16

2) (-1) + (7) + (-3)

Solution.

Class 6 Maths Chapter 6 Integers Long Answer Question 19.2

First move to the left of 0 by 1 steps to reach – 1.

Again move 7 steps to the right of -1 we reach 6.

Now move 3 steps to the left of -6 we reach 3.

∴ (-1) + 7 + (-3) = 3

Question 20. Represent the following statements using signs of integers :

1) An aeroplane is flying at a height of 3000 meters.

Solution. +3000 meters (height)

2) The fish is 10 meters below the water surface.

Solution. -10 meters (depth)

3) The temperature in Vijayawada is 35°C above 0°C.

Solution. +35° C (+ more),

4) Water freezes at 0°C temperature.

Solution. 0° C (neither ‘+’ nor ‘-‘)

5) The average temperature at the mount Everest in January is 36°C below zero degree.

Solution. -36° C (less)

6) The submarine is 500 meters below the surface of the sea.

Solution. -500 meters (below)

7) The average temperature at Darjeeling in July is 19°C below zero degree.

Solution. -19° C (less)

8) The average low temperature in Vishakapatnam during January is 18°C.

Solution. -18° C (more)

Haryana Board Class 6 Maths Solutions For  Chapter 6 Integers Objective Type Questions

Choose the correct answer :

Question 1. Set of integers is denoted by

  1. N
  2. Z
  3. W
  4. Q

Answer. 2. Z

Question 2. Write all the integers lie between 0 and -3

  1. 1, 2
  2. -4, -5
  3. -1, -2
  4. 4, 5

Answer. 3. -1, -2

Question 3. The sum of two negative integers is always

  1. negative
  2. positive
  3. 0
  4. None of these

Answer. 1. negative

Question 4. The sum of two integers one of which is positive and the other is negative then the sum may be

  1. positive
  2. negative
  3. zero
  4. any one of the above

Answer. 4. any one of the above

Question 5. Subtract +5 from (-5)

  1. 0
  2. 10
  3. -10
  4. 5

Answer. 3. -10

Question 6. 18 – 18 + 1 = …..

  1. -4
  2. -1
  3. 3
  4. 1

Answer. 4. 1

Question 7. -2 … 1

  1. =
  2. <
  3. >

Answer. 3. <

Question 8. Sum of 10 and -16 is …….

  1. -6
  2. 6
  3. 12
  4. -26

Answer. 1. -6

Question 9. Choose the correct matching:

Class 6 Maths Chapter 6 Integers Choose The Correct Answer Question 9

  1. 1 – d, 2 – b, 3 – c, 4 – a
  2. 1 – b, 2 – d, 3 – a, 4 – c
  3. 1 – b, 2 – d, 3 – c, 4 – a
  4. 1 – a, 2 – d, 3 – b, 4 – c

Answer. 2. 1 – b, 2 – d, 3 – a, 4 – c

Question 10. The lest negative integer is

  1. 1
  2. -1
  3. -7
  4. does not exist

Answer. 4. does not exist

Question 11. Subract -8 from 8

  1. 19
  2. 0
  3. 11
  4. 16

Answer. 4. 16

Question 12. -4 – 4 + (-4) + (-4) =

  1. -16
  2. -12
  3. 13
  4. 84

Answer. 1. -16

Question 13. Additive inver of 7 – 3 is

  1. 4
  2. -4
  3. 8
  4. -7

Answer. 2. -4

Question 14. -20 – 82 = …

  1. 111
  2. -200
  3. 100
  4. -102

Answer. 4. -102

Question 15. Integer between -1 and + 1

  1. 1
  2. 2
  3. 0
  4. 4

Answer. 3. 0

Question 16. -131 + ……… = 0

  1. -13
  2. 131
  3. 117
  4. 814

Answer. 2. 131

Question 17. 38 – (-40) = …….

  1. 31
  2. 2
  3. 16
  4. 78

Answer. 4. 78

Question 18. -3 – (-3) = ………

  1. -9
  2. 0
  3. 1
  4. -2

Answer. 2. 0

Question 19. -8 + (-9) + (+17) =

  1. -3
  2. 11
  3. -1
  4. 0

Answer. 4. 0

Question 20. The smallest among -7, 0, 1, 5 is

  1. 1
  2. 5
  3. 0
  4. -7

Answer. 4. -7

Question 21. The greater negative integer in the following is

  1. -1
  2. -4
  3. -7
  4. -6

Answer. 1. -1

Question 22. …… is neither positive nor negative.

  1. 3
  2. 1
  3. 7
  4. 0

Answer. 4. 0

Question 23. Identify true statement from the following

  1. -7 is to the left of 0
  2. -3 is to the right of 1
  3. -7 – 3 = -6
  4. -1, -2, -3, -4 ….. are positive integers.

Answer. 1. -7 is to the left of 0

Question 24. The fish is 20 meters below the water surface can be denoted as

  1. 21 m
  2. -10 m
  3. -20 m
  4. 20 m

Answer. 3. -20 m

Question 25. Number of integers between 2017 and 2018 is

  1. 0
  2. 1
  3. 3
  4. does not exist

Answer. 1. 0

Question 26. -6 – 13 + 19 = …….

  1. 10
  2. -11
  3. 0
  4. 8

Answer. 3. 0

Question 27. ….. + (-16) = 20

  1. 36
  2. 20
  3. 10
  4. 12

Answer. 1. 36

Question 28. The profit of Rs 200/- is represented as

  1. +200
  2. -200
  3. Both A & B
  4. None

Answer. 1. +200

Question 29. 4° C below 0° is represented as

  1. +4° C
  2. -4° C
  3. Both A & B
  4. None

Answer. 2. -4° C

Question 30. Positive integers are

  1. Whole numbers
  2. Natural numbers
  3. Both A & B
  4. None

Answer. 2. Natural numbers

Question 31. Non negative integers are

  1. Natural numbers
  2. Either natural numbers or whole numbers
  3. Whole numbers
  4. Both B & C

Answer. 2. Either natural numbers or whole numbers

Question 32. Number of integers between -5 and 5

  1. 10
  2. 8
  3. 9
  4. 11

Answer. 3. 9

Question 33. Identify the false statement from the following

  1. Zero is the right side of -3
  2. Every positive integer is greater than zero
  3. There are infinite integers in the number system
  4. All integers are whole numbers

Answer. 4. All integers are whole numbers.

Question 34. The sum of two positive numbers is always

  1. Positive
  2. Negative
  3. Either positive nor negative
  4. Neither positive nor negative

Answer. 1. Positive

Question 35. The sum of two negative numbers is always

  1. Positive
  2. Negative
  3. Either positive or negative
  4. Neither positive nor negative

Answer. 1. Positive

Question 36. The sum of a positive and a negative number is

  1. Positive
  2. Negative
  3. Either positive or negative
  4. All above

Answer. 2. Negative

Question 37. a – (-a) =

  1. a
  2. 2a
  3. 0
  4. \(a^2\)

Answer. 2. 2a

Question 38. The biggest among -6, 0, 7, 1, -7, -9 is

  1. -9
  2. -7
  3. 7
  4. 0

Answer. 3. 7

Question 39. Add (-5) + (-5) + (-5) + 0

  1. 15
  2. -5
  3. 0
  4. -15

Answer. 4. -15

Question 40. Add the number 5 to additive inverse of -8 is

  1. 11
  2. -1
  3. 1
  4. 0

Answer. 2. -1

Question 41. Subtract 7 from additive inverse of -8 is

  1. -1
  2. 15
  3. 1
  4. -15

Answer. 3. 1

Question 42. Add additive inverse of 6 and additive inverse of -5

  1. 1
  2. -1
  3. 11
  4. -11

Answer. 2. -1

Question 43. Simplify 30 + (-30) + (-70) + 70 + 0

  1. 100
  2. 200
  3. 0
  4. 10

Answer. 3. 200

Question 44. Successor of -7 is ………

  1. -4
  2. -9
  3. -2
  4. -6

Answer. 4. -6

Question 45. Predecessor of 12 is ……

  1. 1
  2. 11
  3. 13
  4. 10

Answer. 2. 11

Question 46. Five numbers less than ‘0’ is ……

  1. -1
  2. -3
  3. -4
  4. -5

Answer. 4. -5

Question 47. The opposite of 2 km south is ………

  1. 2 km south
  2. 2 km east
  3. 2 km north
  4. none

Answer. 3. 2 km east

Question 48. 12 + ……… = 12

  1. 0
  2. 1
  3. 4
  4. 3

Answer. 1. 0

Question 49. (-4) + (+3) = …….

  1. 7
  2. 3
  3. -1
  4. 1

Answer. 3. -1

Question 50. -11 + (-12) = ……

  1. -23
  2. 23
  3. 4
  4. 1

Answer. 1. -23

Question 51. (-1) + (0) = ……

  1. 4
  2. 1
  3. 0
  4. -1

Answer. 4. -1

Question 52. (-9) + (+4) + (-6) + (+3) = ………

  1. 6
  2. -8
  3. -7
  4. 0

Answer. 2. -8

Question 53. -8 – (-10) = ……….

  1. 2
  2. -2
  3. 10
  4. 8

Answer. 1. 2

Haryana Board Class 6 Maths Solutions For  Chapter 6 Integers Fill in the blanks:

Question 54. 3 – 4 – 10 = ……….

Answer. -17

Question 55. a + (-a) ……….

Answer. 0

Question 56. 6 + (-10) + 5 + 17 = ………..

Answer. +6

Question 57. -16 – (-23)…. -6 + (-12) (Use >, < or =)

Answer. >

Question 58. …….. + (-51) = 100

Answer. 151

Question 59. Number of integers lying between-3 and 3 is ………

Answer. 5

Question 60. By arranging the following numbers in descending order, the middle integer is 7,8,0,-6,-5,-4,3 …………

Answer. 0

Question 61. Subtract-10 from – 10, we get ……….

Answer. 0

Question 62. Additive inverse of 17 is ……

Answer. +17

Question 63. ………. is neither positive nor negative integer.

Answer. 0

Question 64. (-4) + 6 …….(7-1) (use > or <)

Answer. <

Question 65. 35 – (20) …………

Answer. 15

Question 66. 12 + (-7) = …………….

Answer. 5

Question 67. (-6) + (-2) = ……….

Answer. -8

Question 68. 5 + (12) + 0 = ……….

Answer. 17

Question 69. ……………… is the successor of 7.

Answer. 8

Question 70. The predecessor of -2 is ………

Answer. -3

Question 71. 0. ………. 8 (use < or >)

Answer. <

Question 72. The collection of numbers 1, 2, 3, 4, …. are called …… numbers.

Answer. natural

Question 73. The sum of 137 and -137 is …………..

Answer. 0

Question 74. (-7) + (-3) + (+3) + (+7) = ……….

Answer. 0

Question 75. The loss of 200 in a business is represented as ……..

Answer. -200

Question 76. At a particular place morning temperature is 30°C. It has increased by 6°C by lunch time. At lunch time the temperature. ……………

Answer. 36°C

Question 77. The temperature recorded in Jammu at 6 o’ clock was 7°C. Every one hour the tempera- ture decreases by 2°C. At 10 o’ clock the temperature is …..

Answer. -1°C

Question 78. 5 + (-2) + (-1)= …….

Answer. 2

Question 79. The nearest positive integer to zero is ……..

Answer. +1

Question 80. Which is greater (-2) or (-1) ……….

Answer. -1

Question 81. Which is smaller 3 and -5 ………

Answer. -5

Question 82. The additive inverse of (-9) is ………

Answer. 9

Question 83. The depth below the sea level is denoted by ……. numbers.

Answer. negative

Question 84. Write the number between the least positive integer and the greatest negative integer is ……..

Answer. 0

Question 85. Use > or < in the box (-4) ……… (-10)

Answer. >

86. The subtraction of integers is the same as the addition of their ………

Answer. additive inverse

Haryana Board Class 6 Maths Solutions For  Chapter 6 Integers Match the following:

Question 87.

Class 6 Maths Chapter 6 Integers Match the Following

Answer. 1-E, 2-D, 3-A, 4-B, 5-C

Haryana Board Class 6 Maths Solutions For Chapter 11 Algebra

Haryana Board Class 6 Maths Solutions For  Chapter 11 Algebra

  • The branch of mathematics in which we study numbers is arithmetic.
  • The branch of mathematics in which we study shapes is geometry.
  • ‘n’ is an example of a variable. Its value is not fixed. It can take any value 1,2,3,4…
  • The main feature of algebra is the use of letters or alphabet to represent numbers.
  • We can make different patterns and shapes using match sticks. The relation between the patterns and number of sticks used is observed to frame rules.
  • Number of matchsticks required = 3x No. of triangles to be formed.
  • We may use any letter (small case alphabet) a, b, m, n, p, q, x, y, z etc…… to represent a variable.
  • An expression is made up of variables and constants using different operations.
    Eg: 2x, 3m – 1, 3s + 7, 8p, \(\frac{y}{3}\) etc..
  • Variables allow us to express many common rules of geometry and arithmetic in a more general way.
  • A mathematical statement consisting of an equal symbol between two algebraic expressions that have the same value is called ‘Algebraic Equation’.
    Eg: 3m = 12, 2x + 1 = 10, 3p + 5 = 2p + 6….. etc.,
  • Algebra is a powerful tool for solving ‘puzzles’, ‘riddles’ and problems in our daily life.

Haryana Board Class 6 Maths Solutions For  Chapter 11 Algebra Exercise 11.1

Question 1. Find the rule which gives the number of matchsticks required to make the following matchstick patterns. Use a variable to write the rule.

a) A pattern of letter T as T

b) A pattern of letter Z as Z

c) A pattern of letter U as U

d) A pattern of letter V as V

e) A pattern of letter E as E

f) A pattern of letter S as S

g) A pattern of letter A as A

Solution. a) Number of matchsticks required = 2n

b) Number of matchsticks required = 3n

c) Number of matchsticks required =3n

d) Number of matchsticks required = 2n

e) Number of matchsticks required = 5n

f) Number of matchsticks required = 5n

g) Number of matchsticks required = 6n

Haryana Board Class 6 Maths Algebra Solutions

Question 2. We already know the rules for the pat- tern of letters L, C and F. Some of the letters from Q. 1 (given above) give us the same rule as that given by L. Which are these? Why does this happen?

Solution. The letters T and V that has pattern 2n, since 2 match sticks are used in all these letters.

Question 3. Cadets are marching in a parade. There are 5 cadets in a row. What is the rule which gives the number of cadets, given the number of rows? (Use ‘n’ for the number of rows.)

Solution. Number of cadets = 5n

Question 4. If there are 50 mangoes in a box, how will you write the total number of mangoes in terms of the number of boxes? (Use ‘b’ for number of boxes.)

Solution. Total number of mangoes = 50 b.

Question 5. The teacher distributes 5 pencils per student. Can you tell how many pencils are needed, given the number of students? (Use’s ‘for the number of students.)

Solution. Number of pencils needed = 5 s.

Question 6. A bird flies 1 kilometer in one minute. Can you express the distance covered by the bird in terms of its flying time in minutes. (Use t’ for flying time in minutes)

Solution. Distance covered by the bird in t minutes = t kilometres.

Question 7. Radha is drawing a dot Rangoli (a beautiful pattern of lines joining dots with chalk powder. She has 9 dots in a row. How many dots will her Rangoli have for r rows? How many dots are there if there are 8 rows? If there are 10 rows?

Solution. Number of dots her Rangoli will have for ‘r’ rows = 9r.

Number of dots if there are 8 rows = 8 x 9 = 72.

Number of dots if there are 10 rows = 10 × 9 = 90.

Class 6 HBSE Maths Chapter 11 Algebra Important Questions

Question 8. Leela is Radha’s younger sister. Leela is 4 years younger than Radha. Can you write Leela’s age in terms of Radha’s age? Take Radha’s age to be x years.

Solution. Radha’s age = x years.

Leela’s age = (x – 4) years

Question 9. Mother has made laddus. She gives some laddus to guests and family members; still 5 laddus remain. If the number of laddus mother gave away is 1, how many laddus did she make?

Solution. Number of laddus she made l + 5.

Question 10. Oranges are to be transferred from larger boxes into smaller boxes. When a large box is emptied, the oranges from it fill two smaller boxes and still 10 oranges remain outside. If the number of oranges in a small box are taken to be x, what is the number of oranges in the larger box?

Solution. Number of oranges in the larger box = 2x + 10.

Question 11. a) Look at the following matchstick pattern of squares. The squares are not separate. Two neighbouring squares have a common matchstick. Observe the patterns and find the rule that gives the number of matchsticks in terms of the number of squares.

(Hint: If you remove the vertical stick at the and you will get a pattern of CS)

Class 6 Maths Chapter 11 Algebra Question 11.1

b) Gives a matchstick pattern of triangles. As in Exercise 11 (a) above, find the general rule that gives the number of matchsticks in terms of the number of triangles.

Class 6 Maths Chapter 11 Algebra Question 11.2

Solution. a) Number of matchsticks = 3n + 1, n= number of squares.

b) Number of matchsticks = 2n + 1, n = number of triangles.

Haryana Board Class 6 Maths Solutions For  Chapter 11 Algebra Very Short Answer Questions

Question 1. Make a rule between the number of blades required and the number of fans (say n) in a hall?

Solution. A rule between the number of blades required and number of fans (say n) in a hall is 3n.

Question 2. Harshini says that she has 5 biscuits more than Padma has. How can you express the relationship using the variable ‘y’?

Solution. Let the number of biscuits that Padma has = ‘y’.

Harshini says that she has 5 biscuits morethan Padma has.

∴ Number of biscuits that Harshini has = y + 5

Solving Simple Equations Class 6 Haryana Board

Question 3. Prabhakar has ‘p’ number of balls. Number of balls with Devi is 3 times the balls with Prabhakar. Write this as an expression.

Solution. Number of balls that Prabhakar has = p Devi has 3 times of the balls with Prabhakar

∴ The required expression is = 3 x p = 3p

Question 4. Find the general rule for the perimeter of a rectangle. Use variables ‘l’ and ‘b’ for length and breadth of the rectangle respectively.

Solution. Length of the rectangle = 1 units

Breadth = b units

Perimeter = 1 + b + 1 + b

= 21 + 2b

= 2(1 + b) units

Question 5. Find the general rule for the area of a square by using the variable ‘s’ for the side of a square.

Solution. Side of a square = s units

Area of a square = side x side = s x s

= s2 square units

Class 6 Maths Chapter 11 Algebra Very Short Answer Question 5.1

Class 6 Maths Chapter 11 Algebra Very Short Answer Question 5.2

Algebraic Expressions Class 6 HBSE Notes

Haryana Board Class 6 Maths Solutions For  Chapter 11 Algebra Short Answer Questions

Question 1. Arrange 2 matchsticks to form the shape

Continue the same shape for 2 times, 3 times and 4 times. Frame the rule for repeating the pattern.

Class 6 Maths Chapter 11 Algebra Short Answer Question 1

Solution.

Number of matchsticks required for shape – 1 is 2 = 2 × 1 = 2 × 1 times

Number of matchsticks required for shape – 2 is 4 = 2 x 2 = 2 × 2 times

Number of matchsticks required for shape – 3 is 6= 2 × 3 = 2 × 3 times

Number of matchsticks required for shape – 4 is 8.2 x 4 = 2 × 4 times

∴ The rule for number of matchsticks required for n times for the shape is = [2 x n] times.

Question 2. A line of shapes is constructed using matchsticks.

Class 6 Maths Chapter 11 Algebra Short Answer Question 2.1

Basic Algebra Problems For Class 6 With Solutions

1) Find the rule that shows how many sticks are needed to make a line of such shapes.

Solution.

Class 6 Maths Chapter 11 Algebra Short Answer Question 2.2

∴ Number of matchsticks = 2 x (Number of shapes) + 1

Let s = Number of shapes

m = Number of matchsticks

∴ Number of matchsticks = m = 2s + 1

2) How many matchsticks are needed to form shape 12 ?

Solution. Number of matchsticks needed to form shape 12

= 2 × 12 + 1

= 24 + 1 = 25

Question 3. Find the rule which gives the number of matchsticks required to make the following matchstick patterns.

1) A pattern of letter ‘T’

2) A pattern of letter ‘E’

3) A pattern of letter ‘Z’

Solution. 1)

Class 6 Maths Chapter 11 Algebra Short Answer Question 3.1

Number of matchsticks required to make a pattern of letter ‘T’ is 3

∴ The rule is 3 m.

2)

Class 6 Maths Chapter 11 Algebra Short Answer Question 3.2

Number of matchsticks required to make a

pattern of letter ‘E’ is 4

∴ The rule is 4m

Word Problems On Algebra For Class 6 Haryana Board

3)

Class 6 Maths Chapter 11 Algebra Short Answer Question 3.3

 

Number of matchsticks required to make a

pattern of letter ‘Z’ is 3

∴ The rule is 3m

Question 4. Find a rule for the following pattern between number of shapes formed and number of matchsticks required.

Class 6 Maths Chapter 11 Algebra Short Answer Question 4

Solution. a) Number of match sticks used for this pattern is 2, 4, 6, ……… Rule is – 2n

b) Number of match sticks used for this pattern is 3, 6, 9, …… Rule is – 3n

Question 5. 1) How many match sticks are required to form a triangle?

Solution. To make a triangle 3 match sticks are required.

2) Complete the following table: Write the rule.

Class 6 Maths Chapter 11 Algebra Short Answer Question 5

Solution. The rule is “number of matchsticks required = 3 times the number of triangles to be formed.”

Question 6. 1) How many matchsticks are required to form a square?

Solution. To make a square 4 matchsticks are required..

2) Complete the following table: Write the rule.

Class 6 Maths Chapter 11 Algebra Short Answer Question 6

Solution. The rule is “Number of matchsticks required = 4 times number of squares to be formed”.

Haryana Board Class 6 Maths Solutions For  Chapter 11 Algebra Long Answer Questions

Question 1. Rita took matchsticks to form the shape

She repeated the pattern and gave a rule

Class 6 Maths Chapter 11 Algebra Long Answer Question 1

Number of matchsticks needed = 6.y, where y is the number of shapes to be formed.

Is it correct? Explain.

What is the number of sticks needed to form 5 such shapes?

Solution.

Class 6 Maths Chapter 11 Algebra Long Answer Question 1.1

Number of match sticks required for shape – 1 is 6 = 6 × 1 times

Number of match sticks required for shape – 2 is 12 = 6 x 2 times

Number of match sticks required for shape – 3 is 18 = 6 x 3 times

Number of match sticks required for shape – 4 is 24 = 6 × 4 times

Number of match sticks required for shape 5 is 30 = 6 x 5 times

∴ Number of match sticks needed for the shape y is = 6 x y

[Where y is the number of shapes to be formed]

Yes, Rita is correct.

Number of match sticks needed to form 5 shapes = 6 x 5 = 30

Question 2. Find the nth term in the following sequences

(1),3,6,9,12,…

(2) 2,5,8,11,…

(3) 1, 4, 9, 16,…

Solution. 1) 3,6,9,12.

3 = 3 × 1

6 = 3 × 2

9 = 3 × 3

12 = 3 × 4

nth term of the sequence = 3 x n

[∴ n is a natural number]

2) 2,5,8,11

2 = 3 x 1 – 1

5 = 3 × 2 – 1

8 = 3 × 3 – 1

11 = 3 x 4 – 1

nth term of the sequence = 3 x n – 1

[∴ n is a natural number]

3) 1, 4, 9, 16……………….

1 = \(\mathrm{1}^2\)

4 = \(\mathrm{2}^2\)

9 = \(\mathrm{3}^2\)

16 = \(\mathrm{4}^2\)

nth term of the sequence = \(\mathrm{n}^2\)

[∴ n is a natural number]

Haryana Board Class 6 Maths Solutions For  Chapter 11 Objective Type Questions

Choose the correct answer :

Question 1. The symbol used in equation is

  1. >
  2. =
  3. <

Answer. 2. =

Question 2. How many sticks are needed to prepare the letter Z?

  1. 6
  2. 7
  3. 2
  4. 3

Answer. 4. 3

Question 3. 2, 4, 6, ……… nth term is …………

  1. \(\frac{n}{2}\)
  2. 2n
  3. 4n
  4. \(\mathrm{n}^2\)

Answer. 2. 2n

Class 6 Maths Variables And Constants Explained

Question 4. Statement – A: An expression is made up of variables and constants using different operations.

Statement – B: A mathematical statement consisting of an equal symbol between two algebraic expressions is called ‘Algebraic equation’

  1. Both A & B are true
  2. A is true, B is false
  3. A is false, B is true
  4. Both A & B are false

Answer. 1. Both A & B are true

Question 5. Write the statement for the expression 3m + 11

  1. 11 more than three times of m
  2. 11 is added to thrice the m
  3. 11 times more than thrice them
  4. Both A & B

Answer. 4. Both A & B

Question 6. Statement – A: Solution of an equation is the value of the variable for which LHS and RHS are equal.

Statement – B: The solution is also called root of the equation.

  1. Both A & B are true
  2. A is true, B is false
  3. A is false, B is true
  4. Both A & B are false

Answer. 1. Both A & B are true

Question 7. 3, 6, 9, 12 ………. nth term is

  1. 2n
  2. n + 3
  3. 3n
  4. \(\frac{n}{3}\)

Answer. 3. 3n

Question 8. The nth term of 2, 5, 8, 11 …….

  1. 3n
  2. 3n – 1
  3. 3n + 1
  4. 2n + 1

Answer. 2. 3n – 1

Question 9. The nth term of 1, 4, 9, 16 ……..

  1. \(\mathrm{n}^2\)
  2. 2n
  3. 2n – 1
  4. n + 2

Answer. 1. \(\mathrm{n}^2\)

Haryana Board Class 6 Maths Solutions For  Chapter 11 Algebra Fill in the blanks:

Question 10. 1, 4, 9, 16. nth term is ……..

Answer. \(\mathrm{n}^2\)

Question 11. The cost of one book is 101 then the cost of p such books is ………

Answer. ₹ 101p

Haryana Board Class 6 Maths Solutions For Chapter 4 Basic Geometrical Ideas

Haryana Board Class 6 Maths Solutions For  Chapter 4 Basic Geometrical Ideas

  • The term ‘GEOMETRY’ is derived from the Greek word ‘GEOMETRON’.
  • ‘Geo’ means earth and ‘metron’ means ‘measurement’.
  • ‘Euclid’ is known as ‘Father of Geometry’. He introduced geometry in a logical order in the book ‘The Elements’..
  • The ancient Indian Mathematician Aryabhata and Brahmagupta contributed their works in geometry.
  • A ‘point’ determines a location. It is usually denoted by capital letters in English.
  • A ‘line segment’ is formed by joining two points. It has a fixed length. A line segment has negligible thickness. It has two end points.

Class 6 Maths Chapter 4 Basic Geometrical Ideas Line Segment

  • A ‘line’ is obtained when a line segment extends on both sides indefinitely. It is denoted by small letters such as l, m, n etc.

Class 6 Maths Chapter 4 Basic Geometrical Ideas Line Segment

  • It has no end points.
  • A’ray’ is a portion of a line starting at a point and goes in one direction endlessly.

Class 6 Maths Chapter 4 Basic Geometrical Ideas Ray

  • Eg: Ray \(\overrightarrow{\mathrm{AB}}\) Ray \(\overrightarrow{\mathrm{CA}}\)
  • By using divider and scale, we can measure the length of line segment.
  • A figure drawn without lifting a pencil is called a ‘curve’. In this sense, a line is also a curve.
  • Parallel Lines: The lines in a plane that never meet even if they are extended any further are called Parallel lines. Eg: The lines in your ruled book are parallel lines.

Class 6 Maths Chapter 4 Basic Geometrical Ideas Parallel Lines

  • If line l is parallel to line ‘m’, then it is denoted by l || m. It can be read as “l is Parallel to m”.
  • Intersecting Lines: The lines that cross one another are called intersecting lines and the point at which they intersect is called intersecting point.

Class 6 Maths Chapter 4 Basic Geometrical Ideas Intersecting Lines

Haryana Board Class 6 Maths Basic Geometrical Ideas solutions

  • Here ‘O’ is the intersecting point.
  • Concurrent Lines: Three or more lines passing through the same point are called concurrent lines. The point is called point of concurrency.

Class 6 Maths Chapter 4 Basic Geometrical Ideas Concurrent Lines

  • Perpendicular Lines: The angle between any two lines is 90° are called ‘perpendicular lines’: It is denoted by ⊥ (perpendicular).
  • If line l is perpendicular to the line m then it is denoted by l ⊥ m and read as ‘I is perpendicular to m’.

Class 6 Maths Chapter 4 Basic Geometrical Ideas Perpendicular Lines

  • Angle: An angle in a figure formed by two rays with a common end point. The common end point is called ‘vertex’. The rays are the ‘sides or arms’ of the angle. It is denoted by ‘∠AOB’ read as angle АОВ.

Class 6 Maths Chapter 4 Basic Geometrical Ideas Angle

Types of angles.

Class 6 Maths Chapter 4 Basic Geometrical Ideas Types of angles

  • We use a tool called “Protractor” to measure the given angle.
  • Two rays with a common initial make two regions.
  • The interior of an angle is the group of all the points between the rays of the angle.
  • The exterior of an angle is the group of all points outside the rays of the angle.

Class 6 Maths Chapter 4 Basic Geometrical Ideas Exterior angle

  • The simple closed figure formed by three line segments is called a triangle.
  • The line segments are called sides.
  • A triangle has three vertices, three sides and three angles.
  • In triangle ABC

Class 6 Maths Chapter 4 Basic Geometrical Ideas Triangle

A, B, C are three vertices \(\overline{A B}, \overline{B C}, \overline{C A}\) are three sides ∠BAC, ∠ABC, ∠ACB are three angles. The triangle ABC is denoted by ΔABC.

  • A triangle divides a plane into three parts
    1. Interior of the triangle
    2. Boundary of the triangle
    3. Exterior of the triangle

Class 6 Maths Chapter 4 Basic Geometrical Ideas Triangle in a plane

Eg. PQR is a triangle.

The points 0, B, S are interior of the tri- angle.

The points A, C are on the Boundary of the trianlge.

The points T, U, V are exterior of the tri- angle.

  • A ‘simple curve’ is one that does not cross itself.
  • Curves are of 2 types – open curve; closed curve

Class 6 Maths Chapter 4 Basic Geometrical Ideas Curve

  • A figure is a ‘polygon’ if it is a simple closed figure made up definite number of line segments
  • A closed figure separates the plane into three parts.
    1. Interior (inside) of the figure.
    2. Boundary of the figure.
    3. Exterior (outside) of the figure.
  • The interior of the figure together with its boundary is its ‘region’.

Class 6 Maths Chapter 4 Basic Geometrical Ideas Interior

Eg. : ABCD is a closed figure formed by 4 line segments AB, BC, CD and DA.

Points P, Q are Interior of the figure, points R, S are on the Boundary line, points T, U are on the exterior of the figure.

  • The interior of the figure together with its boundary is its region

Question 1. A Star in the sky also gives us an idea of a point. Identify at least five such situations in your daily life.

Solution. Five situations giving an idea of a point are:

  1. Sharp tip of pencil.
  2. Tip of a pen
  3. Tip of the compass
  4. Pointed end of the needle
  5. Dot on the paper

Question 2. Name the line segments in the figure 4.2. Is A, the end point of each line segment?

Solution.

Class 6 Maths Chapter 4 Basic Geometrical Ideas Question 1

The line segments in the given figure are

\(\overline{\mathrm{AB}}\) (or \(\overline{\mathrm{BA}}\)) and \(\overline{\mathrm{AC}}\) (or \(\overline{\mathrm{CA}}\))

Yes! A is the end point of each line segment.

Points, lines, and angles Class 6 HBSE Maths

Question 3. 1. Name the rays given in this picture.

2. Is T a starting point of each of these rays?

Solution. 1) \(\overrightarrow{\mathrm{TA}}, \overrightarrow{\mathrm{TN}}, \overrightarrow{\mathrm{~TB}}, \overrightarrow{\mathrm{NB}}\)

2) No! T is not a starting point of each of these rays.

Class 6 Maths Chapter 4 Basic Geometrical Ideas Question 2

Haryana Board Class 6 Maths Solutions For  Chapter 4 Basic Geometrical Ideas Exercise 4.1

Question 1. Use the figure to name :

a) Five points

b) A line

c) Four rays

d) Five line segments

Class 6 Maths Chapter 4 Basic Geometrical Ideas Exercise 4.1 Question 1

Solution. a) O, B, C, D, E

b) DB

c) \(\overrightarrow{O B}, \overrightarrow{O C}, \overrightarrow{O E}, \overrightarrow{O D}\)

d) \(\overline{O B}, \overline{O C}, \overline{O E}, \overline{O D}, \overline{E D}\)

Question 2. Name the line given in all possible (twelve) ways, choosing only two letters at a time from the four given.

Solution. Let the four points be A, B, C and D.

Class 6 Maths Chapter 4 Basic Geometrical Ideas Exercise 4.1 Question 2

AB, AC, AD, BA, BC, BD, CA, CB, CD, DA, DB, DC.

Question 3. Use the figure to name:

a) Line containing point E.

b) Line passing through A.

c) Line on which O lies.

d) Two pairs of intersecting lines.

Class 6 Maths Chapter 4 Basic Geometrical Ideas Exercise 4.1 Question 3

Solution.

a) EF

b) AE

c) OC

d) AE and CO; AE and EF.

Question 4. How many lines can pass through

a) one given point?

b) two given points?

Solution. a) Infinitely many lines can pass through one given point.

b) One and only one line can pass through two given points.

Question 5. Draw a rough figure and label suitably in each of the following cases:

a) Point Plies on \(\overleftrightarrow{\mathrm{AB}}\)

b) \(\overleftrightarrow{\mathrm{XY}}\) and \(\overleftrightarrow{\mathrm{PQ}}\) intersect at M.

c) Line I contains É and F but not D.

d) \(\overleftrightarrow{\mathrm{OP}}\) and \(\overleftrightarrow{\mathrm{OQ}}\) meet at O.

Solution.

Class 6 Maths Chapter 4 Basic Geometrical Ideas Exercise 4.1 Question 5

Question 6. Consider the following figure of line \(\overline{\mathbf{M N}}\). Say whether following statements are true or false in context of the given figure.

Class 6 Maths Chapter 4 Basic Geometrical Ideas Exercise 4.1 Question 6

a. Q, M, O, N, P are points on the line \(\overline{\mathbf{M N}}\)

Solution. True.

b. M, O, N are points on a line segment \(\overline{\mathbf{M N}}\)

Solution. True

c. M and N are end points of line segment \(\overline{\mathbf{M N}}\)

Solution. True

d. O and N are end points of line segment \(\overrightarrow{\mathrm{OP}}\)

Solution. False.

e. M is one of the end points of the line segment \(\overline{\mathbf{Q O}}\)

Solution. False.

f. M is point on ray \(\overrightarrow{\mathrm{OP}}\).

Solution. False.

g. Ray \(\overrightarrow{\mathrm{OP}}\) is different from ray \(\overrightarrow{\mathrm{QP}}\).

Solution. True.

h. Ray \(\overrightarrow{\mathrm{OP}}\) is same as ray \(\overrightarrow{\mathrm{OM}}\).

Solution. False.

i. Ray \(\overrightarrow{\mathrm{OM}}\) is not opposite to ray \(\overrightarrow{\mathrm{OP}}\).

Solution. False.

j. ‘O’ is not an initial point of ray \(\overrightarrow{\mathrm{OP}}\).

Solution. False.

k. N is the initial point of \(\overrightarrow{\mathrm{NP}} \text { and } \overrightarrow{\mathrm{NM}}\).

Solution. True

Haryana Board Class 6 Maths Solutions For  Chapter 4 Basic Geometrical Ideas Exercise – 4.2

Question 1. Classify the following curves as

(1) Open or

(2) Closed.

Class 6 Maths Chapter 4 Basic Geometrical Ideas Exercise 4.2 Question 1

Solution. (1) a, c – open curves

(2) b, d, e – closed curves.

Question 2. Draw rough diagrams to illustrate the following:

(a) Open curve

(b) Closed curve

Solution.

Class 6 Maths Chapter 4 Basic Geometrical Ideas Exercise 4.2 Question 2

Types of angles in geometry Class 6 Haryana Board

Question 3. Draw any polygon and shade its interior.

Solution.

Class 6 Maths Chapter 4 Basic Geometrical Ideas Exercise 4.2 Question 3

Question 4. Consider the given figure and answer the questions:

a) Is it a curve?

b) Is it closed?

Class 6 Maths Chapter 4 Basic Geometrical Ideas Exercise 4.2 Question 4

Solution. a) Yes! it is a curve.

b) Yes! it is closed.

Question 5. Illustrate, if possible, each one of the following with a rough diagram:

Solution. a) A closed curve that is not a polygon.

Class 6 Maths Chapter 4 Basic Geometrical Ideas Exercise 4.2 Question 5.1

b) An open curve made up entirely of line segments.

Class 6 Maths Chapter 4 Basic Geometrical Ideas Exercise 4.2 Question 5.2

c) A polygon with two sides.

Not Possible.

A: A polygon is a closed plane figure with three or more sides that are all straight. Therefore, A polygon with two side cannot be drawn.

Haryana Board Class 6 Maths Solutions For  Chapter 4 Basic Geometrical Ideas Exercise – 4.3

Question 1. Name the angles in the given figure.

Solution. ∠ABC, ∠BCD, ∠CDA, ∠DAB

Class 6 Maths Chapter 4 Basic Geometrical Ideas Exercise 4.3 Question 1

Question 2. In the given diagram, name the point(s)

a) In the interior of DOE

b) In the exterior of ZEOF

c) On ∠EOF

Answer. a) A

b) C, A, D

c) O, B, E, F

Class 6 Maths Chapter 4 Basic Geometrical Ideas Exercise 4.3 Question 2

Polygons and their properties Class 6 HBSE Maths

Question 3. Draw rough diagrams of two angles such that they have

a. One point in common

Solution.

Class 6 Maths Chapter 4 Basic Geometrical Ideas Exercise 4.3 Question 3

∠AOB and ∠BOC have one point ‘O’ in common.

b. Two points in common.

Solution. ∠AOB and ∠OBC have two points O and B in common.

Class 6 Maths Chapter 4 Basic Geometrical Ideas Exercise 4.3 Question 3.1

c. Three points in common.

Solution. Not possible

d. Four points in common.

Solution. Not possible

e. One ray in common.

Solution. ∠AOB and ∠BOC have one ray OB in common.

Class 6 Maths Chapter 4 Basic Geometrical Ideas Exercise 4.3 Question 3.2

Chapter 4 Basic Geometrical Ideas Very Short Answer Questions

Question 1. Draw a line, ray and a line segment.

Solution.

Class 6 Maths Chapter 4 Basic Geometrical Ideas Very Short Answer Question 1

Question 2. Mark two points X and Y. Also mark three more points between them and another three points that are not in between those two points.

Class 6 Maths Chapter 4 Basic Geometrical Ideas Very Short Answer Question 2

Solution. X, Y are any two points.

P, Q, R are three points between X and Y.

A, B, C are three points not between X and Y.

Question 3. Define a triangle. Write the parts in it.

Solution. A simple colsed figure formed by three line segments is called a triangle. It has 3 sides, 3 vertices and 3 angles.

Question 4. Join the points given below. Name the line segments so formed in the figure.

Class 6 Maths Chapter 4 Basic Geometrical Ideas Very Short Answer Question 4.1

Class 6 Maths Chapter 4 Basic Geometrical Ideas Very Short Answer Question 4.2

Solution. 1)

Class 6 Maths Chapter 4 Basic Geometrical Ideas Very Short Answer Question 4.3

Line segments so formed are: \(\overline{\mathrm{AB}}, \overline{\mathrm{BC}}, \overline{\mathrm{CA}}\)

2)

Class 6 Maths Chapter 4 Basic Geometrical Ideas Very Short Answer Question 4.4

Line segments so formed are: \(\overline{\mathrm{PQ}}, \overline{\mathrm{QR}}, \overline{\mathrm{RS}}, \overline{\mathrm{ST}}, \overline{\mathrm{TP}}\)

Parts of a circle Class 6 HBSE Maths

Question 5. Which of the following has a definite length?

1) Line

2) Point

3) Line segment

4) Ray

Solution. Line segment. Because it has two end points.We can measure it.

Question 6. Tell which letter is an example of simple curve.

Solution. The letter O is an example of simple curve.

Question 7. What is the least no. of sticks needed to form a closed figure?

Class 6 Maths Chapter 4 Basic Geometrical Ideas Very Short Answer Question 7

Solution. Three sticks are needed to form a closed figure.

Haryana Board Class 6 Maths Solutions For  Chapter 4 Basic Geometrical Ideas Short Answer Questions

Question 1. From the adjacent figure, write down

1) The sides of the triangle

2) The angles of the triangle

3) The vertices of the triangle.

Class 6 Maths Chapter 4 Basic Geometrical Ideas Short Answer Question 1

Solution. 1) The sides of the triangle – \(\overline{\mathrm{MN}}, \overline{\mathrm{NH}}, \overline{\mathrm{HM}}\)

2) The angles of the triangle – ∠HMN, ∠MNH, ∠NHM

3) The vertices of the triangle – H, M, N

Question 2. Write the set of parallel lines and perpendicular lines in the given figure by using symbols.

Class 6 Maths Chapter 4 Basic Geometrical Ideas Short Answer Question 2

Solution. Parallel lines: \(\overline{\mathrm{AB}} \text { and } \overline{\mathrm{DC}} ; \overline{\mathrm{AB}} \| \overline{\mathrm{DC}} ; \overline{\mathrm{AQ}} \text { and } \overline{\mathrm{PC}} ; \overline{\mathrm{AQ}} \| \overline{\mathrm{PC}}\)

Perpendicular lines: \(\overline{\mathrm{AD}} \perp \mathrm{AB} \text { and } \overline{\mathrm{AD}} \perp \mathrm{DC}\)

Question 3. Draw

1) Intersecting lines

2) Parallel lines

3) Perpendicular lines.

Solution.

1) Intersecting lines

Class 6 Maths Chapter 4 Basic Geometrical Ideas Short Answer Question 3.1

2) Parallel lines

Class 6 Maths Chapter 4 Basic Geometrical Ideas Short Answer Question 3.2

3) Perpendicular lines

Class 6 Maths Chapter 4 Basic Geometrical Ideas Short Answer Question 3.3

Question 4. Name the following from the figure.

Class 6 Maths Chapter 4 Basic Geometrical Ideas Short Answer Question 4

1) Any five points : A, B, C, D, O

2) Any five line segments : \(\overline{\mathrm{AB}}, \overline{\mathrm{BC}}, \overline{\mathrm{CD}}, \overline{\mathrm{DA}}\)

3) Any three rays : \(\overrightarrow{\mathrm{OA}}, \overrightarrow{\mathrm{OB}}, \overrightarrow{\mathrm{OC}}\)

4) Any two lines : \(\overrightarrow{\mathrm{AC}}, \overrightarrow{\mathrm{BD}}\)

Question 5. Move your pencil along the following english letters and state which are open and which are closed.

Class 6 Maths Chapter 4 Basic Geometrical Ideas Short Answer Question 5

Solution. (i), (iii), (iv) are open, (ii) is closed.

Question 6. Tick the figures which are simple curves.

Class 6 Maths Chapter 4 Basic Geometrical Ideas Short Answer Question 6

Solution. Simple curves: (i), (ii), (iv)

Question 7. State which curves are open and which are closed.

Class 6 Maths Chapter 4 Basic Geometrical Ideas Short Answer Question 7

Solution. Open curves: (i), (v)

Closed curves: (ii), (iii), (iv)

Question 8. Mark the points in the figure which satisfy all the three conditions.

1) A, B in the interior of ∠DOF

2) A, C in the exterior of ∠EOF

3) B is on ∠DOE

Solution.

Class 6 Maths Chapter 4 Basic Geometrical Ideas Short Answer Question 8

Question 9. Identify simple closed and open figures from the figures given here under.

Class 6 Maths Chapter 4 Basic Geometrical Ideas Short Answer Question 9

Solution. Simple closed figures – a, c, d;

Simple open figure – b

Question 10. Identify simple curved figures.

Class 6 Maths Chapter 4 Basic Geometrical Ideas Short Answer Question 10

Solution. (ii), (iii), (vi) are simple curved figures.

Haryana Board Class 6 Maths Solutions For  Chapter 4 Basic Geometrical Ideas Long Answer Questions

Question 1. Write the names of the angles, their vertices and arms.

Class 6 Maths Chapter 4 Basic Geometrical Ideas Long Answer Question 1.1

Solution.

Class 6 Maths Chapter 4 Basic Geometrical Ideas Long Answer Question 1.2

Question 2. From the adjacent figure write the points that belongs to

1) the angle

2) the interior of the angle

3) the exterior of the angle

Solution. The points that belongs to

1) The angle – B, R, E, O, C, A

2) The interior of the angle – D, F, Q

3) The exterior of the angle – M,L, N

Class 6 Maths Chapter 4 Basic Geometrical Ideas Long Answer Question 2

Question 3. From the adjacent figure write

1) Interior points of the triangle.

2) Exterior points of the triangle.

3) Points on the boundary of the triangle.

4) Vertices of the triangle.

Class 6 Maths Chapter 4 Basic Geometrical Ideas Long Answer Question 3

Solution. 1) Interior points of the triangle – B, M

2) Exterior points of the triangle – A, C, N

3) Points on the boundary of the triangle – P, Q, R, D, L

4) Vertices of the triangle-P, Q, R

Question 4. Write ‘True’ or ‘False”.

1) A line has no end points.

True

2) Ray is a part of a line.

True

3) A line segment has no definite length.

False

4) A line segment has only one end point.

False

5) We can draw many lines through a point.

True

Question 5. Identify which are simple curves and which are not?

Class 6 Maths Chapter 4 Basic Geometrical Ideas Long Answer Question 5

Solution. If a curve does not cross itself then it is a simple curve.

(i), (ii), (iv) are simple curves.

(iii) The curve crosses it self so it is not a simple curve.

Question 6. Name the points that lie in the interior, on boundary and in the exterior of the figure.

Class 6 Maths Chapter 4 Basic Geometrical Ideas Long Answer Question 6

Solution. The points that lie

1) Interior – A, B, E, G, I;

2) Boundary – K, F, C;

3) Exterior – J, D

Question 7. Draw three simple closed figures:

1) By straight lines only.

Solution.

Class 6 Maths Chapter 4 Basic Geometrical Ideas Long Answer Question 7.1

2) By straight lines and curved lines both.

Solution.

Class 6 Maths Chapter 4 Basic Geometrical Ideas Long Answer Question 7.2

Question 8. Name the angles, vertex and arms of the angles from the figure.

Class 6 Maths Chapter 4 Basic Geometrical Ideas Long Answer Question 8

Class 6 Maths Chapter 4 Basic Geometrical Ideas Long Answer Question 8.1

Difference between line and line segment Class 6

Question 9. In the figure given below, Identify which points lie

1) in the interior,

2) in the exterior,

3) on the boundary.

Class 6 Maths Chapter 4 Basic Geometrical Ideas Long Answer Question 9

Solution. 1) Interior points – A, T, Q, I

2) Exterior points – B, D, L, K

3) On the boundary-C, E

Question 10. Observe the adjacent figure. Write answers for the following.

Class 6 Maths Chapter 4 Basic Geometrical Ideas Long Answer Question 10

Solution. 1) Name : PQRS Quadrilateral

2) Vertices : P, Q, R, S

3) Sides : \(\overline{\mathrm{PQ}}, \overline{\mathrm{QR}}, \overline{\mathrm{RS}}, \overline{\mathrm{SP}}\)

4) Angles : ∠SPQ, ∠PQR, ∠QRS, ∠RSP

5) Opposite sides : \(\overline{\mathrm{PQ}}, \overline{\mathrm{RS}} ; \overline{\mathrm{QR}}, \overline{\mathrm{PS}}\)

6)Opposite angles : ∠SPQ, ∠QRS; ∠PQR, ∠RSP

7) Adjacent angles : ∠SPQ, ∠PQR, ∠QRS, ∠RSP, ∠SPQ

Haryana Board Class 6 Maths Solutions For  Chapter 4 Basic Geometrical Ideas Objective Type Questions

Choose the correct answer:

Question 1. How many end points does a line have?

  1. Infinite
  2. 2
  3. 0
  4. 1

Answer. 4. 1

Question 2. How many end points does a line have?

  1. No end points
  2. 2
  3. Infinite
  4. 1

Answer. 1. No end points

Question 3. The closed figure formed by more line segments is called

  1. square
  2. polygon
  3. triangle
  4. circle

Answer. 2. polygon

Question 4. We denote line AB as

  1. \(\overrightarrow{\mathrm{AB}}\)
  2. \(\overline{\mathrm{AB}}\)
  3. \(\overleftrightarrow{\mathrm{AB}}\)
  4. \(\widehat{\mathrm{AB}}\)

Answer. 3. \(\overleftrightarrow{\mathrm{AB}}\)

Question 5. Number of points on a line

  1. 3
  2. 2
  3. 4
  4. Infinite

Answer. 4. Infinite

Question 6. There are …… vertices in a triangle.

  1. 4
  2. 3
  3. 2
  4. 8

Answer. 2. 3

Question 7. In the figure vertex of the angle is …..

Class 6 Maths Chapter 4 Basic Geometrical Ideas Choose the correct answer 10

  1. P
  2. O
  3. R
  4. \(\overrightarrow{\mathrm{OP}}\)

Answer. 2. O

Observe the following figure and answer the following questions from 8 to 10.

Class 6 Maths Chapter 4 Basic Geometrical Ideas Choose the correct answer 11

Word problems on basic geometrical ideas for Class 6 HBSE

Question 8. Interior point

  1. X
  2. L
  3. P
  4. Z

Answer. 3. P

Question 9. Exterior points

  1. X,L
  2. Z,L
  3. O,P
  4. X,Z

Answer. 2. Z,L

Question 10. Points on the ray \(\overrightarrow{\mathrm{OB}}\)

  1. X
  2. Z
  3. P
  4. O

Answer. 1. X

Question 11. Number of vertices of a square is ……..

  1. 3
  2. 4
  3. 8
  4. 13

Answer. 2. 4

Question 12. Triangle PQR can be denoted by …….

  1. PΔQR
  2. ΔPQR
  3. QRPΔ
  4. PQR

Answer. 2. ΔPQR

Question 13. How many lines can be drawn through a point in a plane?

  1. 16
  2. 12
  3. 3
  4. Infinite

Answer. 4. Infinite

Question 14. The symbol of perpendicular is

  1. |
  2. X
  3. T

Answer. 3. ⊥

Question 15. The symbol of parallel is

  1. X
  2. T

Answer. 2. ∥

Question 16. Which of the following are intersecting lines?

  1. X
  2. *

Answer. 2. X

Question 17. Two adjacent edges of blackboard are example for …… lines.

  1. parallel
  2. concurrent
  3. perpendicular
  4. all above

Answer. 3. perpendicular

Question 18. Two opposite edges of a paper is example for …….. lines.

  1. intersecting
  2. perpendicular
  3. parallel
  4. concurrent

Answer. 3. parallel

Question 19. To measure the angle we use

  1. divider
  2. ruler
  3. setsquare
  4. protractor

Answer. 4. protractor

Question 20. How many right angles can make a complete angle?

  1. 1
  2. 2
  3. 3
  4. 4

Answer. 4. 4

Question 21. End points of a line segment are

  1. 0
  2. 1
  3. 2
  4. Infinite

Answer. 3. 2

Question 22. How many lines can be drawn through two points?

  1. 0
  2. 1
  3. 2
  4. Infinite

Answer. 2. 1

Question 23. How many line segments lie in the given line?

Class 6 Maths Chapter 4 Basic Geometrical Ideas Choose the correct answer 26

  1. 4
  2. 5
  3. 6
  4. 7

Answer. 3. 6

Question 24. Father of Geometry

  1. Phythagoras
  2. Ramanujan
  3. Euclid
  4. Thales

Answer. 3. Euclid

Question 25. The word Geometry is derived from ……. language.

  1. Greek
  2. Latin
  3. Italian
  4. Indian

Answer. 1. Greek

Question 26. If two lines cross one another that lines are called

  1. Parallel lines
  2. Intersecting lines
  3. Concurrent lines
  4. Perpendicular lines

Answer. 2. Intersecting lines

Question 27. If three or more lines passing through the same point are …….. lines.

  1. parallel
  2. intersecting
  3. concurrent
  4. perpendicular

Answer. 3. concurrent

Question 28. The angle between two lines are 90° then they are …… lines.

  1. parallel
  2. intersecting
  3. concurrent
  4. perpendicular

Answer. 4. perpendicular

Question 29. If two lines does not meet at any where are ….. lines.

  1. parallel
  2. intersecting
  3. concurrent
  4. perpendicular

Answer. 1. intersecting

Question 30. The lines in your ruled book are example of …… lines.

  1. intersecting
  2. parallel
  3. perpendicular
  4. concurrent

Answer. 2. parallel

Question 31. Straight angle =

  1. 90°
  2. 180°
  3. 360°

Answer. 3. 180°

Question 32. Complete angle =

  1. 90°
  2. 180°
  3. 360°

Answer. 4. 360°

Question 33. Reflexive angle =

  1. 190°
  2. 210°
  3. 315°
  4. all the above

Answer. 4. 315°

Question 34. Flag pole on the earth is an example for …… lines.

  1. perpendicular
  2. parallel
  3. concurrent
  4. intersecting

Answer. 1. perpendicular

Question 35. How many common points that parallel lines have

  1. 0
  2. 1
  3. 2
  4. Infinite

Answer. 1. 0

Observe the following figure and answer the questions (36 – 37)

Class 6 Maths Chapter 4 Basic Geometrical Ideas Choose the correct answer 40

Question 36. Identify the perpendicular lines.

  1. l, m, n
  2. p, m
  3. p, n
  4. l, m

Answer. 2. p, m

Question 37. Identify the intersecting lines

  1. p, l
  2. l, m, n
  3. p, n and p, m
  4. None

Answer. 3. p, l

Question 38. The closed figure formed by more line segments is called …..

  1. square
  2. polygon
  3. triangle
  4. circle

Answer. 2. polygon

Question 39. We denote line AB as

  1. \(\overrightarrow{\mathrm{AB}}\)
  2. \(\overline{\mathrm{AB}}\)
  3. \(\overleftrightarrow{\mathrm{AB}}\)
  4. \(\overparen{\mathrm{AB}}\)

Answer. 3. \(\overleftrightarrow{\mathrm{AB}}\)

Question 40. The word ‘Geo means ……

  1. Geometry
  2. Geogre
  3. Earth
  4. Metron

Answer. 3. Earth

Question 41. A point determines a ……….

  1. location
  2. line
  3. object
  4. none

Answer. 1. location

Question 42. Two points determine a …..

  1. circle
  2. line
  3. triangle
  4. sector

Answer. 2. line

Question 43. If two lines l1 and l2 are parallel, then we represent this with …..

  1. l1,l2
  2. l1 ∥ l2
  3. l1/l2
  4. l2 ∥ l1

Answer. 2. l1 ∥ l2

Question 44. A …… is a portion of line.

  1. circle
  2. point
  3. triangle
  4. ray

Answer. 4. ray

Question 45. The line segments forming a polygon are called its …….

  1. vertex
  2. circle
  3. interior
  4. side

Answer. 1. vertex

Question 46. The two rays forming the angle are called ………. of the angle.

  1. segment
  2. common
  3. vertex
  4. arms

Answer. 4. arms

Question 47. The three sided polygon is called …..

  1. quadrilateral
  2. line segment
  3. side
  4. triangle

Answer. 4. triangle

Haryana Board Class 6 Maths Solutions For  Chapter 4 Basic Geometrical Angles Fill in the blanks:

Question 48. 72° is a measure of ……….. angle

Answer. acute

Question 49. A straight angle measures……….

Answer. 180°

Question 50. Your 6th class mathematics text book corner will be taken as a measure of ………

Answer. 90°

Question 51. Half of complete angle. ……………

Answer. 180°

Question 52. Symbolic form of “The line I is parallel to the line m”……….

Answer. 1 || m

Question 53. The angle shown in the figure represents …… angle

Class 6 Maths Chapter 4 Basic Geometrical Ideas Fill in the blanks Question 58

Answer. reflex

Question 54. In the figure x° = …………

Class 6 Maths Chapter 4 Basic Geometrical Ideas Fill in the blanks Question 59

Answer. 90°

Question 55. The union of two rays having same initial point is called …………..

Answer. angle

Question 56. Ray is a part of line (T/F)

Answer. True

Question 57. In the figure p || q, r is a transversal then x° = …………

Class 6 Maths Chapter 4 Basic Geometrical Ideas Fill in the blanks Question 62

Answer. 70°

Question 58………………………… is one that does not cross itself.

Answer. simple curve

Question 59. ……….is the union of two rays with a common point.

Answer. Angle

Question 60. A triangle with its boundary and its interior is called the ………..

Answer. Triangle region

Question 61. Number of end points of a line segment ……….

Answer. 2

Question 62.

Class 6 Maths Chapter 4 Basic Geometrical Ideas Fill in the blanks Question 67

Vertex of this angle ………….

Answer. B

Question 63.

Class 6 Maths Chapter 4 Basic Geometrical Ideas Fill in the blanks Question 68

Number of closed figures in the adjoining diagram……

Answer. 3

Question 64. Mathematical form of a line segment PQ is.

Answer. PQ

Question 65. How we denote triangle ABC?..

Answer. ΔΑΒC

Question 66. Parallel lines never

Answer. Intersect

Question 67. If two lines \(\overleftrightarrow{\mathrm{AB}}\) and \(\overleftrightarrow{\mathrm{CD}}\) are parallel, we write.

Answer. AB ||CD

Question 68. In a closed curve there are……………………… disjoint parts.

Answer. 3

Question 69. The interior of a curve together with its boundary is called …..

Answer. region

Question 70. A polygon with least number of sides is ……..

Answer. triangle

Haryana Board Class 6 Maths Solutions For Chapter 2 Whole Numbers

Haryana Board Class 6 Maths Solutions For Chapter 2 Whole Numbers

  • If we add 1 to a number we get successor of that number.
  • The natural numbers along with zero form the collection of whole numbers.
  • Whole numbers are closed under addition and also under multiplication is called closure property.
  • Division means repeated subtraction.
  • Division of whole number by ‘0’ is not defined.
  • The addition of two numbers can be done in any order.
  • We can multiply two whole numbers in any order.
  • Addition and multiplication are commutative for whole numbers.
  • ‘0’ is the additive identity of whole numbers.
  • ‘1’ is the multiplicative identity for whole numbers.
  • The numbers which we use for counting are known as natural numbers. It is denoted by ‘N’.
    Set of natural numbers N = {1, 2, 3, 4, …………..}
  • The natural numbers along with the zero are called whole numbers. It is denoted by ‘W’.
    Set of whole numbersW= {0, 1, 2, 3, ……………}
  • The number just before any natural number is called the ‘Predecessor’.
  • The next number of any natural number is called the ‘Successor’.
  • Every natural number has a successor. Every natural number except 1 has a predecessor.
  • Every whole number has a successor. Every whole number except zero has a predecessor.
  • All natural numbers are whole numbers and all whole numbers except zero are natural numbers.
  • Whole numbers can be represented on a number line. Operations of addition, subtraction and multiplication can easily be performed on a number line.
  • Addition corresponds to moving to the right on the number line, whereas subtraction corresponds to moving to the left. Multiplication corresponds to making jumps of equal distance from zero.
  • “The sum of any two whole numbers is always a whole number”. This property is known as the closure property under addition for whole numbers.
    Eg: If a, b are two whole numbers then their sum ‘a + b’ is also a whole number.
  • The product of any two whole numbers is always a whole number. This property is known as the ‘closure property under multi- plication for whole numbers’.
    Eg: If a, b are two whole numbers, their product ‘a x b’ is also a whole number.
  • Whole numbers are closed under addition and multiplication. But, whole numbers are not closed under subtraction and division.
  • Division by zero is not defined.
  • If ‘a’ and ‘b’ are two whole numbers then ‘a + b = b + a’ this property is known as ‘Commutative Property’ under addition of whole numbers.
  • If ‘a’ and ‘b’ are two whole numbers then ‘a x b = b x a’ this property is known as ‘Commutative Property’ under multiplication of whole numbers.
  • Addition and multiplication are ‘Commutative’ over whole numbers. But whole. numbers are not commutative under subtraction and division.
  • If a, b and c are three whole numbers then ‘a x (b + c) = (a x b) + (a x c)’ is known as ‘distributive property of multiplication over addition’.
  • Addition and multiplication are ‘Associative’ over whole numbers. But whole numbers not associative under subtraction and division.
  • If a, b and c are three whole numbers then ‘ax (b+c) = (a x b) + (ax c)’ is known as ‘distributive property of multiplication over addition’.
  • Commutativity, associativity and distributivity of whole numbers are useful in simplifying calculations. We often use them without being aware of them.
  • Pattern with numbers are not only interesting but also useful especially for mental calculations. They help us to understand properties of numbers better.

Question 1. Write the predecessor and successor of 19; 1997; 12,000; 49; 1,00,000

Solution.

Class 6 Maths Chapter 2 Whole Numbers Question 1

Question 2. Is there any natural number that has no predecessor ?

Solution. Yes, the natural number 1 has no predecessor.

Question 3. Is there any natural number which has no successor? Is there a last natural number?

Solution. No, there is no natural number which has no successor. No, there is no last natural number..

Question 4. Are all natural numbers also whole numbers ?

Solution. Yes, all natural numbers are also whole numbers.

Question 5. Are all whole numbers also natural numbers ?

Solution. No, because the whole number ‘O’ is not natural number.

Question 6. Which is the greatest whole number?

Solution. There is no greatest whole number.

Haryana Board Class 6 Maths Whole Numbers solutions

Question 7. Find 4 + ; 2 + 6; 3 + 5 and 1 + 6 using the number line.

Solution.

Class 6 Maths Chapter 2 Whole Numbers 4 + 5

∴ 4 + 5 = 9

Class 6 Maths Chapter 2 Whole Numbers 2 + 6

∴ 2 + 6 = 8

Class 6 Maths Chapter 2 Whole Numbers 3 + 5

∴ 3 + 5 = 8

Class 6 Maths Chapter 2 Whole Numbers 1 + 6

∴ 1 + 6 = 7

Question 8. Find 8 – 3; 6 – 2; 9 – 6 using number line.

Solution. 8 – 3

Class 6 Maths Chapter 2 Whole Numbers 8 - 3

∴ 8 – 3 = 5

6 – 2

Class 6 Maths Chapter 2 Whole Numbers 6 - 2

∴ 6 – 2 = 4

9 – 6

Class 6 Maths Chapter 2 Whole Numbers 9 - 6

∴ 9 – 6 = 3

Properties of whole numbers Class 6 HBSE

Question 9. Find 2 x 6; 3 x 3; 4 x 2 using the number line.

Solution.

Class 6 Maths Chapter 2 Whole Numbers 2 x 6

∴ 2 x 6 = 12

Class 6 Maths Chapter 2 Whole Numbers 3 x 3

∴ 3 x 3 = 9

Class 6 Maths Chapter 2 Whole Numbers 4 x 2

∴ 4 × 2 = 8

Chapter 2 Whole Numbers Exercise – 2.1

Question 1. Write the next three natural numbers af- ter 10999.

Solution. The next three natural numbers after 10999 are 11,000, 11,001 and 11,002.

Question 2. Write the three whole numbers occuring just before 10001.

Solution. The three whole numbers occurring just before 10001 are 10000, 9999 and 9998.

Question 3. Which is the smallest whole number?

Solution. The smallest whole number is 0.

Question 4. How many whole numbers are there between 32 and 53 ?

Solution. There are 20 whole numbers between 32 and 53. They are 33, 34, 35, 36, 37, 38, 39, 40, 41, 42, 43, 44, 45, 46, 47, 48, 49, 50, 51 and 52.

Question 5. Write the successor of:

(a) 2440701

(b) 100199

(c) 1099999

(d) 2345670

Solution. a) The successor of 2440701 is 2440701 + 1 = 2440702

b) The successor of 100199 is 100199 + 1 = 100200

c) The successor of 1099999 is 1099999 + 1 = 1100000

d) The successor of 2345670 is 2345670 + 1 = 2345671

Whole numbers and their operations Class 6

Question 6. Write the predecessor of:

(a) 94

(b) 10000

(c) 208090

(d) 7654321

Solution. a) The predecessor of 94 is 94 – 1 = 93

b) The predecessor of 10000 is 10000 – 1 = 9999

c) The predecessor of 208090 is 208090 – 1 = 208089

d) The predecessor of 7654321 is 7654321 – 1 = 7654320

Question 7. In each of the following pairs of numbers, state which whole number is on the left of the other number on the number line. Also write them with the appropriate sign (>, <) between them.

(a) 530,503

(b) 370,307

(c) 98765, 56789

(d) 9830415, 10023001

Solution. a) The whole number 503 is on the left of the whole number 530 on the num- ber line.

∴ 503 < 530.

b) The whole number 307 is on the left of the whole number 370 on the num- ber line.

∴ 307 < 370.

c) The whole number 56789 is on the left” of the whole number 98765 on the number line.

∴ 56789 < 98765

d) The whole number 10023001 is on the right of the whole number 9830415 on the number line.

∴ 10023001 > 9830415.

Question 8. Which of the following statements are true (T) and which are false (F)?

a) Zero is the smallest natural number.

Solution. False (F)

b) 400 is the predecessor of 399.

Solution. False (F)

c) Zero is the smallest whole number.

Solution. True (T)

d) 600 is the successor of 599.

Solution. True (T)

e) All natural numbers are whole numbers.

Solution. True (T)

f) All whole numbers are natural numbers.

Solution. False (F)

Number line representation of whole numbers HBSE

g) The predecessor of a two digit number is never a single digit number.

Solution. False (F)

h) 1 is the smallest whole number.

Solution. False (F)

i) The natural number 1 has no predecessor.

Solution. True (T)

j) The whole number 1 has no predecessor.

Solution. False (F)

k) The whole number 13 lies between 11 and 12.

Solution. False (F)

l) The whole number 0 has no predecessor.

Solution. True (T)

m) The successor of a two digit number is always a two digit number.

Solution. False (F)

Haryana Board Class 6 Maths Solutions For Chapter 2 Whole Numbers Very Short Answer Questions

Question 1. Which is the smallest whole number?

Solution. ‘0’ is the smallest whole number.

Question 2. Are all natural numbers whole numbers ?

Solution. Yes, all natural numbers are whole numbers because all natural numbers along with zero are called whole numbers.

Question 3. Are all whole numbers natural numbers ?

Solution. No, all whole numbers are not natural numbers. Except zero the remaining all whole numbers are natural numbers.

Question 4. How many whole numbers are there in between 27 and 46?

Solution. The whole numbers lie between 27 and 46 are 28, 29, 30, 31, 32, 33, 34, 35, 36, 37, 38, 39, 40, 41, 42, 43, 44, 45.

There are 18 whole numbers in between 27 and 46:

Question 5. Mark the smallest whole number on the number line.

Solution. The smallest whole number is zero (0).

Class 6 Maths Chapter 2 Whole Numbers Very Short Answer Question 5

Distributive property of whole numbers Class 6

Question 6. Choose the appropriate symbol form < or > and place it in the blanks.

1) 8 > 7

2) 5 > 2

3) 0 < 1

4) 10 > 5

Question 7. Present the successor of 11 and predecessor of 5 on the number line.

Solution. The successor of 11 is 12 and the predecessor of 5 is 4.

Class 6 Maths Chapter 2 Whole Numbers Very Short Answer Question 7

Haryana Board Class 6 Maths Solutions For Chapter 2 Whole Numbers Short Answer Questions

Question 8. Which of the statements are true (T) and which are false (F). Correct the false statements.

1) There is a natural number that has no predecessor.

Solution. True.

1 is a natural number that has no predecessor.

2) Zero is the smallest whole number.

Solution. True.

3) A whole number on the left of another number on the number line, is greater than that number.

Solution. False.

A whole number on the left of another number on the number line is smaller than that number.

Question 9. Show (1) 3+4 (2) 7-3 on number line.

1) 3+ 4

Solution. Consider 3 + 4

Class 6 Maths Chapter 2 Whole Numbers Short Answer Question 9.1

Start from 3. We add 4 to three. We make 4 jumps to the right on the number line as shown above then we will reach at 7

∴ 3 + 4 = 7

2) 7 – 3

Solution. Consider 7 – 3

Class 6 Maths Chapter 2 Whole Numbers Short Answer Question 9.2

Start from 7. Since we subtract 3 from 7 we take 3 steps to the left on the number line as shown above then we will reach 4.

∴ 7 – 3 = 4.

Question 10. Show (1) 5 x 2 (2) 4 x 3 on number line.

Solution. Consider 5 x 2 = 2 + 2 + 2 + 2 + 2

Class 6 Maths Chapter 2 Whole Numbers Short Answer Question 10.1

Start from 0. Move 2 units to the right each time, making 5 such moves we reach 10.

So, 5 x 2 = 10

2) 4 × 3

Solution. Consider 4 x 3 = 3 + 3 + 3 + 3

Class 6 Maths Chapter 2 Whole Numbers Short Answer Question 10.2

Start from 0. Move 3 units to the right each time, making 4 such moves we reach 12.

So, 4 x 3 = 12

Question 11. Find the sum of the predecessor of 300 and successor of 427.

Solution. The predecessor of 300 is 299.

The successor of 427 is 428.

∴ The required sum = 299 + 428

= 727

Haryana Board Class 6 Maths Solutions For Chapter 2 Whole Numbers Long Answer Questions

Question 12. Show these on number line:

1) 5+3

Answer.

Class 6 Maths Chapter 2 Whole Numbers Long Answer Question 12.1

5 + 3 = 8

2) 5 – 3

Answer.

Class 6 Maths Chapter 2 Whole Numbers Long Answer Question 12.2

5 – 3 = 2

Word problems on whole numbers for Class 6 HBSE

3) 3 + 5

Answer.

Class 6 Maths Chapter 2 Whole Numbers Long Answer Question 12.3

3 + 5 = 8

4) 10 + 1

Answer.

Class 6 Maths Chapter 2 Whole Numbers Long Answer Question 12.4

10 + 1 = 11

5) 8 – 5

Answer.

Class 6 Maths Chapter 2 Whole Numbers Long Answer Question 12.5

8 – 5 = 3

Question 13. Find the following using number line.

1) 6 + 7 + 7

Solution.

Class 6 Maths Chapter 2 Whole Numbers Long Answer Question 13.1

Start from 0, reach 6; we make 7 jumps from 6 to the right on the number line as shown above. We will reach 13.

Again start from 13, make 7 jumps to the right on the number line as shown above. We will reach 20.

6 + 7 + 7 = 20

2) 18 – 9

Solution.

Class 6 Maths Chapter 2 Whole Numbers Long Answer Question 13.2

Start from 0, reach 18. Since we subtract 9 from 18, Start from 18. We take 9 steps to the left on the number line as shown above. We will reach 9.

18 – 9 = 9

Identity elements in whole numbers Class 6

3) 5 x 3

Solution.

Class 6 Maths Chapter 2 Whole Numbers Long Answer Question 13.3

Start from 0, move 5 units to the right each time making 3 such moves. We will reach 15.

5 x 3 = 15

Haryana Board Class 6 Maths Solutions For Chapter 2 Whole Numbers Objective Type Questions

Choose the correct answer :

Question 1. Natural numbers are represented by

  1. I
  2. W
  3. N
  4. O

Answer. 3. N

Question 2. The successor of 16 is

  1. 17
  2. 15
  3. 18
  4. 16

Answer. 1. 17

Question 3. The predecessor of 25 is

  1. 26
  2. 25
  3. 24
  4. 27

Answer. 3. 24

Question 4. The smallest whole number is

  1. 0
  2. 1
  3. 2
  4. 3

Answer. 1. 0

Question 5. Which number has no predecessor in natural numbers?.

  1. 0
  2. 1
  3. 2
  4. 3

Answer. 2. 1

Question 6. ……. + 25 = 25

  1. 1
  2. 0
  3. 2
  4. 25

Answer. 2. 0

Question 7. The number just before a number is called.

  1. Predecessor
  2. Successor
  3. Both A & B
  4. None

Answer. 1. Predecessor

Question 8. The next number of any natural number is called.

  1. Predecessor
  2. Successor
  3. Both A & B
  4. None

Answer. 2. Successor

Question 9. The predecessor of the smallest natural number

  1. 0
  2. 1
  3. 2
  4. None

Answer. 4. None

Question 10. The successor of the greatest whole number

  1. 0
  2. 1
  3. 100
  4. cannot find

Answer. 4. cannot find

Question 11. Which of the following whole number does not have predecessor

  1. 0
  2. 1
  3. 99
  4. None

Answer. 1. 0

Question 12. Whole numbers are represented by

  1. I
  2. W
  3. N
  4. R

Answer. 2. W

Question 13. Additive identity of whole numbers is

  1. 1
  2. 2
  3. -1
  4. 0

Answer. 4. 0

Question 14. Multiplicative identity of whole numbers is

  1. 1
  2. 2
  3. -1
  4. 0

Answer. 1. 1

Question 15. Taking ‘2 steps four times’ represented by

  1. 2 + 4
  2. 4 + 2
  3. 4 x 2
  4. 2 × 4

Answer. 3. 4 x 2

Question 16. The sum of smallest natural number and smallest whole number is

  1. 0
  2. 1
  3. 2
  4. Not defined

Answer. 2. 1

Question 17. Division by zero is

  1. 0
  2. 1
  3. same number
  4. Not defined

Answer. 4. Not defined

Question 18. The sum of first 5 natural numbers

  1. 14
  2. 15
  3. 16
  4. 10

Answer. 2. 15

Question 19. The sum of first 5 whole numbers

  1. 14
  2. 15
  3. 10
  4. 16

Answer. 3. 10

Question 20. ……..+46 = 46

  1. 1
  2. 0
  3. 46
  4. 2

Answer. 2. 0

Question 21. Set of whole numbers

  1. {1, 2, 3, 4,……}
  2. {0,1,2,3,…………….}
  3. {-3, -2, -1, 0, 1, 2, 3,…….}
  4. None

Answer. 2. {0,1,2,3,…………….}

Question 22. Smallest whole number

  1. 0
  2. 1
  3. -1
  4. Not exist

Answer. 1. 0

Question 23. The predecessor of 1000 is

  1. 1001
  2. 999
  3. 1000
  4. 998

Answer. 2. 999

Question 24. The successor of 9999 is

  1. 9998
  2. 9999
  3. 10000
  4. 10001

Answer. 3. 10000

Question 25. Which number has no predecessor?

  1. 1 in natural numbers
  2. 0 in whole numbers
  3. Both A & B
  4. Neither A nor B

Answer. 3. Both A & B

Question 26. Which of the following represents the given number line?

Class 6 Maths Chapter 2 Whole Numbers Choose The Correct Answer Question 28

4 + 2 = 6

4 x 2 = 8

2 × 4 = 8

10 – 2 = 8

Answer. 2. 4 x 2 = 8

Question 27. The difference of predecessor of 1000 and successor of 998 is……..

1

2

3

0

Answer. 4. 0

Chapter 2 Whole Numbers Fill in the blanks:

Question 28. The numbers 1, 2, 3 ….. which we use for counting are known as …………..

Answer. natural numbers

Question 29. The natural numbers along with the zero form the collection of ……… numbers.

Answer. whole

Question 30. The number just before a number is called the …………..

Answer. predecessor

Question 31. The next number of any natural number is called its …………

Answer. successor

Question 32. 3, 4 relation between these numbers is …………..

Answer. 4 > 3

Question 33. The sum of any two whole numbers is always a ………..

Answer. whole number

Question 34. …… by zero is not defined.

Answer. Division

Question 35. Every whole number has a ………

Answer. Successor

Question 36. We add …….. to a whole number we get the same whole number.

Answer. 0

Question 37. Every number can be arranged as a …………..

Answer. line

Question 38. ………… is the smallest whole number.

Answer. 0

Question 39. The number of whole numbers between 1 and 2 is ……..

Answer. 0

Question 40. 2021 ÷ 0 is ……..

Answer. Not defined

Question 41. ……. is the additive identiy.

Answer. 0

Haryana Board Class 7 Maths Solutions For Chapter 2 Fractions and Decimals

Haryana Board Class 7 Maths Solutions For Chapter 2 Fractions and Decimals

  • Fraction: The numbers of the form \(\frac{a}{b}\), where a and b are whole numbers and b #0, are called “fractions”.
  • Types of fractions:
    1. Proper fraction: In a proper fraction, the numerator is less than the denominator
      Examples: \(\frac{14}{19}, \frac{7}{9}, \frac{2}{3}, \frac{6}{13}, \frac{3}{4}\)
    2. Improper fraction: In an improper fraction, the .numerator is bigger than or equal to the denominator.
      Examples: \(\frac{9}{8}, \frac{7}{4}, \frac{21}{8}, \frac{35}{17}, \frac{43}{19}, \frac{4}{4}\)
    3. Mixed fraction: It is a combination of a whole number and a proper fraction
      Examples: \(1 \frac{3}{4}, 4 \frac{1}{9}, 5 \frac{6}{11}, 7 \frac{3}{4}, 3 \frac{4}{7}\)
      An improper fraction can be converted into a mixed fraction
      Examples: \(\frac{9}{8}=1 \frac{1}{8}\)
      \(\frac{21}{8}=2 \frac{5}{8}\)
    4. Fractions such as \(\frac{1}{2}, \frac{2}{4}, \frac{3}{6}\)…………. are called equivalent fractions
    5. ) Like fractions: Fractions with same denominators are called like fractions
      Examples: \(\frac{1}{7}, \frac{2}{7} ; \frac{3}{5}, \frac{2}{5}\)
    6.  Unlike fractions: Fractions with different denominators are called unlike fractions
      Examples: \( \frac{8}{9}, \frac{2}{7}, \frac{18}{17} \)……….
  • Division of fractions: To divide a fraction with another fraction we multiply with its reciprocal.
  • Example: \( \frac{2}{3} \div \frac{3}{4}=\frac{2}{3} \times \frac{4}{3}=\frac{8}{9} \)
  • Decimal fractions or Decimal numbers:
  • Fractions whose denominators are multiples of 10 only are called decimal fractions or decimal numbers.
    Example: \( 2.3=\frac{23}{10}, 0.47=\frac{47}{100} \text { etc. } \)
    Multiplication of decimal number by 10,100, 1000 etc.: When a decimal number is multiplied by16, 100, 1000 etc., the decimal point in the product shifts to the right as many zeros as in 10, 100, 1000 etc.
  • We can change an improper fraction to . a mixedfraction and vice – versa.
  • Multiplying a fraction with a whole number:
    To multiply a fraction with a whole number we multiply the whole number with the numerator and keeping the denominator same.
    Example: \( 2 \times \frac{7}{5}=\frac{14}{5} \)
  • Product of two fractions \( =\frac{\text { Product of Numerators }}{\text { Product of Denominators }}\)Example: \(\frac{5}{6} \times \frac{2}{7}=\frac{5 \times 2}{6 \times 7}=\frac{10}{42}\)
  • ‘of’ represents multiplication.
    Example: \(\frac{1}{2} \text { of } 3=\frac{1}{2} \times 3\)
  • Reciprocal of a fraction: If \( \frac{a}{b} \) then \( \frac{b}{a}\) is called its reciprocal.
  • A fraction means a part of a group of a region.
  • Every fraction contains a numerator and a denominator
    Example: In \(\frac{4}{7}\) is the numerator and 7 is the denominator.

Solutions To Try These

1. Find:

Solution: \( \frac{2}{7} \times 3=\frac{2 \times 3}{7}=\frac{6}{7}\)

2. \( \frac{9}{7} \times 6\)

Solution:

\( \frac{9}{7} \times 6=\frac{9 \times 6}{7}=\frac{54}{7}=7 \frac{5}{7}\)

3. \( 3 \times \frac{1}{8}\)

Solution:

\( 3 \times \frac{1}{8}=\frac{3 \times 1}{8}=\frac{3}{8}\)

HBSE Class 7 Fractions and Decimals Solutions

4. \( \frac{13}{11} \times 6\)

Solution:

\( \frac{13}{11} \times 6=\frac{13 \times 6}{11}=\frac{78}{11}\) \( =7 \frac{1}{11}\)

2. Represent pictorially = \( 2 \times \frac{2}{5}=\frac{4}{5}\)

Represent pictorially 2 x 2/5=4/5

 

Exercise – 2.1

1) Which of the drawings (1) to (4) show:

1. \( 2 \times \frac{1}{5}\)

2. \( 2 \times \frac{1}{2}\)

3. \( 3 \times \frac{2}{3}\)

4. \( 3 \times \frac{1}{4}\)

Which of the drawings (a) to (d) show

Solution:

1-d
2-b
3-a
4-c

Haryana Board Class 7 Maths Fractions and Decimals solutions

2. Some pictures (a) to (c) are given below. Tell which of them show:

1. \( 3 \times \frac{1}{5}=\frac{3}{5}\)

2. \( 2 \times \frac{1}{3}=\frac{2}{3}\)

3. \( 3 \times \frac{3}{4}=2 \frac{1}{4}\)

 

Some pictures (a) to (c) are given

Solution:

1. \( 3 \times \frac{1}{5}=\frac{3}{5}=(\mathrm{c})\)

2. \( 2 \times \frac{1}{3}=\frac{2}{3}=(a)\)

3. \( 3 \times \frac{3}{4}=2 \frac{1}{4}=(b)\)

HBSE 7th Class Fraction and Decimal Word Problems – Focuses on word problems in this chapter.

3. Multiply and reduce to lowest form and convert into a mixed fraction:

1. \( 7 \times \frac{3}{5}\)

Solution: \( 7 \times \frac{3}{5}=\frac{7 \times 3}{5}=\frac{21}{5}=4 \frac{1}{5}\)

2. \( 4 \times \frac{1}{3}\)

Solution: \( 4 \times \frac{1}{3}=\frac{4 \times 1}{3}=\frac{4}{3}=1 \frac{1}{3}\)

3. \( 2 \times \frac{6}{7}\)

Solution:\( 2 \times \frac{6}{7}=\frac{2 \times 6}{7}=\frac{12}{7}=1 \frac{5}{7}\)

4.\( 5 \times \frac{2}{9}\)

Solution:\( 5 \times \frac{2}{9}=\frac{5 \times 2}{9}=\frac{10}{9}=1 \frac{1}{9}\)

5. \( \frac{2}{3} \times 4\)

Solution:\( \frac{2}{3} \times 4=\frac{2 \times 4}{3}=\frac{8}{3}=2 \frac{2}{3}\)

6.\( \frac{5}{2} \times 6\)

Solution:\( \frac{5}{2} \times 6=\frac{5 \times 6}{2}=\frac{30}{2}=15\)

7. \( 11 \times \frac{4}{7}\)

Solution:\( 11 \times \frac{4}{7}=\frac{11 \times 4}{7}=\frac{44}{7}=6 \frac{2}{7}\)

8. \( 20 \times \frac{4}{5}\)

Solution:\( 20 \times \frac{4}{5}=\frac{20 \times 4}{5}=\frac{80}{5}=16\)

9.\( 13 \times \frac{1}{3}\)

Solution: \( 13 \times \frac{1}{3}=\frac{13 \times 1}{3}=\frac{13}{3}=4 \frac{1}{3}\)

10.\( 15 \times \frac{3}{5}\)

Solution:\( 15 \times \frac{3}{5}=\frac{15 \times 3}{5}=\frac{45}{5}=9\)

4. Shade:

1. \( \frac{1}{2}\) of the circles in box (1)

2. \( \frac{2}{3}\) of the trianglesin box (2)

3. \( \frac{3}{5}\) of the squares in box (3)

1)0.5 circles in a box 

2)2 3rd of triangles in a box

Sample Problems Fractions and Decimals Haryana Board Class 7

3) 3 of 5 squares in a box

Addition and subtraction of fractions Class 7 HBSE

Solution:

1)0.5 circles in a box 1

2)2 3rd of triangles in a box 1

3) 3 of 5 squares in a box 1

5. Find:

1) \( \frac{1}{2}\) of (1) 24 (2) 46

Solution: (1) \( \frac{1}{2}\) of 24 =\( \frac{1}{2} \times 24\)\( =\frac{1 \times 24}{2}=\frac{24}{2}=12\)

(2)\( \frac{1}{2} \text { of } 46=\frac{1}{2} \times 46\)

\( =\frac{1 \times 46}{2}=\frac{46}{2}=23\)

2)\( \frac{2}{3}\) of (1) 18 (2) 27

Solution:

1)\( \frac{2}{3}\) of 18 \( =\frac{2}{3} \times 18=\frac{2 \times 18}{3}=\frac{36}{3}=12 \)

(2)\( \frac{2}{3} \text { of } 27=\frac{2}{3} \times 27\)

\( =\frac{2 \times 27}{3}=\frac{54}{3}=18\)

3) \( \frac{3}{4}\) of (1) 16 (2) 36

Solution:

(1)\( \frac{3}{4} \text { of } 16=\frac{3}{4} \times 16=\frac{3 \times 16}{4}=\frac{48}{4}=12\)

(2)\( \frac{3}{4} \text { of } 36=\frac{3}{4} \times 36\)

\( =\frac{3 \times 36}{4}=\frac{108}{4}=27\)

4. \( \frac{4}{5} \) of (1) 20 (2) 35

Solution:

(1)\( \frac{4}{5} \text { of } 20=\frac{4}{5} \times 20=\frac{4 \times 20}{5}=\frac{80}{5}=16 \)

(2) \( \frac{4}{5} \text { of } 35=\frac{4}{5} \times 35 \)

\( =\frac{4 \times 35}{5}=\frac{140}{5}=28 \)

6. Multiply and express as a mixed fraction:

1) \( 3 \times 5 \frac{1}{5} \)

Solution:

\( 3 \times 5 \frac{1}{5}=3 \times\left(\frac{5 \times 5+1}{5}\right)=3 \times\left(\frac{25+1}{5}\right) \) \( =3 \times \frac{26}{5}=\frac{78}{5}=15 \frac{3}{5} \)

2) \( 5 \times 6 \frac{3}{4} \)

Solution:

\( 5 \times 6 \frac{3}{4}=5 \times\left(\frac{6 \times 4+3}{4}\right) \) \( =5 \times\left(\frac{24+3}{4}\right)=5 \times \frac{27}{4} \) \( =\frac{135}{4}=33 \frac{3}{4} \)

3) \( 7 \times 2 \frac{1}{4} \)

Solution: \( 7 \times 2 \frac{1}{4}=7 \times\left(\frac{2 \times 4+1}{4}\right) \)

\( =7 \times\left(\frac{8+1}{4}\right)=\frac{7 \times 9}{4}=\frac{63}{4}=15 \frac{3}{4} \)

4) \( 4 \times 6 \frac{1}{3} \)

Solution: \( 4 \times 6 \frac{1}{3}=4 \times\left(\frac{6 \times 3+1}{3}\right) \)

\( =4 \times\left(\frac{18+1}{3}\right)=\frac{4 \times 19}{3}=\frac{76}{3} \) \( =25 \frac{1}{3} \)

5) \( 3 \frac{1}{4} \times 6 \)

Solution: \( \left(\frac{3 \times 4+1}{4}\right) \times 6=\left(\frac{12+1}{4}\right) \times 6 \)

\( =\frac{13}{4} \times 6=\frac{13 \times 6}{4}=\frac{78}{4} \) \( =\frac{78 \div 2}{4 \div 2}=\frac{39}{2}=19 \frac{1}{2} \)

Multiplying and Dividing Fractions Class 7 Haryana Board

6) \( 3 \frac{2}{5} \times 8 \)

Solution: \( \left(\frac{3 \times 5+2}{5}\right) \times 8=\left(\frac{15+2}{5}\right) \times 8 \)

\( =\frac{17 \times 8}{5}=\frac{136}{5}=27 \frac{1}{5} \)

7) Find:

1) \( \frac{1}{2} of \) (1) \( 2 \frac{3}{4} \) (2)\( 4 \frac{2}{9} \)

Solution:

(1) \( \frac{1}{2} \text { of } 2 \frac{3}{4}=\frac{1}{2} \times\left(\frac{2 \times 4+3}{4}\right) \)

\( =\frac{1}{2} \times\left(\frac{8+3}{4}\right)=\frac{1}{2} \times \frac{11}{4} \) \( =\frac{1 \times 11}{2 \times 4}=\frac{11}{8}=1 \frac{3}{8}\)

(2) \( \frac{1}{2} \text { of } 4 \frac{2}{9}=\frac{1}{2} \times\left(\frac{4 \times 9+2}{9}\right) \)

\( =\frac{1}{2} \times\left(\frac{36+2}{9}\right)=\frac{1}{2} \times \frac{38}{9} \) \( =\frac{1 \times 38}{2 \times 9}=\frac{38}{18}=\frac{38 \div 2}{18 \div 2}=\frac{19}{9}=2 \frac{1}{9} \)

Multiplication and division of decimals Class 7 HBSE

2) \( \frac{5}{8} \) of (1) \( 3 \frac{5}{6} \) (2) \( 9 \frac{2}{3} \)

Solution:

(1) \( \frac{5}{8} \text { of } 3 \frac{5}{6} \)

\( =\frac{5}{8} \times\left(\frac{3 \times 6+5}{6}\right)=\frac{5}{8} \times\left(\frac{18+5}{6}\right) \) \( =\frac{5}{8} \times \frac{23}{6}=\frac{115}{48}=2 \frac{19}{48} \)

(2) \( \frac{5}{8} \text { of } 9 \frac{2}{3} \)

\( =\frac{5}{8} \times\left(\frac{9 \times 3+2}{3}\right)=\frac{5}{8} \times\left(\frac{27+2}{3}\right) \) \( =\frac{5}{8} \times \frac{29}{3}=\frac{5 \times 29}{8 \times 3}=\frac{145}{24}=6 \frac{1}{24} \)

 

8) Vidya and Pratap went for a picnic. Their mother gave them a water bottle that contained 5 litres of water. Vidya consumed \( \frac{2}{5} \) of the water. Pratap consumed the remaining water

(1) How much water did Vidya drink?

(2) What fraction of the total quantity of water did Pratap drink?

Solution:

Quantity of water in the bottle = 5 litres

(1) Water consumed by Vidya = \( \frac{2}{5} \) of 5 litres

\( =\frac{2}{5} \times 5=\frac{2 \times 5}{5}=\frac{10}{5}=2 \text { litres } \)

(2) Water consumed by Pratap = \( \frac{1}{1}-\frac{2}{5} \)

\( =\frac{5-2}{5}=\frac{3}{5} \text { litres } \)

 

Solutions To Try These

Find:

1. \( 7 \div \frac{2}{5} \)

Solution: \( 7 \div \frac{2}{5}=7 \times \frac{5}{2}=\frac{7 \times 5}{2}=\frac{35}{2}=17 \frac{1}{2} \)

2. \( 6 \div \frac{4}{7} \)

Solution: \( \begin{aligned}
6 \div \frac{4}{7} & =6 \times \frac{7}{4}=\frac{6 \times 7}{4} \\
& =\frac{42}{4}=\frac{42 \div 2}{4 \div 2}=\frac{21}{2}=10 \frac{1}{2}
\end{aligned} \)

3. \( 2 \div \frac{8}{9} \)

Solution: \( \begin{aligned}
2 \div \frac{8}{9}=2 \times \frac{9}{8} & =\frac{2 \times 9}{8}=\frac{18}{8}=\frac{18 \div 2}{8 \div 2} \\
& =\frac{9}{4}=2 \frac{1}{4}
\end{aligned} \)

Solutions To Try These

Find:

1. \( 6 \div 5 \frac{1}{3} \)

Solution: \( \begin{aligned}
6 \div 5 \frac{1}{3} & =6 \div \frac{16}{3}=6 \times \frac{3}{16}=\frac{6 \times 3}{16} \\
& =\frac{18}{16}=\frac{18 \div 2}{16 \div 2}=\frac{9}{8}=1 \frac{1}{8}
\end{aligned} \)

2. \( 7 \div 2 \frac{4}{7} \)

Solution:

\( \begin{aligned}
7 \div 2 \frac{4}{7} & =7 \div \frac{18}{7} \\
& =7 \times \frac{7}{18}=\frac{49}{18}=2 \frac{13}{18}
\end{aligned} \)

HBSE Class 7 Maths Chapter 2 Guide

Solutions To Try These

Find:

1. \( \frac{3}{5} \div \frac{1}{2} \)

Solution: \( \frac{3}{5} \div \frac{1}{2}=\frac{3}{5} \times \frac{2}{1}=\frac{3 \times 2}{5 \times 1}=\frac{6}{5}=1 \frac{1}{5} \)

2. \( \frac{1}{2} \div \frac{3}{5} \)

Solution:

\( \frac{1}{2} \div \frac{3}{5}=\frac{1}{2} \times \frac{5}{3}=\frac{1 \times 5}{2 \times 3}=\frac{5}{6} \)

3. \( 2 \frac{1}{2} \div \frac{3}{5} \)

Solution:

\( \begin{gathered}
2 \frac{1}{2} \div \frac{3}{5}=\frac{5}{2} \div \frac{3}{5}=\frac{5}{2} \times \frac{5}{3}=\frac{5 \times 5}{2 \times 3} \\
=\frac{25}{6}=4 \frac{1}{6}
\end{gathered} \)

4. \( 5 \frac{1}{6} \div \frac{9}{2} \)

Solution:

\( \begin{aligned}
5 \frac{1}{6} \div \frac{9}{2} & =\frac{31}{6} \div \frac{9}{2}=\frac{31}{6} \times \frac{2}{9} \\
& =\frac{31 \times 2}{6 \times 9}=\frac{62}{54}=\frac{62 \div 2}{54 \div 2} \\
& =\frac{31}{27}=1 \frac{4}{27}
\end{aligned} \)

Exercise 2.3

1. Find:

1. \( 12 \div \frac{3}{4} \)

Solution:

\( 12 \div \frac{3}{4}=\frac{12}{1} \times \frac{4}{3}=\frac{12 \times 4}{1 \times 3}=\frac{48}{3}=16 \)

2. \( 14 \div \frac{5}{6} \)

Solution:

\( 14 \div \frac{5}{6}=\frac{14}{1} \times \frac{6}{5}=\frac{14 \times 6}{1 \times 5}=\frac{84}{5}=16 \frac{4}{5} \)

3. \( 8 \div \frac{7}{3} \)

Solution:

\( 8 \div \frac{7}{3}=\frac{8}{1} \times \frac{3}{7}=\frac{8 \times 3}{1 \times 7}=\frac{24}{7}=3 \frac{3}{7} \)

4. \( 4 \div \frac{8}{3} \)

Solution:

\( \begin{aligned}
4 \div \frac{8}{3} & =\frac{4}{1} \times \frac{3}{8}=\frac{4 \times 3}{1 \times 8}=\frac{12}{8}=\frac{12 \div 4}{8 \div 4} \\
& =\frac{3}{2}=1 \frac{1}{2}
\end{aligned} \)

5. \( 3 \div 2 \frac{1}{3} \)

Solution:

\( \begin{aligned}
3 \div 2 \frac{1}{3}=3 \div \frac{7}{3}=\frac{3}{1} \times \frac{3}{7} & =\frac{3 \times 3}{1 \times 7} \\
& =\frac{9}{7}=1 \frac{2}{7}
\end{aligned} \)

Important Concepts Fractions and Decimals Class 7 HBSE

6. \( 5 \div 3 \frac{4}{7} \)

Solution:

\( \begin{aligned}
5 \div 3 \frac{4}{7} & =5 \div \frac{25}{7}=\frac{5}{1} \times \frac{7}{25}=\frac{5 \times 7}{1 \times 25} \\
& =\frac{35}{25}=\frac{35 \div 5}{25 \div 5}=\frac{7}{5}=1 \frac{2}{5}
\end{aligned} \)

2. Find the reciprocal of each of the following fractions. Classify the reciprocal as proper fraction,improper fraction and whole numbers.

1) \( \frac{3}{7} \)

Solution: \( \text { Reciprocal of } \frac{3}{7} \text { is } \frac{7}{3} \)

\( \frac{7}{3} \text { is an improper fraction. } \)

Word problems on fractions and decimals Class 7 HBSE

2) \( \frac{5}{8} \)

Solution: \( \text { Reciprocal of } \frac{5}{8} \text { is } \frac{8}{5} \)

\( \frac{8}{5} \text { is an improper fraction. } \)

3) \( \frac{9}{7} \)

Solution: \( \text { Reciprocal of } \frac{9}{7} \text { is } \frac{7}{9} \)

\( \frac{7}{9} \text { is a proper fraction. } \)

4) \( \frac{6}{5} \)

Solution: \( \frac{12}{7} \)

\( \text { Reciprocal of } \frac{6}{5} \text { is } \frac{5}{6} \) \( \frac{5}{6} \text { is a proper fraction. } \)

5) \( \frac{12}{7} \)

Solution:

\( \text { Reciprocal of } \frac{12}{7} \text { is } \frac{1}{12} \) \( \frac{7}{12} \text { is a proper fraction. } \)

6) \( \frac{1}{8} \)

Solution: \( \text { Reciprocal of } \frac{1}{8} \text { is } \frac{8}{1}=8 \)

∴ 8 is a whole number

7) \( \frac{1}{11} \)

Solution:

\( \text { Reciprocal of } \frac{1}{11} \text { is } \frac{11}{1}=11 \)

∴ 11 is a whole number.

3. Find:

1) \( \frac{7}{3} \div 2 \)

Solution:

\( \frac{7}{3} \div \frac{2}{1}=\frac{7}{3} \times \frac{1}{2}=\frac{7 \times 1}{3 \times 2}=\frac{7}{6}=1 \frac{1}{6} \)

2) \( \frac{4}{9} \div 5 \)

Solution:

\( \frac{4}{9} \div \frac{5}{1}=\frac{4}{9} \times \frac{1}{5}=\frac{4 \times 1}{9 \times 5}=\frac{4}{45} \)

3) \( \frac{6}{13} \div 7 \)

Solution:

\( \frac{6}{13} \div \frac{7}{1}=\frac{6}{13} \times \frac{1}{7}=\frac{6 \times 1}{13 \times 7}=\frac{6}{91}\)

4) \( 4 \frac{1}{3} \div 3 \)

Solution:

\( \begin{aligned}
4 \frac{1}{3} \div 3=\frac{13}{3} \div \frac{3}{1} & =\frac{13}{3} \times \frac{1}{3} \\
& =\frac{13 \times 1}{3 \times 3}=\frac{13}{9}=1 \frac{4}{9}
\end{aligned} \)

5) \( 3 \frac{1}{2} \div 4 \)

Solution:

\( 3 \frac{1}{2} \div 4=\frac{7}{2} \div \frac{4}{1}=\frac{7}{2} \times \frac{1}{4}=\frac{7 \times 1}{2 \times 4}=\frac{7}{8} \)

6) \( 4 \frac{3}{7} \div 7 \)

Solution:

\( \begin{aligned}
4 \frac{3}{7} \div 7=\frac{31}{7} \div \frac{7}{1} & =\frac{31}{7} \times \frac{1}{7} \\
& =\frac{31 \times 1}{7 \times 7}=\frac{31}{49}
\end{aligned} \)

4. Find:

1) \( \frac{2}{5} \div \frac{1}{2} \)

Solution:

\( \frac{2}{5} \div \frac{1}{2}=\frac{2}{5} \times \frac{2}{1}=\frac{2 \times 2}{5 \times 1}=\frac{4}{5} \)

2) \( \frac{4}{9}+\frac{2}{3} \)

Solution:

\( \begin{aligned}
\frac{4}{9} \div \frac{2}{3}=\frac{4}{9} \times \frac{3}{2} & =\frac{4 \times 3}{9 \times 2} \\
& =\frac{12}{18}=\frac{12 \div 6}{18 \div 6}=\frac{2}{3}
\end{aligned} \)

3) \( \frac{3}{7} \div \frac{8}{7} \)

Solution:

\( \frac{3}{7} \div \frac{8}{7}=\frac{3}{7} \times \frac{7}{8}=\frac{3 \times 7}{7 \times 8}=\frac{21}{56}=\frac{21 \div 7}{56 \div 7}=\frac{3}{8} \)

4) \( 2 \frac{1}{3} \div \frac{3}{5} \)

Solution:

\( \begin{aligned}
2 \frac{1}{3} \div \frac{3}{5}=\frac{7}{3} \div \frac{3}{5} & =\frac{7}{3} \times \frac{5}{3} \\
& =\frac{7 \times 5}{3 \times 3}=\frac{35}{9}=3 \frac{8}{9}
\end{aligned} \)

5) \( 3 \frac{1}{2} \div \frac{8}{3} \)

Solution:

\( \begin{aligned}
3 \frac{1}{2} \div \frac{8}{3}=\frac{7}{2} \div \frac{8}{3} & =\frac{7}{2} \times \frac{3}{8} \\
& =\frac{7 \times 3}{2 \times 8}=\frac{21}{16}=1 \frac{5}{16}
\end{aligned} \)

6) \( \frac{2}{5} \div 1 \frac{1}{2} \)

Solution:

\( \frac{2}{5} \div 1 \frac{1}{2}=\frac{2}{5} \div \frac{3}{2}=\frac{2}{5} \times \frac{2}{3}=\frac{2 \times 2}{5 \times 3}=\frac{4}{15} \)

7) \( 3 \frac{1}{5} \div 1 \frac{2}{3} \)

Solution:

\( \begin{aligned}
3 \frac{1}{5} \div 1 \frac{2}{3}=\frac{16}{5} \div \frac{5}{3} & =\frac{16}{5} \times \frac{3}{5} \\
& =\frac{16 \times 3}{5 \times 5}=\frac{48}{25}=1 \frac{23}{25}
\end{aligned} \)

8) \( 2 \frac{1}{5} \div 1 \frac{1}{5} \)

Solution:

\( \begin{aligned}
2 \frac{1}{5} \div 1 \frac{1}{5} & =\frac{11}{5} \div \frac{6}{5}=\frac{11}{5} \times \frac{5}{6}=\frac{11 \times 5}{5 \times 6}=\frac{55}{30} \\
& =\frac{55 \div 5}{30 \div 5}=\frac{11}{6}=1 \frac{5}{6}
\end{aligned} \)

1. Find:

1) 2.7×4

Solution: 2.7×4 = 10.8

2) 1.8 x 1.2

Solution: 1.8 x 1.2 = 2.16

3) 2.3 x 4.35

Solution: 2.3 x 4.35 = 10.005

2. Arrange the products obtained in (1) in descending order.

Solution:

The three products obtained in (1) are

10.8, 2.16,10.005. Their descending order is 10.8,10.005,2.16.

Solutions To Try These

Find:

1) 0.3 x 10

Solution: 0.3 x l0 = 3

2) 1.2×100

Solution: 1.2 x100 = 1.20 x100 = 120

3) 56.3 x1000

Solution: 56.3 x1000 = 56.300 x1000 = 56300

Exercise -2.4

1. Find:

1) 0.2 x 6

Solution: 0.2 x 6 = 1.2

2) 8 x 4.6

Solution: 8×4.6 = 36.8

3) 2.71 x 5

Solution: 2.71 x 5 = 13.55

4) 20.1 x 4

Solution: 20.1 x 4 = 80.4

5) 0.05 x 7

Solution: 0.05×7 = 0.35

6) 211.02×4

Solution: 211.02 x4 = 844.08

7) 2x 0.86

Solution: 2×0.86 = 1.72

2. Find the area of rectangle whose length is 5.7 cm and breadth is 3 cm.

Solution:

Length of the rectangle = 5.7 cm

Breadth of the rectangle = 3 cm

Area of the rectangle = Length x Breadth

= 5.7 cm x 3 cm = 17.1 cm2

3. Find:

1) 1.3 x 10

Solution: 1.3 x 10 = 13.0 or 13

2) 36.8 x 10

Solution: 36.8 x10 = 368.0 or 368

3) 153.7 x 10

Solution: 153.7 x 10 = 1537.0 or 1537

4) 168.07 x 10

Solution: 168.07 x 10 = 1680.7

5) 31.1 x 100

Solution: 31.1 x100 = 3110

6) 156.1 xl00

Solution: 156.1 x100 = 15610

7) 3.62 x 100

Solution: 3.62 x 100 = 362

8) 43.07 x100

Solution: 43.07 x 100 = 4307

9) 0.5 x 10

Solution: 0.5 x10 = 5

10) 0.08 x 10

Solution: 0.08 x 10 = 0.80 or 0.8

11) 0.9 x 100

Solution: 0.9 x 100 = 90.0 or 90

12) 0.03 x 1000

Solution: 0.03 x 1000 = 30.0 or 30

How to convert fractions to decimals Class 7

4. A two-wheeler covers a distance of 55.3 km with one litre of petrol. How much distance will it cover in 10 litres of petrol ?

Solution:

Distance covered with one litre of petrol = 55.3 km

Distance covered with10litres of petrol = 55.3 x10 = 553 km

5. Find:

1) 2.5 x 0.3

Solution: 2.5 x 0.3 = 0.75

2) 0.1 x 51.7

Solution: 0.1 x 51.7 = 5.17

3) 0.2 x 316.8

Solution: 0.2 x 316.8 = 63.36

4) 1.3 x 3.1

Solution: 1.3×3.1=4.03

5) 0.5 x 0.05

Solution: 0.5 x 0.05 = 0.025

6) 11.2 x 0.15

Solution: 11.2 x 0.15 =1.680

7) 1.07 x. 0.02

Solution: 1.07 X 0.02 = 0.0214 .

8) 10.05 x 1.05

Solution: 10.05 x 1.05 = 10.5525

9) 101.01 x 0.01

Solution: 101.01 x 0.01 = 1.0101

10) 100.01 x 1.1

Solution: 00.01 x 1.1 = 110.011

Solutions To Try These

1. Find:

1) 235.4 – 10

Solution: 235.4 + 10 = 23.54

2) 235.4 +100

Solution: 235.4 = 2.354

3) 235.4 +1000

Solution: 235.4 +1000 = 0.2354

2. Find:

1) 35.7 +3 = ?

Solution: 35.7+3 = 11.9

2) 25.5 +3 =?

Solution: 25.5 +3 = 8.5

Practice Problems Fractions and Decimals Class 7 Haryana Board

3. Find:

1) 43.15+5 = ?

Solution: 43.15 +5 = 4315 +5 = 863

43.15 +5 = 8.63

2) 82.44 +6 =?

Solution: 8244 + 6 = 1374

82.44 + 6 = 13.74

Solutions To Try These

1. Find:

1) 15.5+5

Solution: 155 +5=31

15.5 +5 = 3.1

2) 126.35 +7

Solution: 12635 + 7 = 1805

126.35 +7 = 18.05

2. Find:

1. \( \frac{7.75}{0.25} \)

Solution:

\( \frac{7.75}{0.25}=\frac{7.75 \times 100}{0.25 \times 100}=\frac{775}{25}=31 \)

2. \( \frac{42.8}{0.02} \)

Solution:

\( \frac{42.8}{0.02}=\frac{42.8 \times 100}{0.02 \times 100}=\frac{4280}{2}=2140 \)

3) \( \frac{5.6}{1.4} \)

Solution: \( \frac{5.6}{1.4}=\frac{5.6 \times 10}{1.4 \times 10}=\frac{56}{14}=4 \)

Exercise – 2.5

1. Find:

1. 0.4 ÷ 2

Solution:

\( 0.4 \div 2=0.4 \times \frac{1}{2}=\frac{4}{10} \times \frac{1}{2}=\frac{2}{10}=0.2 \)

2) 0.35 ÷ 5

Solution:

\( \begin{gathered}
0.35 \div 5=0.35 \times \frac{1}{5}=\frac{35}{100} \times \frac{1}{5} \\
\quad=\frac{5 \times 7}{100 \times 5}=\frac{7}{100}=0.07
\end{gathered} \)

3) 2.48 ÷ 4

Solution:

\( \begin{gathered}
2.48 \div 4=2.48 \times \frac{1}{4}=\frac{248}{100} \times \frac{1}{4} \\
=\frac{62}{100}=0.62
\end{gathered} \)

4) 65.4 ÷ 6

Solution:

\( \begin{aligned}
65.4 \div 6=65.4 \times \frac{1}{6}=\frac{654}{10} & \times \frac{1}{6} \\
& =\frac{109}{10}=10.9
\end{aligned} \)

5) 651.2 ÷ 4

Solution:

\( \begin{aligned}
651.2 \div 4=651.2 \times \frac{1}{4} & =\frac{6512}{10} \times \frac{1}{4} \\
& =\frac{1628}{10}=162.8
\end{aligned} \)

6) 14.49 ÷ 7

Solution:

\( \begin{aligned}
14.49 \div 7=14.49 \times \frac{1}{7} & =\frac{1449}{100} \times \frac{1}{7} \\
& =\frac{207}{100}=2.07
\end{aligned} \)

7) 3.96 ÷ 4

Solution:

\( \begin{aligned}
3.96 \div 4=3.96 \times \frac{1}{4}=\frac{396}{100} \times \frac{1}{4}= & \frac{99}{100} \\
& =0.99
\end{aligned} \)

8) 0.80 ÷ 5

Solution:

\( \begin{aligned}
0.80 \div 5=0.80 \times \frac{1}{5} & =\frac{80}{100} \times \frac{1}{5} \\
& =\frac{16}{100}=0.16
\end{aligned} \)

2. Find:

1) 4.8 ÷ 10

Solution:

\( \begin{aligned}
& 4.8 \div 10=4.8 \times \frac{1}{10} \\
& =\frac{48}{10} \times \frac{1}{10}=\frac{48}{100}=0.48
\end{aligned} \)

2) 52.5 ÷ 10

Solution:

\( \begin{aligned}
& 52.5 \div 10=52.5 \times \frac{1}{10}=\frac{525}{10} \times \frac{1}{10} \\
& =\frac{525}{100}=5.25
\end{aligned} \)

3) 0.7 ÷ 10

Solution:

\(
\begin{aligned}
& 0.7 \div 10=0.7 \times \frac{1}{10}=\frac{7}{10} \times \frac{1}{10} \\
& =\frac{7}{100}=0.07
\end{aligned} \)

4) 33.1 ÷ 10

Solution:

\( \begin{aligned}
33.1 & \div 10=33.1 \times \frac{1}{10}=\frac{331}{10} \times \frac{1}{10} \\
& =\frac{331}{100}=3.31
\end{aligned} \)

5) 272.23 ÷ 10

Solution: 272.23 ÷10 = 27.223

6) 0.56 ÷ 10

Solution: 0.56 ÷ 10 = 0.056

7) 3.97 ÷ 10

Solution: 3.97 ÷10 = 0.397

Key Questions in Fractions and Decimals for Class 7 HBSE

3. Find:

1) 2.7 ÷100

Solution: 2.7 ÷100 = 0.027

2) 0.3 ÷ 100

Solution: 0.3 ÷100 = 0.003

3) 0.78 ÷100

Solution: 0.78 ÷100 = 0.0078

4) 432.6 ÷100

Solution: 432.6÷100 = 4.326

5) 23.6 ÷100

Solution: 23.6 ÷100 = 0.236

6) 98.53 ÷100

Solution: 98.53 ÷100 = 0.9853

4. Find:

1) 7.9 ÷1000

Solution: 7.9 ÷1000 = 0.0079

2) 26.3 ÷1000

Solution: 26.3÷1000 = 0.0263

3) 38.53÷1000

Solution: 38.53÷1000 = 0.03853

4) 128.9÷1000

Solution: 128.9÷1000 = 0.1289

5) 0.5 ÷1000

Solution: 0.5 ÷1000 = 0.0005

5. Find:

1) 7 ÷ 3.5

Solution: \( 7 \div 3.5=\frac{7.0}{3.5}=\frac{70}{35}=2 \)

2) 36 ÷ 0.2

Solution: \( 36 \div 0.2=\frac{36.0}{0.2}=\frac{360}{2}=180 \)

3) 3.25 ÷ 0.5

Solution: \( 3.25 \div 0.5=\frac{3.25}{0.50}=\frac{325}{50}=6.5\)

4)30.94 ÷ 0.7

Solution: \( 30.94 \div 0.7=\frac{30.94}{0.70}=\frac{3094}{70}=44.2 \)

5) 0.5 ÷0.25

Solution: \( 0.5 \div 0.25=\frac{0.50}{0.25}=\frac{50}{25}=2 \)

6) 7.75 0.25

Solution: \( 7.75 \div 0.25=\frac{7.75}{0.25}=\frac{775}{25}=31 \)

7) 76.5 -0.15

Solution: \( 76.5 \div 0.15=\frac{76.50}{0.15}=\frac{7650}{15}=510 \)

8) 37.8 -1.4

Solution: \( 37.8 \div 1.4=\frac{37.8}{1.4}=\frac{378}{14}=27 \)

9) 2.73 -1.3

Solution: 2.73 -1.3

\( =\frac{2.73}{1.3}=\frac{2.73}{1.30}=\frac{273}{130}=\frac{21}{10}=2.1 \)

 

6. A vehicle covers a distance of 43.2 km in 2.4 litres of petrol. How much distance will it cover with one litre of petrol?

Solution: Distance covered with 2.4 litres of petrol = 43.2 km

Distance covered with1 litre ofpetrol = 43.2 4- 2.4

\( =\frac{43.2}{2.4}=\frac{432}{24}=18 \mathrm{~km} \)

Additional Questions

Very Short Answer Questions

1. Surya can walk \( \frac{18}{5} \) kmin an hour. How much distance can he walk in \( 2 \frac{1}{2} \) hours?

Solution:

The distance walked by Suryain an hour \( =\frac{18}{5} \mathrm{~km} \)

The distance walked by

\( \text { Surya in } 2 \frac{1}{2} \text { hours }=2 \frac{1}{2} \times \frac{18}{5} \)

\(\frac{5}{2}\) x \(\frac{18}{5}\)

= 9 km

2. If 24 students share \( 4 \frac{4}{5} \) kg of cake, then how much cake does each one get?

Solution:

Total number of students = 24

Total weight of cake \( =4 \frac{4}{5} \mathrm{~kg} \)

\( =\frac{24}{5} \mathrm{~kg} \)

The share of a cake that each one get

\( \begin{aligned}
& =\frac{24}{5} \div 24 \\
& =\frac{24}{5} \times \frac{1}{24}=\frac{1}{5} \mathrm{~kg}(200 \mathrm{~g})
\end{aligned} \)

3. If the cost of each cement bagis 326.50,then find the cost of 24 bags of cement.

Solution:

The cost of each cement bag = 326.50

The cost of 24 bags of cement = 24 x 326.50

= 7836

= 7836

4. Dharmika purchased chudidhar material of 1.40m at the rate of 152.5 per metre. Find the amount to be paid.

Solution:

The length of chudidhar material purchased by Dharmika = 1.40 m

The cost of material per meter = 152.5

The total amount to be paid = 1.40 x 152.5

= 213.5

= 213.50

5. If a picture chart costs 4.25. Amrutha wants to buy 16 charts to make an album. How much money does she have; to pay?

Solution:

The cost of picture chart = 4.25

Number of charts that she want to buy = 16

The amount of money she has to pay

= 4.25×16

= 68.00 = 68

6. Which is bigger \( \frac{5}{8} \text { or } \frac{3}{5} ? \)

Solution:

\( \begin{gathered}
\frac{5}{8}=\frac{5 \times 5}{8 \times 5}=\frac{25}{40}, \frac{3}{5}=\frac{3 \times 8}{5 \times 8}=\frac{24}{40} \\
\frac{25}{40}>\frac{24}{40} \text { and So, } \frac{5}{8}>\frac{3}{5}
\end{gathered} \)

[Hint: To compare, convert the fractions into like fractions]

Short Answer Questions

7. In Jagananna Gorumudda (MDM) scheme each student got \( \frac{3}{20} \) kg. rice per day, find the weight of the rice required for 60 students in a class per day.

Solution: The weight of rice for each student per day = \( \frac{3}{20} \) kg

Number of students in a class = 60

Total weight of rice required for 60 students in a class per day

\( =\frac{3}{20} \times 60 \)

\( \frac{3}{20} \times \frac{60}{1} \) = 3 x 3 = 9 kg

8. Find the product:

1. 32.5 x 8

Solution: 1) 32.5 x 8

\( \begin{aligned}
& =\frac{325}{10} \times 8 \\
& =\frac{2600}{10}
\end{aligned} \)

= 260.0

= 260

2. 94.62 x7

Solution: 94.62 x7

\( \begin{aligned}
& =\frac{9462}{100} \times 7 \\
& =\frac{66234}{100}
\end{aligned} \)

= 662.34

3.109.761 x 3

Solution: 109.761 x 3

\( \begin{aligned}
& =\frac{109761}{1000} \times 31 \\
& =\frac{3402591}{1000}
\end{aligned} \)

= 3402.591

4. 61 x 2.39

Solution: 61 x 2.39

\( \begin{aligned}
& =61 \times \frac{239}{100} \\
& =\frac{14579}{100}
\end{aligned} \)

= 145.79

9. Find the product of the following

1. 23.4×6
2. 681.25×9
3. 53.29×14
4. 8 x 2.52
5. 25 x 2.013

Solution:

1. 23.4 x 6

23.4 x 6 = 140.4

(or)

\( \begin{aligned}
23.4 \times 6 & =\frac{234}{10} \times 6 \\
& =\frac{1404}{10}=140.4
\end{aligned} \)

2. 681.25 x 9

681.25 x 9 = 6131.25

(or)

\( 681.25 \times 9=\frac{68125}{100} \times 9=\frac{613125}{100}=6131.25 \)

3. 53.29 x 14

53.29 x 14 – 746.06

or

\( \begin{aligned}
53.29 \times 14=\frac{5329}{100} \times 14 & =\frac{74606}{100} \\
& =746.06
\end{aligned} \)

4. 8 x-2.52

8 x 2.52 = 20.16

or

\( 8 \times 2.52=8 \times \frac{252}{100}=\frac{2016}{100}=20.16 \)

5. 25 x 2.013

25 x 2.013 = 50.325

or

\( 25 \times 2.013=25 \times \frac{2013}{1000}=\frac{50325}{1000}=50.325 \)

10. Represent \( 2 \frac{1}{4} \) pictorially. How many units are needed for this?

Solution:

representing 9 th 4 th of pictorially

The shaded region in the above figure represents the fraction \( 2 \frac{1}{4} \).

Three units are needed for this.

11. Arrange the following in ascending order.

1. \( \frac{5}{8}, \frac{5}{6}, \frac{1}{2} \)

2. \( \frac{2}{5}, \frac{1}{3}, \frac{3}{10} \)

Solution:

1. Given fractious are \( \frac{5}{8}, \frac{5}{6}, \frac{1}{2} \)

L.C.M. of the denominators 8, 6 and 2 = 24

Now \( \frac{5}{8}=\frac{5 \times 3}{8 \times 3}=\frac{15}{24} \)

\( \begin{aligned}
& \frac{5}{6}=\frac{5 \times 4}{6 \times 4}=\frac{20}{24} \\
& \frac{1}{2}=\frac{1 \times 12}{2 \times 12}=\frac{12}{24}
\end{aligned} \)

Clearly

\( \begin{aligned}
& \frac{12}{24}<\frac{15}{24}<\frac{20}{24} \\
& \frac{1}{2}<\frac{5}{8}<\frac{5}{6}
\end{aligned}\)

Second method

\( \frac{1}{2}=\frac{1 \times 5}{2 \times 5}=\frac{5}{10} \)

clearly 10 > 8 > 6

\( \begin{aligned}
& \frac{5}{10}<\frac{5}{8}<\frac{5}{6} \\
& \frac{1}{2}<\frac{5}{8}<\frac{5}{6}
\end{aligned} \)

2. Given fractions are \( \frac{2}{5}, \frac{1}{3}, \frac{3}{10} \)

LCM of the denominators 5, 3, 10 = 30

Now \( \begin{aligned}
& \frac{2}{5}=\frac{2 \times 6}{5 \times 6}=\frac{12}{30} \\
& \frac{1}{3}=\frac{1 \times 10}{3 \times 10}=\frac{10}{30} \\
& \frac{3}{10}=\frac{3 \times 3}{10 \times 3}=\frac{9}{30}
\end{aligned} \)

Clearly

\( \begin{aligned}
& \frac{9}{30}<\frac{10}{30}<\frac{12}{30} \\
& \frac{3}{10}<\frac{1}{3}<\frac{2}{5}
\end{aligned} \)

12. Write the following fractions in ascending order.

1. \( \frac{3}{2}, \frac{5}{2}, \frac{1}{2}, \frac{17}{2}, \frac{9}{2} \)

2. \( \frac{6}{5}, \frac{11}{10}, \frac{19}{5}, \frac{7}{10}, \frac{5}{10} \)

3. \( \frac{8}{3}, \frac{7}{6}, 3 \frac{1}{4}, \frac{5}{3}, \frac{11}{4} \)

Solution:

1. Ascending order :

\( \frac{1}{2}<\frac{3}{2}<\frac{5}{2}<\frac{9}{2}<\frac{17}{2} \)

2. \( \frac{6}{5}, \frac{11}{10}, \frac{19}{5}, \frac{7}{10}, \frac{5}{10} \)

LCM of denominators = 10

\( \frac{6}{5}=\frac{6}{5} \times \frac{2}{2}=\frac{12}{10} ; \frac{19}{5}=\frac{19}{5} \times \frac{2}{2}=\frac{38}{10} \)

Ascending order:

\( \begin{aligned}
& =\frac{5}{10}<\frac{7}{10}<\frac{11}{10}<\frac{12}{10}<\frac{38}{10} \\
& =\frac{5}{10}<\frac{7}{10}<\frac{11}{10}<\frac{6}{5}<\frac{19}{5}
\end{aligned} \)

3. \( \frac{8}{3}, \frac{7}{6}, 3 \frac{1}{4}, \frac{5}{3}, \frac{11}{4} \)

LCM of denominators = 12

\( \begin{aligned}
& \frac{8}{3}=\frac{8}{3} \times \frac{4}{4}=\frac{32}{12} ; \frac{7}{6}=\frac{7}{6} \times \frac{2}{2}=\frac{14}{12} \\
& 3 \frac{1}{4}=\frac{13}{4} \times \frac{3}{3}=\frac{39}{12} \\
& \frac{5}{3}=\frac{5}{3} \times \frac{4}{4}=\frac{20}{12} ; \frac{11}{4}=\frac{11}{4} \times \frac{3}{3}=\frac{33}{12}
\end{aligned} \)

Ascending order:

\( \begin{aligned}
& \frac{14}{12}<\frac{20}{12}<\frac{32}{12}<\frac{33}{12}<\frac{39}{12} \\
& =\frac{7}{6}<\frac{5}{3}<\frac{8}{3}<\frac{11}{4}<3 \frac{1}{4}
\end{aligned} \)

13. Determine if the following pairs are equal by writing each in their simplest form.

1. \( \frac{3}{8} \text { and } \frac{375}{1000} \)

2. \( \frac{18}{54} \text { and } \frac{23}{69} \)

3. \( \frac{6}{10} \text { and } \frac{600}{1000} \)

4. \( \frac{17}{27} \cdot \text { and } \frac{25}{45} \)

Solution:

\( \begin{aligned}
&\frac{3}{8} \text { is in the simplest form. }\\
&\frac{375}{1000}=\frac{25 \times 15}{25 \times 40}=\frac{15}{40}=\frac{5 \times 3}{5 \times 8}=\frac{3}{8}
\end{aligned} \)

Shortly, \( \frac{375}{1000}\) = \( \frac{3}{8} \)

2. \( \frac{18}{54} \) = \( \frac{1}{3} \) and \( \frac{23}{69} \) = \( \frac{1}{3} \)

\( \text { So, } \frac{18}{54}=\frac{23}{69}\)

3. \( \frac{6}{10} \) = \( \frac{3}{5} \) and \( \frac{600}{1000}\) = \( \frac{3}{5} \)

\( \text { So, } \frac{6}{10}=\frac{600}{100}\)

4. \( \frac{17}{27} \text { is in the simplest form. } \)

\( \frac{25}{45} \) = \( \frac{5}{9} \)

But \( \frac{17}{27} \neq \frac{5}{9}\)

So, they are not equivalent

14; Compute the following and express the result as a mixed fraction

1. \( 2+\frac{3}{4} \)

2. \( \frac{7}{9}+\frac{1}{3} \)

3. \( 1-\frac{4}{7} \)

4. \( 2 \frac{2}{3}+\frac{1}{2} \)

5. \( \frac{5}{8}-\frac{1}{6} \)

6. \( 2 \frac{2}{3}+3 \frac{1}{2} \)

Solution:

1) \( \begin{aligned}
& \begin{aligned}
2+\frac{3}{4}=\frac{2 \times 4+3}{4} & =\frac{11}{4}=2 \frac{3}{4} \\
\text { Alter }: 2+\frac{3}{4} & =\frac{2}{1}+\frac{3}{4}=\frac{8}{4}+\frac{3}{4} \\
& =\frac{8+3}{4}=\frac{11}{4}=2 \frac{3}{4}
\end{aligned}
\end{aligned} \)

2. \( \frac{7}{9}+\frac{1}{3}=\frac{7}{9}+\frac{3}{9}=\frac{7+3}{9}=\frac{10}{9}=1 \frac{1}{9} \)

3. \( 1-\frac{4}{7}=\frac{7}{7}-\frac{4}{7}=\frac{7-4}{7}=\frac{3}{7} \)

4. \( \begin{aligned}
2 \frac{2}{3}+\frac{1}{2}=\frac{8}{3}+\frac{1}{2}= & \frac{16}{6}+\frac{3}{6} \\
& =\frac{16+3}{6}=\frac{19}{6}=3 \frac{1}{6}
\end{aligned} \)

5. \( \frac{5}{8}-\frac{1}{6}=\frac{15}{24}-\frac{4}{24}=\frac{15-4}{24}=\frac{11}{24} \)

6. \( \begin{aligned}
2 \frac{2}{3} & +3 \frac{1}{2}=\frac{8}{3}+\frac{7}{2} \\
& =\frac{16}{6}+\frac{21}{6}=\frac{16+21}{6}=\frac{37}{6}=6 \frac{1}{6}
\end{aligned} \)

15. Check whether in this square the sum of the numbers in each row and in each column and along the diagonals is the same

Check whetherin this square the sum of the no

Solution: Sum of the fractions of first row

\( =\frac{6}{13}+\frac{13}{13}+\frac{2}{13}=\frac{6+13+2}{13}=\frac{21}{13} \)

Sum of the fractions of second row

\( =\frac{3}{13}+\frac{7}{13}+\frac{11}{13}=\frac{3+7+11}{13}=\frac{21}{13} \)

Sum of the fractions of third row

\( =\frac{12}{13}+\frac{1}{13}+\frac{8}{13}=\frac{12+1+8}{13}=\frac{21}{13} \)

Sum of the fractions of first column

\( =\frac{6}{13}+\frac{3}{13}+\frac{12}{13}=\frac{6+3+12}{13}=\frac{21}{13} \)

Sum of the fractions of second column

\( =\frac{13}{13}+\frac{7}{13}+\frac{1}{13}=\frac{13+7+1}{13}=\frac{21}{13} \)

Sum of the fractions of third column

\( =\frac{2}{13}+\frac{11}{13}+\frac{8}{13}=\frac{2+11+8}{13}=\frac{21}{13} \)

Sum of die fractions of the first diagonal

\( =\frac{6}{13}+\frac{7}{13}+\frac{8}{13}=\frac{6+7+8}{13}=\frac{21}{13} \)

Sum of the fractions of the second diagonal

\( =\frac{2}{13}+\frac{7}{13}+\frac{12}{13}=\frac{21}{13} \)

Thus, the sum of the numbers in each row and in each column and along the diagonals is \( \frac{21}{13} \) which is sam.

Hint: Such type of squares are called magic squares. You can try some more also.

 

Fill in the blanks:

101. Fractions with same denominators are called ………………

Answer: like fractions

102. The product of two improper fractions is………. the two fractions

Answer: greater than

103. A ……of a fraction is obtained by inverting it upside down.

Answer: reciprocal

104. \(\frac{2}{7}\) x …… = 1

Answer:

\(\left(\frac{7}{2}\right)\)

105.\(10 \frac{3}{7}=\)…….

Answer:

\(\left(\frac{73}{7}\right)\)

106. Simplest form of

\(\frac{16}{40}\) is………

Answer:

\(\left(\frac{2}{5}\right)\)

107. \(\frac{8}{15}\)…….\(\frac{2}{3}\) (Use > or <)

Answer: (<)

108. \(\frac{1}{4} \text { of } \frac{4}{3}=\) = …………

Answer:

\(\left(\frac{1}{3}\right)\)

109. 21.36 + 37.3 =………

Answer: (58.66)

110. How much less is 28 km than 42.6 km ?………..

Answer: (14.6 km)

111. Match the following:

1. \( \frac{1}{2}, \frac{2}{4}, \frac{3}{6} \text { are } \)              (  ) A) Like fractions

2. \( \frac{1}{7}, \frac{2}{7}, \frac{5}{7} \text { are }\)               (  ) B) Improper fractions

3. \( 1 \frac{3}{4}, 2 \frac{2}{3}, 3 \frac{5}{8} \text { are }\)      (  ) C) Decimal fractions

4. \( \frac{7}{4}, \frac{8}{5}, \frac{9}{7} \text { are }\)               (  ) D) Equivalent fractions

5. \( \frac{5}{10}, \frac{7}{100}, \frac{9}{1000} \text { are }\)   (  ) E) Mixed fractions

Answer:

1. D 2. A 3. E 4. B 5. C

 

Haryana Board Class 7 Maths Solutions For Chapter 1 Integers

Haryana Board Class 7 Maths Solutions For Chapter 1 Integers

  • Properties of integers under addition:
    1. Closure property: The sum of any two integers is also an integer. It is called “closure property” under addition in integers.
      Example: 2 + (-5) = -3,2 + 3 = 5 etc.
    2. Associative property: In general for any three integers a, b and c we have a + (b + c) = (a + b) + c
    3. Commutative property: for any two integers a, b we have a + b = b + a.
    4. Additive identity: There exists a number ‘0’ in integers such that for any integer a, we have a + 0 = a = 0 + a. ‘0’ is called the additive identity.
    5. Additive inverse: To each integer a, there is an integer -a such that a + (-a).= 0
      = -a + a.
      ‘-a’ is called the additive inverse of ‘a’.
  • Properties of integers under multiplication:
    1. Closure property:
      If a,b are two integers then a.b is also an integer.
    2. Associative property:
      If a, b and c are any three integers, then a(b. c)=(a .b). c
    3. Commutative property:
      If a, b are any two integers, we have a.b = b.a.
    4. Distributive property:
      If a, b,c are any three integers then we have
      a.(b + c)= a.b +a.c (Left distributive law)
      (a + b).c = a.c + b.c(Right distributive law)
    5. Multiplicative identity: There exists an element 1 in integers such that for any integer a, we have a.1 =1.a = a ‘1’ is called the multiplicative identity.
    6. Multiplication by zero: The product of any integer and zero is zero.
    7. Multiplicative inverse: To each integer ‘a’, there is an integer 1/a such that a x 1/a = 1/a x a = 1. ‘1/a’ is called multiplicative inverse of ‘a’.
  • Closure property under subtraction: The difference of any two integers is also an integer. i.e., a, b are integers ⇒ a -b is also an integer
  • The subtraction of integers is not commutative.
    i.e., a -b ≠ b – a where a, b are integers
  • Division of integers is not closed because 4 and 8 are integers but 4/8 = 1/2 is not an integer.
  • Division of integers is neither commutative nor associative.
  • Division with zero is not defined.
  1. We now study the properties satisfied by addition and subtraction.
    1. Integers are closed for addition and subtraction both. That is, a + b and a -b are again integers, where a and b are any integers.
    2. Addition is commutative for integers, i.e., a + b = b + a for all integers a and b.
    3. Addition is associative for integers, i.e., (a + b) + c = a + (b + c) for all integers a, b and c.
    4. Integer 0 is the identity under addition. That is, a + 0 = 0 + a = a for every integer a.
  2. We studied,how integers could be multiplied, and found that product of a positive and a negative integer is’a negative integer, whereas the product of two negative integers is a positive integer.
    For example, – 2 x 7 =- 14 and – 3 x – 8 = 24.
  3. Product of even number of negative integers is positive, whereas the product of odd number of negative integers is negative.
  4. Integers show some properties under multiplication.
    1. Integers are closed under multiplication. That is, a x b is an integer for any two integers a and b.
    2. Multiplication is commutative for integers. That is,a x b= b x a for any integers a and b.
    3. The integer 1 is the identity under multiplication, i.e.,l x a = a x l=a for any integer a.
    4. Multiplication is associative for integers, i.e., (a x b) x c = a x (b x c) for any three integers a, b and c.
  5. Under addition and multiplication, integers show a property called distributive property.
    That is, a x (b + c) = (a x b) + (a x c) for any three integers a, b and c.
  6. The properties of commutativity, associativity under addition and multiplication, and the distributive property help us to make our calculations easier.
  7. We also learnt how to divide integers. We found that
    1. When a positive integer is divided by a negative integer, the quotient obtained is negative and vice-versa.
    2. Division of a negative integer by another negative integer gives positive as quotient.
  8. For any integer a, we have
    1. a + 0 is not defined
    2. a +1= a

Solutions To Try These

1. Write a pair of integers whose sum gives

  1. a negative integer.
    Solution: -24 and -15
  2. zero
    Solution: -10 and 10
  3. an integer smaller than both the integers.
    Solution: -8 and -5
  4. an integer smaller than only one of the integers.
    Solution: -5 and 9
  5. an integer greater than both the integers.
    Solution: 6 and 10

HBSE Class 7 Integers Solutions

2. Write a pair of integers whose difference gives

  1. a negative integer.
    Solution: 5 and 7
  2. zero.
    Solution: 8 and 8
  3. an integer smaller than both the integers.
    Solution: -3 and1
  4. an integer greater than only one of the integers.
    Solution: 11 and 3
  5. an integer greater than both the integers.
    Solution: 9 and -5

Exercise 11

1. Write down a pair of integers whose:

  1. sum is -7
  2. difference is -10
  3. sum is 0

Solution:

  1. -15 and 8
  2. 8 and 18
  3.  -9 and 9

2. 1) Write a pair of negative integers whose difference gives 8.

Solution: -18 and -26

2) Write a negative integer and a positive integer whose sum is -5.

Solution: -8 and 3

3) Write a negative integer and a positive integer whose difference is -3.

Solution: -5 and 2

Haryana Board Class 7 Maths Integers solutions

3. In a quiz, team A scored – 40, 10, 0 and team B scored 10, 0, – 40 in three successive rounds. Which team scored more? Can we say that we can add integers in any order?

Solution:

Total score of team A = (- 40) + 10 + 0

=(- 40) + 10 = -30

Total score of team B = 10 + 0 + (- 40)

10 + (- 40)= -30

Both teams A and B scored equally.

Yes, we can add integers in any order.

Key Questions in Integers for Class 7 HBSE

4. Fill in the blanks to make the following statements true:

Solution:

  1. (-5) + (-8)= (-8) + (-5)
  2. -53 + 0 = -53
  3. 17 + (-17) = 0
  4. [13 (-12)] + (-7) = 13 + [(-12) + (-7)]
  5. (-4) + [15 + (-3)] = [-4 + 15] + (-3)

Solutions To Try These

1) Find 4 x (-8) using number line.

Solution:

4 x (-8) using number line

∴ 4 x (-8) = -32

2) Find 8 x (-2) using number line

Solution:

8 x (-2) using number line

∴ 8x (-2) = -16

3) Find 3 x (-7) usingnumber line.

Solution:

3 x (-7) usingnumber line

∴ 3X (-7) = -21

Practice Problems Integers Class 7 Haryana Board

4) Find 10 x (-1) using number line

Solution:

10 x (-1) using number line

∴ 10 x (-1) = -10

Solutions To Try These

Find:

1) 6 x (-19)

Solution: 6x (-19) = -(6×19) = -114

2) 12 x (-32)

Solution: 12 x (-32) =- (12 x 32) = -384

3) 7 x (-22)

Solution: 7 x (-22) = – (7 x 22) = -154

Solutions To Try These

1. Find:

1) 15 x (-16)

Solution.

15 x (-16) = – (15 x 16) = -240

2) 21 x (-32)

Solution: 21 x (-32) = – (21 x 32) = -672

3) (- 42) x 12

Solution. (- 42) x 12 = – (42 x 12) = -504

4) -55×15

Solution. (-55) x 15 = – (55 x 15) = -825

2. Check if

1) 25 x (-21) = (-25) x 21

Solution:

25 x (-21) = -(25 x 21) = -525

(-25) x 21 = -(25 x 21) = -525

∴ 25 x (-21) = (-25) x 21

2) (-23) x 20 = 23 x (-20)

Solution:

(-23) x 20 = -(23 x 20) = -460

23 x (-20) = -(23 x 20) = -460

∴  (-23) x 20 = 23 x (-20)

Other examples:

1) 10 x (-43) =43 x (-10)

2) (-29) x 25 = 29 x (-25)

3) (-63) x 37 = 63 x (-37)

4) 40 x (-28) = (-40) x 28

5) 30 x (-19) = (-30)x 19 .

Solutions To Try These

1) Starting from (-5) x 4, find (-5) x (-6)

Solution:

(-5) x 4 = -20 .

(-5) x 3 = -15 = [-20 + 5]

(-5) x 2 = -10 = [-15 + 5]

(-5) x 1 = -5 = [—10 + 5]

(-5) x 0 = 0 = [-5 + 5]

(-5) x (-1) = 5 = [0 + 5] ,

(-5) x (-2) = 10 = [5 + 5]

(-5) x (-3) = 15 = [10 + 5]

(-5) x (-4) =20 = [15 + 5] ‘

(-5) x (-5) =25 = [20 + 5]

(-5) x (-6) = 30 = [25 + 5]

Important Concepts Integers Class 7 HBSE

2) Starting from (- 6) x 3, find (- 6) x (-7)

Solution:

(- 6) x 3 = -18

(- 6) x 2 = -12 = [-18 + 6]

(- 6) x 1 = -6 = [-12 + 6]

(- 6) x 0 = 0 = [-6 + 6]

(- 6) x (-1) = 6 = [0 + 6]

(- 6) x (-2) = 12 = [6 + 6]

(- 6) x (-3) = 18 = [12 + 6]

(-6) x (-4) = 24 = [18 + 6]

(- 6) x (-5) = 30 = [24 + 6]

(- 6) x (-6) = 36 = [30 + 6]

(- 6) x (-7) = 42 = [36 + 6]

The product of two negative integers is a positive integer. We multiply the two negative integers as whole numbers and put the positive sign before the product.

(Negative integer) x (Negative integer) = Positive integer

Solutions To Try These

Find: (-31) x (-100), (-25) x(-72), (-83) x (-28).

1) (-31) x (-100)

Solution: (-31) x (-100) = 31 x 100 = 3100

2) (-25) x (-72)

Solution: (-25) x (-72) = 25 x 72 = 1800

3) (-83) x (-28)

Solution: (-83) x (-28) = 83 x 28 = 2324

Addition and subtraction of integers Class 7 HBSE

Solutions To Try These

1) Is 10 x [6 + (-2)] = 10 x 6 + 10 x (-2) ?

Solution:

10 x [6 + (-2)] =10 x 4 = 40

10 x 6 + 10 x (-2) = 60 -20 =40

∴ 10 x [6 + (-2)] =10 x 6 + 10 x (-2)

2) Is (-15) x [(-7) + (-1)] = (-15) x (-7) + (-15) x (-1) ?

Solution:

(-15) x [(-7) + (-1)] = (-15) x (-8) = 120

(-15) x (-7) + (-15)x (-1) = 105 + 15 = 120

∴ (-15) x [(-7) +,(-1)] = (-15) x (-7) + (-15) x (-1)

Solutions To Try These

1) Is 10 x [6-(-2)] = 10 x 6 – 10 x (-2) ?

Solution: 10 x [6 -(-2)] =10 x 8 = 80

10 x 6 -10 x (-2)= 60-(-20) = 80

∴ 10 x [6-(-2)] = 10 x 6-10 x (-2)

HBSE Class 7 Maths Chapter 1 Guide

2) Is (-15) x [(-7- (-1)] = (-15) x (-7) – (-15) x (-1) ?

Solution: (-15) x [(-7) – (-1)] = (-15) x (- 6) = 90

(-15) x (-7) – (-15) x (-1) =105-15 = 90

∴ (-15) x [(-7)- (-1)] = (-15)x (-7)-(-15) x (-1)

Exercise – 1.2:

1. Find each of the following products:

1) 3 x (-1)

Solution: 3 x (-1) = -(3X1)= -3

2) (-1) x 225

Solution: (-1) x 225 = – (1 x 225) = -225

3) (-21) x (-30)

Solution: (-21) x (-30) = 21 x 30 = 630

4) (-316) x (-1)

Solution: (-316) x (-1) = 316 x 1 = 316

5) (-15) x 0 x (-18)

Solution: (-15) x 0 x (-18)- [(-15) x 0] x (-18)= 0 x (-18) = 0

6) (-12) x (-11) x (10)

Solution: (-12)x(-11) x 10 = [(-12) x (-11)] x 10 = 132 x10 = 1320

7) 9 x(-3)x(-6)

Solution:

9 x (-3) x (-6) = 9 x [(-3)x(-6)]

= 9 x 18 = 162

8) (-18) x (-5) x (-4)

Solution: (-18) x (-5)x (-4) = [(-18)x (-5)]x (-4)= 90 x (-4) = -360

9) (-1) x (-2) x (-3) x 4

Solution: (-1) x (-2) x (-3) x 4

= [(-1) x (- 2)] x [(-3) x 4]

= 2 x (-12) = -24

Integer Operations Class 7 Haryana Board

10) (-3) x (-6) x (-2) x (-1)

Solution:

(-3) x (-6) x (-2) x (-1) =

[(-3) x (-6)]x[(-2)x(-l)]

=18 x 2 =36

2. Verify the following:

1) 18 x [7 + (-3)] = [18 x 7] + [18 x(-3)]

Solution:

18 x [7 + (-3)] = 18 [7-3] = 18x 4 = 72

[18 x 7] +[18 x (-3)] = 126 + (-54) = 72

∴ 18 x [7 +(-3)] = [18 x 7] + [18x (-3)]

2) (-21) x [(-4) + (-6)] = [(-21) x(-4)] + [(-21) x (-6)]

Solution:

(-21) x [(-4) + (-6)]= (- 21)x(-10) = 210

[(-21) x (-4)] +[(-21) x (-6)]

= 84 + 126 = 210

∴ (-21) X [(-4) + (-6)] = [(-21) x (-4)] + [(-21) x (-6)]

3. 1) For any integer a, what is (-1) x a equal to?

Solution: (-1) x a =-a

Multiplication and division of integers Class 7

2) Determine the integer whose product with (-1) is

  1. -22
  2. 37
  3. 0

Solution:

  1. (-1) x 22 = -22;
  2. (-1) x (-37)=37;
  3. (-1)x 0 = 0

4. Starting from (-1) x 5, write various products showing some pattern to show (-1) x (-1) = 1.

Solution:

(-1) x 5 = -5

(-1) x 4 = -4 = [(-5) + 1]

(-1) x 3 = -3 = [(-4) +l ]

(-1) x 2 = -2 = [(-3) +1 ]

(-1) x 1 = -1 = [(-2) + 1]

(-1) x 0 = 0 = [(-1) + 1]

(-1) x (-1)=1 = [0 + 1]

Very Short Answer Questions

1. Write four negative integers greater than -20.

Solution: -19, -18, -17 and -16.

2. Write any four negative integers less than -10.

Solution: -11, -12, -13 and -14.

3. Find: 50 -(-40) -(-2)

Solution: 50 – (-40) – (-2) =50 + 40 + 2 = 90 + 2 = 92

4. Write the properties of integers under addition.

Solution

  1. Closure property
  2. Commutative property
  3. Associative property
  4. Additive identity

5. Write down a pair of integers whose sum is 0.

Solution: (-25) + 25 = 0

6. Give an example for ‘Subtraction is not commutative for integers’.

Solution: Consider the integers 9 and (-5)

9- (-5) = 9 + 5 = 14 and

(-5) -9 = -5-9 = -14

∴ 9-(-5)=(-5)-9

7. Find (-45) x (-100).

Solution: (-45) x (-100) =4500

8. Find (-72) + 8.

Solution: (-72) + 8 = -9

9. Write the additive identity and multiplicative identity for integers.

Solution:

Additive Identity for integers is ‘0’.

MultiplicativeIdentity for integers is ‘1’.

HBSE 7th Class Integer Rules and Properties

10. The product of two integers is -165. If one number is -15, Find the other integer.

Solution:

The product of two integers = – 165

First number = -15

Second number = “x”

(-15) × “x” = -165

“x” = \(\frac{-165}{-15}=\frac{165}{15}\)

“x” =11

∴ Second number (integer) = 11

11. Because of COVID-19 a company locked down for 6 months and got loss of 1,32,000 in the year 2020. Find the average loss of each month.

Solution: A company locked down for 6 months because of covid-19

Loss of company in the year 2020 is 1,32,000

The average loss of each month = 1,32,000 ÷ 6 = 22,000

12. Write the additive inverses of 5, -8,1 and 0.

Solution:

The additive inverse of 5 is = -(+5) =-5

The additive inverse of -8 is = -(-8) = 8

The additive inverse of 1 is = -(+1)=-1

The additive inverse of 0 is = -(0) = 0

Integer number line Class 7 Haryana Board

13. Compute the following.

1) -36 ÷ (-4)

2) (-201) ÷ (-3)

3) (-325) ÷ (-13)

Solution:

1) -36 ÷ (-4) = 9

2) (-201) ÷ (-3) =67

3) (-325) ÷ (-13) =25

14. Write the following integers in ascending order (smallest to biggest).

1) -5, 2,1, -8
2) -4, -3, -5, 2
3) -10, -15, -7

Solution:

1) -8, -5,1, 2 (or) -8 < -5 <1 < 2
2) -5, -4, -3, 2 (or) -5 < -4 < -3 <2
3) -15, -10, -7 (or) -15 < -10 < -7

15. Write the following integers in descending order (biggest to smallest)

1) -2,-3,-5
2) -8,-2,-1
3) 5, 8, -2

Solution:

1)-2, -3, -5 (or) -2> -3> -5

2) -l,-2-8 (or) -1>-2>-8

3) 8,5,-2 (or) 8>5>-2

16. Multiply the following.

1) 5×7

Solution: 5×7=35

2) (-9) x (6)

Solution: (-9) x (6) =-(9×6) =-54

3) (9) x (-4)

Solution: (9) x (-4) = -(9 x 4) =-36

4) (8) x (-7)

Solution: (8) x (-7) =-(8×7) =-56

5) (-124) x (-1)

Solution: (-124) x(-1) =124

6) (-12) x (-7)

Solution: (-12) x (-7) =84

7) (-63) x 7

Solution: (-63) x 7= -(63 x 7) = -441

8) 7 x (-15)

Solution: 7 x (-15) = -(7 x 15) = -105

Sample Problems Integers Haryana Board Class 7

17. Write the pair of integers whose product will be

1) A negative integer

Solution: (-4) x 6 =-(4 x 6) = -24

3 x (-7) = -( 3 x 7)=- 21

2) A positive integer

Solution: 5 x 6 = 30 (-4) x (-5) = (4 x 5) = 20

3) Zero

Solution: 0 x 5 = 0 ; 3  x 0= 0

18. During the summer, the level of water in a pond decreases by 5 inches every week due to evaporation. What is the change in the level of the water over a period of 6 weeks?

Solution:

The level of water that decreases ever week = 5 inches

The level of water that decreases 6 weeks = 6 x 5 inches

= 30 inches

The change in the level of the water over a period of 6 weeks

= The water in a pond decreases by 30 inches

= -30 inches

19. A green grocer earns a profit of 7 per kg on tomato and got loss of per kg on brinjal by selling. On Monday he got neither profit nor loss, if he sold 68 kgs of tomato. How many kgs of brinjal did he sell?

Solution:

Profit on tomato per1 kg that a green grocer earns = 7

Loss on brinjal per kg = 4

Number of kgs of tomato he sold on Monday = 68 kg

Let number of kgs of brinjal he sold on Monday = x kg

On Monday he gets neither profit nor loss

68 x 7 = x × 4

x = \( \frac{68 \times 7}{4} \)

x = 119

Number of kgs of brinjal he sold on Monday = 119 kgs

20. In a test, + 3 marks are given for every correct answer and-1 mark is given for every incorrect answer. Sona attempted all the questions and scored + 20 marks though she got 10 correct answers.

1) How many incorrect answers she attempted?

2) How many questions were given in the test?

Solution:

In a test,

Marks given for every correct answer = + 3

Marks given for each incorrect answer =-1

Marks scored by Sona =20

Number of correct answers she got =10

Marks scored for 10 correct answers

= 10 x (+3) = + 30

The difference of marks

= 20 -30 = -10

1) The number of incorrect answers she attempted = -10/-1 =10

Thus Sona attempted 10 incorrect answers.

2) Number of questions were given in the test

= Number of correct answered questions + Number ofincorrect answered questions.

= 10 + 10 = 20

Word problems on integers for Class 7 HBSE

21. Verify -3 x [ (-4)- 2] = [(-3) x (-4)]– [(-3)x 2].Is multiplication distributive over subtraction of integers? Write your observations.

Solution: LHS = (-3) x [(-4) -2]

= (-3) x (-6)

= 18

RHS = [(-3)x(-4)]-[(-3)x2]

= (3×4) -[-(3×2)]

= 12 -(-6)

= 12 + 6 = 18

Observation: LHS = RHS

Yes, multiplication is distributive over subtraction of integers.

22. Identify the laws in the following statements:

1) -3 + 5 = 5 + (-3)

Solution: Commutative property for addition.

2) -2 x1 =1 x (-2) = -2

Solution: Multiplicative Identity.

3) [(-5) x 2)] x 3 = (-5) x [(2 x 3)]

Solution: Associative property for multiplication.

4) 18 x [7+ (-3)] = [18 x 7] + [18 x (-3)]

Solution: Distributive over addition and multiplication.

5) -5×6 = -30

Solution: Closure property for multiplication

6) -3 + 0 = 0 + (-3) = -3

Solution: Additive Identity

23. Simplify the following using suitable laws.

1) -11 x (-25) x (-4)

Solution:

= -11 x[+(25×4)]      ( Closureproperty)

= -11×100

= -(11×100)

= -1100

2) 3 x (-18) +3 x (-32)

Solution:

= 3x[-18(-32)]        ( Distributive property)

= 3x[-18-32]

= 3x[-50]

= -150

24. Sankar, a fruit vendor sells 100kg of oranges and 75 kg of pomegranates. If he makes a profit of 11 per one kg of pomegranates and loss of 8 per one kg oranges, what will be his overall profit or loss?

Solution: Profiton pomegranates per one kg= 11

Loss on oranges per one kg = 8

Number of kgs of pomegranates sold = 75

Number of kgs of oranges sold = 100

The amount of profiton pomegranates = 75×11 =825

The amount of loss on oranges = 100 x 8 = 800

As the profit is more than loss he got profit.

Profit = 825- 800

= 25.

25. Solve the following.

1) 17 -(-14)

2) 13 -(-8)

3) 19 -(-5)

4) 15-28

5) 25-33

6) 80 -(-50)

7) 150-25

8) 32 -(-18)

Solution:

1) 17 -(-14) =17 + 14 = 31

2) 13 – (-8) = 13 + 8- 21

3) 19- (-5) = 19 + 5 = 24

4) 15 -28 = -13

5) 25 – 33 = -8

6) 80 -(-50) = 80 + 50 = 130

7) 150 – 75 = 75

8) 32 -(-18) =32 + 18 = 50

Properties of integers Class 7 HBSE Maths

26. A merchant on selling rice earns a profit of 10 per bag of basmati rice sold and a loss of 5 per bag of non-basmati rice.

1) He sells 3, 000 bags of basmati rice and 5,000 bags of non-basmati rice in a month. What is his profit or loss in a month?

2) What is the number of basmati rice bags he must sell to have neither profit nor loss,if the number of bags of non-basmati rice sold is 6,400 ?

Solution:

Profit that the merchant gets per bag of Basmati rice = 10.

Loss that the merchant gets per bag ofnon basmati rice = 5.

1) No. of Basmati rice bags he sold = 3,000

No. of Non-Basmati rice bags he sold = 5,000

∴ The total result = 3,000 x 10- 5,000 x 5

= 30,000- 25,000

= 5,000

Since the result is positive, the merchant get a profit of 5,000

2) No. of non-basmati rice bags sold = 6400

Let the No. of basmati rice bags sold = x

The no.of basmati rice bags he must sell to have neither profit nor loss,

= x × 10 = 6400 x 5

x = 6400×5/10

x = 3200

∴ Merchant has to sell 3200 basmati rice bags to have neither profit nor loss.

27. Find the product, using suitable properties.

1) 26 x (-48) + (-48) x (-36)

2) 8 x 53 x (-125)

3) 15 x (-25) x (-4) x (-10)

4) (-41) x102

5) 625 x (-35) + (-625) x 65

6) 7 x (50- 2)

7) (-17) x (-29)

8) (-57) x (-19) +57

Solution:

1) 26 x (-48) + (-48) x (-36)

= [26 + (-36)] x (-48) [Multiplicative Distributive property]

= (-10) x (-48)

= 480 [Product of two negatives is positive]

∴ 26 x (-48) + (-48) x (-36) =480

2) 8 x 53 x (-125) = [8 x 53] x (-125) [Associative property]

= 424 x (-125)

= -53000

Hint: To multiply 424 x 125, simply write 424 x 1000/8 which is easier

2) 15 x (-25) x (-4) x (-10) = 15 x [(-25) x (-4)] x -10 [Associative property]

= 15 x 100 x -10

= [15 x 100] x -10 [Associative property]

= 1500 x -10

= -15000

4) (-41) x 102 = (-41) x [100 + 2]

= (-41) x 100 + (-41) x 2 [Distributive property]

= -4100 + (-82)

= -4182

5) 625 x (-35) + (-625) x 65 = (-625) x 35 + (-625) x 65

= (-625) x (35 + 65) [Distributive property]

= -625×100

= -62500

6) 7 x (50 -2) = 7 x [50 + (-2)](Distributive property)

= 7 x 50 + 7 x (-2)

= 350 – 14 = 336

7) (-17) x (-29) = (-17) [-30 + 1]

= (-17) (-30) + (-17) (1) (Distributive property)

= 510-17 = 493

8) (-57) x (-19) +57 = (-57) x (-19) +(-57) x (-1)

= (-57) x [(-19) + (-1)] (Distributive property)

= (-57) x -20

= 1140

28. In a class test containing 15 questions, 4 marks are given for every correct answer and (-2) marks are given for every incorrect answer.

(1) Bharathi attempts all questions but only 9 answers are correct. What is her total score?

(2) One of her friends Hema answers only 5 questions correctly. What will be her total score?

Solution:

No. of questions in the test = 5

Marks given for correct answer = 4

Marks given for wrong answer = (-2)

1) No. of correct answers answered by Bharathi = 9

No. of incorrect answers =15-9 = 6 [ She attempted all questions]

Thus, the total score of Bharathi = 9×4 + 6 (-2)

= 36- 12 = 24 marks.

∴ The total score of Bharathi is 24 marks.

2) No. of correct answers answered by Hema = 5

No. of incorrect answers answered by Hema = 0

∴ Total score of Hema = 5 x 4 = 20 marks.

 

Fill in the blanks:

77. -4 + 6…………. 7 -1 (Use < or >)

Answer: <

78. If die number of negative integers in a product is……….then the product is a positive integer.

Answer: even

79. The product ofa negative integer and zero is………

Answer: zero

80. For any integer a, we have a + 0 is……..

Answer: not defined

81. -87 +……. = 87

Answer: -1

82.(Positive integer) x (Negative integer) =……..

Answer: negative integer

83. …………is not commutative for integers.

Answer: Subtraction

84. 7, 3, -1,-5,?………

Answer: -9

85. 17+ …… = 0

Answer: -17

86………….in his book Ankitung Zur Algebra (1770), was one of the first mathematicians to attempt to prove (-1) x (-1) =1

Answer: Euler

Match the following:

87.

1. For any two integers a and b,a + b is an integer                                  (   )             A) Associative property under addition

2. For any integers a, b and c a + (b + c) = (a + b) + c                            (   )              B) Multiplicative identity

3. For any integers a and b; a xb = b x a                                                   (   )              C) Distributive property

4. For any integers a, b and c a x (b + c) = a xb + a xc                            (   )              D) Closure under addition

5. For any integer a;1 x a = a x 1= a. The integer1 is called                    (   )              E) Commutative property for multiplication

Answer: 1. D 2. A 3. E 4. C 5. B

 

Haryana Board Class 7 Maths Solutions For Chapter 4 Simple Equations

Haryana Board Class 7 Maths Solutions For Chapter 4 Simple Equations

  • Equation: An open sentence containing the sign of equality is called an “equation. “
  • Linear equation: An equation involving one variable with highest power T, is called a “Linear equation” or ‘Simple equation’.
  • Solution: The value which when substituted for the variable in an equation makes LHS = RHS is called ti “solution” or “root of the given equation”.
  • Properties of equality:
    1. Reflexive property: Every number is equal to itself.
      Eg: 6 = 6, x = x etc.
    2. Symmetric property: If a, b are two numbers and a = b, then b = a
      Eg: (6 + 3) = (8 + 1) hence (8+1) = (6+3)
    3. Transitive property: Ifa = b and b = c then a = c for any three numbers a, b, c.
      Eg: 10 + 5 = 15, 15 = 3×5
      10 + 5 = 3 x 5
      If x = a and x = b, then a = b, for any three numbers x, a, b.
      Eg: 12 = 8 + 4, 12 = 7 + 5 .-.8 + 4 = 7 + 5
    4. Addition Property: If a = b, then a + c = b + c where a, b, c are any three numbers.
    5. Subtraction Property: If a = b, then a-c =b-c where a, b, c are any three numbers.
    6. Multiplicative Property: Ifa = b, then ab = be, where a, b, c are any three numbers.
    7. Division Property:Ifa = b, then = \( \frac{a}{c}=\frac{b}{c} \) where a,b,c are any three numbers and c=0.
  • We can add the same number to both sides ofan equation.
  • We can subtract the same number from both sides of an equation.
  • We can multiply both sides of an equation by the same non-zero number.
  • We can divide both sides of an equation by the same non-zero number.
  • In transforming terms from L.H.S. to R.H.S.
    ‘+ quantity’ becomes ‘- quantity’
    ‘- quantity’ becomes ‘+ quantity’
    ‘x quantity’ becomes ‘÷quantity’
    ‘÷ quantity’ becomes ‘x quantity’
  • Transposition: We can drop any term from one side of an equation and take it to the otherside with its sign changed. This process is called “Transposition”.
  • Simple equation: An equation in one variable with highest power 1 is known as Simple equation.
    Eg: y – 7 = 11; 2m- 5 = m + 7
  • Bhaskara – II: Bhaskara-II is a famous mathematician of an ancient India. He wrote ‘Siddhanta Shiromani’.
  • Variables: The word variable means something that can vary i e. change. A vanable takes on different numerical values; its value is not fined Variables ore denoted usually by letters of the alphabets such as x, y, z, l, m, n, p. etc.
  • Expressions: Expressions ore formed by performing operations like addition, subtraction, multiplication and division on the variables.
    Example: 4x + 5
  • Equations: An equation is a kind of condition on a vanable. An expression equated to some value is called an equation.
    Example: 4i + 5 – 9
  • Value of an expression:
    The value of an expression depends upon the value of the variable from which the expression is formed.
    Example: 4x + 5
    At x-0; 4×0+ 5= 0 + 5=5
    At x=l; 4×1+ 5=4 + 5=9
    At x=2; 4×2+ 5=8 + 5=13
    At x=5; 4×5+ 5=20 + 5=25

Haryana Board Class 7 Maths Solutions For Chapter 4 Exercise – 4.1 :

1. Complete the last column of the table

Complete the last column of the table

Haryana Board Class 7 Maths Simple Equations solutions

2. Check whether the value given in the brackets is a solution to the given equation or not:

1) n + 5 = 19 (n = 1)

Solution: 11 + 5 = 19

n =1;

LHS =1+5=6

RHS =19

LHS ≠ RHS

n =1 is not a solution.

HBSE Class 7 Simple Equations Solutions

2) 7n + 5 = 19 (n = -2)

Solution: 7H + 5 = 19

n= -2;

LHS = 7n + 5

= 7 (-2) + 5 = -14 + 5 = -9

RHS = 19

LHS ≠ RHS

n = – 2 is not a solution.

3) 7n + 5 =19 (n=2)

Solution: 7n + 5 = 19

n = 2

LHS = 7n + 5

= 7 x 2 + 5 = 14 + 5-19

RHS = 19

LHS = RHS

n = 2 is a solution

How to solve linear equations Class 7 HBSE

4) 4p – 3 = 13 (p = 1)

Solution:

4p-3 = 13

p-1 ;

LHS = 4p -3

=4 x 1- 3 = 4 – 3 = 1

RHS = 13

LHS ≠ RHS

p =1 is not a solution.

5) 4p – 3 = 13 (p = – 4)

Solution:

4p – 3=13

p = -4;

LHS = 4p – 3

4(- 4) – 3 = – 16 – 3 = -19

RHS = 13

LHS ≠ RHS

p = – 4 is not a solution.

6) 4p-3 = 13 (p = 0)

Solution:

4p – 3 = 13

p = 0;

LHS =4p-3

= 4(0) – 3 = 0- 3 = -3

RHS = 13

LHS ≠ RHS

p = 0 is not a solution

3. Solve the following equations by trial and error method:

1) 5p + 2 = 17

5p + 2 = 17

p = 3 is the solution.

2) 3m- 14 = 4

3m - 14 = 4

Solution:

m= 6 is the solution.

Key Questions in Simple Equations for Class 7 HBSE

4. Write equations for the following statements:

1) The sum of numbers x and 4 is 9.

Solution: 1 + 4-9

2) 2 subtracted from y is 8.

Solution: y- 2 = 8

3) Ten times a is 70.

Solution: 10a = 70

Transposition method in algebra Class 7 Haryana Board

4) The number h divided by 5 gives 6.

Solution: \( \frac{b}{5} \)

5) Three-fourth of t is 15

Solution: \( \frac{3}{4} t=15 \)

6) Seven times m plus 7 gets you 77.

Solution: 7m + 7 = 77

7) One – fourth of a number ‘x’ minus 4 gives 4.

Solution: \( \frac{1}{4} x-4=4 \)

8) If you take away 6 from 6 times y, you get 60.

Solution: 6y- 6 60

9) If you add 3 to one – third of z, you get 30.

Solution: \( \frac{1}{3} z+3=30 \)

5. Write the following equations in statement forms :

1) p + 4 = 15

Solution: Sum of p and 4 is 15.

2) m – 7- 3

Solution: The difference between m and 7 is 3.

3) 2m = 7

Solution: Two times m is 7

4) \( \frac{m}{5}=3 \)

Solution: One-fifth of m is 3

5. \( \frac{3 m}{5}=6 \)

Solution: Three-fifth of m is 6.

6) 3p + 4 = 25

Solution:

If we add 4 to three times of p we get 25.

Sample Problems Simple Equations Haryana Board Class 7

7) 4p – 2 =18

Solution: 2 subtracted from four times of p gives 18

8) \( \frac{p}{2}+2=8 \)

Solution: Adding 2 to half of p gives 8

Haryana Board Class 7 Maths Solutions For Chapter 4

1) Irfan says that he has 7 marbles more than five times the marbles Parmit has. Irfan has 37 marbles. (Take m to be the number of Parmit’s marbles.)

Solution:

Let Parmit have ’m’ marbles

Five times of m is 5m

7 more than five times of m – 5m + 7

i.e. Irfan has 5m + 7 marbles.

But it is given that Irfan has 37 marbles

5m + 7 = 37 is the requirrd equation.

2) Laxmi’s father is 49 years old. He is 4 years older than three times Laxmi’s age. (Take Laxmi’s age to be ‘y’ years.)

Solution:

Let the age of Laxmi be ‘y’ years

Three times Laxmi’s age = 3y yearss

Laxmi’s father is 4 years older than three times Laxmi’s age.

Age of Lnxmi’s father = 3y + 4 years.

It is given that Laxmi’s father is 40 years old.

3y + 4 = 49 is the required equation.

Solving simple equations step by step Class 7 HBSE

3) The teacher tells the class that the highest marks obtained by a student in her class is twice the lowest marks plus 7. The highest score is 87. (Take the lowest score to be T.)

Solution:

Let the lowest score be l.

Twice the lowest marks = 2l.

Highest score obtained by a student in her class is twice the lowestmarks plus 7.

i.e., Highest score =2l+ 7

Given that the highest score is 87.

2l+ 7 = 87 is the required equation.

4) In an isosceles triangle, the vertex angle is twice either base angle. (Let the base angle be ‘b’in degrees. Remember that the sum of angles of a triangle is 180 degrees.)

Solution:

Let the base angle be ‘b’ degrees

Vertical angle = Twice the base angle = 2b degrees

Sum of the angles of triangle = 180° b + b + 2b = 180°

4b = 180° or b = 45° is the required equation.

Haryana Board Class 7 Maths Solutions For Chapter 4

1) When you multiply anumber by 6 and subtract 5 from the product you get 7. Can you tell what the number is?

Solution:

Let the number be x

Multiplying by 6it becomes 6x

Subtract 5 from the product it becomes 6x-5

Given its value is 7

6x- 5 = 7

Transposing- 5 from LHS to RHS

6x = 7 + 5

6x = 12

(or)

\( x=\frac{12}{6}=2 \)

The required numberis 2.

2) What is that number one-thirdofwhich added to 5 gives 8?

Solution:

Let the number be x

One – third of that number is \( \frac{1}{3} \times x=\frac{x}{3} \)

Add 5 to it

\( \frac{x}{3}+5 \)

Given its value is 8

\( \frac{x}{3}+5=8 \)

Transposing 5 from LHS to RHS

\( \begin{aligned}
\frac{x}{3} & =8-5 \\
& \Rightarrow \frac{x}{3}=3
\end{aligned} \)

Multiplying both sides by 3 we get

\( \frac{x}{3} \times 3=3 \times 3 \text { or } \mathrm{x}=9 \)

The required number is 9.

Haryana Board Class 7 Maths Solutions For Chapter 4

1. There are two types of boxes containing mangoes. Each box of the larger type contains 4 more mangoes than the number of mangoes contained in 8 boxes of the smaller type. Each larger box contains 100 mangoes. Find the number of mangoes contained in the smaller box.

Solution:

Let the number of mangoesin the small box be x.

Number of mangoesin larger type box are 4 more than the number of mangoes contained in 8 boxes of the smaller type.

Equation is 8x + 4 = 100

Transposing 4 froms LHS to RHS

8x = 100 – 4

8x = 96

Dividing both sides by 8 we get

\( \frac{8 x}{8}=\frac{96}{8} \Rightarrow 64 x=768 \) \( x=\frac{768}{64} \)

x= 12

The number of mangoes contained in the smaller box are 12

Solving Equations Class 7 Haryana Board

Haryana Board Class 7 Maths Solutions For Chapter 4 Exercise-4.3

1. Setup equations and solve them to find the unknown numbers in the following cases:

(1) Add 4 to eight times a number; you get 60.

Solution: Let the number be x

Eight times the number is 8x

Add 4 to this product it becomes 8x + 4

Given its value is 60

8x + 4 = 60

Transposing 4 from LHS to RHS

8x = 60- 4

=> 8x = 56

Dividing both sides by 8 we get

\( \begin{aligned}
& \frac{8 x}{8}=\frac{56}{8} \Rightarrow 64 x=448 \\
& x=\frac{448}{64}
\end{aligned} \)

⇒ x = 7

The required number is 7.

2) One-fifth of a number minus 4 gives 3.

Solution:

Let the number be x

One -fifth of this number is \( x \times \frac{1}{5}=\frac{x}{5} \)

Subtracting 4 from it we get

\( \frac{x}{5}-4 \)

Given its value is 3

\( \frac{x}{5}-4=3 \)

Transposing – 4 from LHS to RHS

\( \begin{aligned}
& \frac{x}{5}=3+4 \\
& \Rightarrow \frac{x}{5}=7
\end{aligned} \)

Multiplying both sides by 5 we get

\( \frac{x}{5} \times 5=7 \times 5 \)

=> x = 35

The required number is 35.

3) If I take three-fourths of a number and add 3 to it,I get 21.

Solution:

Let the number be x

Three-fourth of this number is

\( x \times \frac{3}{4}=\frac{3 x}{4} \)

Adding 3 to this value we get

\( \frac{3 x}{4}+3 \)

Given its value is 21

\( \frac{3 x}{4}+3=21 \)

Transposing 3 from LHS to RHS

\( \begin{aligned}
& \frac{3 x}{4}=21-3 \\
& \Rightarrow \frac{3 x}{4}=18
\end{aligned} \)

Multiplying both sides by 4 we get

\( \begin{aligned}
& \frac{3 x}{4} \times 4=18 \times 4 \\
& \Rightarrow 3 x=72
\end{aligned} \)

Dividing both sides by 3 we get

\( \begin{aligned}
& \frac{3 x}{4}=\frac{72}{3} \Rightarrow 9 x=216 \\
& \Rightarrow x=24
\end{aligned} \)

=> x = 24

The required number is 24.

4) When I subtracted 11 from twice a number, the result was 15.

Solution:

Let the number be x

Twice of that number is 2x

Subtracting 11 from this product, we get 2x-11

Given its value is 15

2x-11 = 15

Transposing – 11 from LHS to RHS

2x = 15 + 11

=> 2x = 26

Dividing both sides by 2 we get

\( \begin{aligned}
& \frac{2 x}{2}=\frac{26}{2} \Rightarrow 4 x=52 \\
& x=\frac{52}{4}
\end{aligned} \)

=>x= 13

The required number is 13

5) Munna subtracts thrice the number of notebooks he has from 50, he finds the result to be 8.

Solution:

Let the number be x

Thrice of the number is 3x

On subtracting thrice the number of notebooks from 50, it becomes 50 – 3x

The result is 8

50 -3x = 8

On transposing 50 from LHS to RHS

– 3x = 8 – 50

– 3x =- 42

Dividing both sides by -3 we get

\( \begin{aligned}
& \frac{-3 x}{-3}=\frac{-42}{-3} \Rightarrow 9 x=126 \\
& \Rightarrow x=\frac{126}{9} \Rightarrow x=14
\end{aligned} \)

The required number is 14.

HBSE Class 7 Maths Chapter 4 Guide

6) Ibenhal thinks a number. If she adds 19 to it and divides the sum by 5 she will get 8.

Solution:

Let the number be x

Adding 19 to it, we get x + 19

Dividing the sum by 5 we get \( \frac{x+19}{5} \)

The result is 8

\( \frac{x+19}{5}=8 \)

Multiplying both sides by 5 we get

\( \frac{x+19}{5} \times 5=8 . \times 5 \)

=> x+ 19 = 40

Transposing 19 from LHS to RHS

x= 40 – 19

x= 21

The required number is 21

7) Anwar thinks of a number. If he takes away 7 from \( \frac{5}{2} \) of the number, the result is 23.

Solution:

Let the number be x

\( \frac{5}{2} \text { of the number is } \frac{5 x}{2} \)

Take away 7 from it we get

\( \frac{5 x}{2}-7 \)

The result is 23

\( \frac{5 x}{2}-7=23 \)

Transposing -7 from LHS to RHS

\( \frac{5 x}{2}=23+7 \Rightarrow \frac{5 x}{2}=30 \)

Multiplying both sides by 2 we get

\( \frac{5 x}{2} \times 2=30 \times 2 \Rightarrow 5 x=60 \)

Dividing both sides by 5 we get

\( \frac{5 x}{2}=\frac{60}{5} \Rightarrow 25 x=300 \)

=> x = 12

The required number is 12

2. Solve the following:

1) The teacher tells the class that the highest marks obtained by a student in her class is twice the lowest marks plus 7. The highest score is 87. What is the lowest score?

Solution: Let the lowest score be x

Twice the lowest marks plus 7 is 87

2x + 7 = 87

Transposing 7 from LHS to RHS

2x = 87-7

=> 2x = 80

Dividing both sides by 2 we get

\( \begin{aligned}
& \frac{2 x}{2}=\frac{80}{2} \Rightarrow 4 x=160 \\
& \Rightarrow x=40
\end{aligned} \)

The lowest score in the class is 40.

2) In an isosceles triangle, the base angles are equal. The vertex angle is 40°. What are the base angles of the triangle? (Remember, the sum of three angles of a triangle is 180°)

Solution:

ABC is an isosceles triangle.

The base angles are equal.

Let one of its measure be x.

Given: Vertex angle is 40°

ABC is an isosceles triangle

Sum of the three angles of a triangle is 180°.

i.e. ∠A+∠B+∠C = 180°

=> 40° + x + x = 180°

=> 40° + 2.V = 180°

Transposing 40° from LHS to RHS we get

2x = 180° – 40°

=> 2x = 140°

Dividing both sides by 2 we get

\( \begin{aligned}
&\frac{2 x}{2}=\frac{140^{\circ}}{2} \Rightarrow 4 x=280^{\circ}\\
&\Rightarrow x=\frac{280^{\circ}}{4}
\end{aligned} \)

=> x= 70°

The base angle is 70° each

Important Concepts Simple Equations Class 7 HBSE

3) Sachin scored twice as many runs as Rahul. Together, their runs fell two short of a double century. How many runs did each one score?

Solution: Let runs scored by Rahul be x

Then runs scored by Sachin = 2x

The sum of their runs is two short of a double century.

x + 2.x + 2 = 200

=> 3x + 2 = 200

Transposing 2 froms LHS to RHS

3x = 200 -2

=> 3x = 198

Dividing both sides by 3 we get

\( \begin{aligned}
& \frac{3 x}{3}=\frac{198}{3} \Rightarrow 9 x=594 \\
& \Rightarrow x=\frac{594}{9}
\end{aligned} \)

=> X = 66

Runs scored by Rahul = 66

Runs scored by Sachin = 132

3. Solve the following:

1) Irfan says that he has 7 marbles more than five times the marbles Parmithas. Irfan has 37 marbles. How many marbles does Parmit have?

Solution:

Let the number of marbles with Parmit be

If five times the number is added to 7 we get 37 marbles.

5x + 7 = 37

Transposing 7 from LHS to RHS

5x = 37-7 => 5x = 30

Dividing both sides by 5 we get

\( \begin{aligned}
& \frac{5 x}{5}=\frac{30}{5} \Rightarrow 25 x=150 \\
& \Rightarrow x=\frac{150}{25} \\
& \Rightarrow x=6
\end{aligned} \)

Parmit has 6 marbles.

2) Laxmi’s father is 49 years old. He is 4 years older than three times Laxmi’s age. What is Laxmi’s age?

Solution:

Let Laxmi’s age be x years

Laxmi’s father is 49 years old and is 4 years older than three times Laxmi’s age.

3x + 4 = 49

Transposing 4 from LHS to RHS

3x = 49- 4

=> 3x = 45

Dividing both sides by 3 we get

\( \begin{aligned}
& \frac{3 x}{3}=\frac{45}{3} \Rightarrow 9 x=135 \\
& \Rightarrow x=\frac{135}{9} \Rightarrow \mathrm{x}=15
\end{aligned} \)

Laxmi’s age is 15 years.

Word problems on simple equations Class 7 HBSE

3) People of Sundargram planted trees in the village garden. Some of the trees were fruit trees. The number of non fruit trees were two more than three times the number of fruit trees. What was the number of fruit trees planted if the number of non – fruit trees planted was 77?

Solution:

Let the number of fruit trees planted be x

Number of non – fruit trees = 3x + 2

Given non – fruit trees = 77

3x + 2 = 77

Transposing 2 from LHS to RHS

3x = 77- 2

=> 3x = 75

Dividing both sides by 3 we get

\( \begin{aligned}
& \frac{3 x}{3}=\frac{75}{3} \Rightarrow 9 x=225 \\
& x=\frac{225}{9}
\end{aligned} \)

x = 25

The number of fruit trees planted are 25.

4. Solve the following riddle:

I am a number,

Tell my identity!

Take me seven times over

And add a fifty !

To reach a triple century

You still need forty !

Solution:

Let the number be x

Seven times of this number is 7 x x = 7x

On adding 50 it becomes 7x + 50

To reach a triple century we need forty

7x + 50 + 40 = 300

=> 7x + 90- 300

Transposing 90 bom LHS to RHS

7x-300-90

=> 7x- 210

Dividing both sides by 7 we get

\( \begin{aligned}
& \frac{7 x}{7}=\frac{210}{7} \Rightarrow 49 x=1470 \\
& x=\frac{1470}{49} \Rightarrow x=30
\end{aligned} \)

The required number is 30.

Haryana Board Class 7 Maths Chapter 4 Very Short Answer Questions

1. What is meant by simple equation?

Solution: An equation is a condition on a variable such that two expressionsin the variable should have equal value.

2. What is meant by solution of an equation ?

Solution:

The value of the variable for which the equation is satisfied is called the solution of the equation.

3. Find the value of the expression 4x + 5 at x = 1, 2, 3

Solution:

Given expression is 4x + 5

At x =1 its value is4(l) + 5- 4 + 5- 9

x = 2 its value is 4 (2) + 5 – 8 + 5 – 13

x- 3 its value is 4 (3) + 5 = 12 + 5-17

4. “If you subtract 5 from 6 times a number you get 7” write this statement in the form of an equation.

Solution:

Let the number be x; x multiplied by 6 is 6x. Subtracting 5 from 6x, we get 6x-5.

The obtained result is 7.

6x – 5- 7

5. Solve 4 (m + 3) – 18

Solution:

4(m + 3)-18

Divide both sides by 4

m + 3 = \( \frac{18}{4} \)

=> m + 3 = \( \frac{9}{2} \)

m = \( \frac{9}{2} \) – 3

\( m=\frac{9}{2}-\frac{6}{2}=\frac{3}{2} \)

Practice Problems Simple Equations Class 7 Haryana Board

6. The sum of three times a number and 11 is 32. Find the number.

Solution:

Let the number be x

3x + 11 = 32

3x = 32 – 11

3x- 21

x = \( \frac{21}{3} \) = 7

The required number is 7.

7. The sum of twice a number and 4 is 80, find the number.

Solution:

Let the number be ‘x’

Twice a number = 2x

The sum of twice a number and 4 is 80

=> 2x + 4 = 80

=> 2x = 80 – 4

=> 2x – 76

x= \( \frac{76}{2} \)

=> x = 38

The number = 38

8. Find the value of x from the adjacent diagram.

Solution: From the figure

Adjacent diagram

12 + x + 5 – 24

17 + x – 24

x – 24 – 17

x – 7 cm

9. To convert temperature from Fahrenheit to Centigrade, we use the formula (F-32) = \( \frac{9}{5} \) x C.If C = -40° C then find F.

Solution:

F-32 = \( \frac{9}{5} \) x C

If C = – 40°C then F – 32 = \( \frac{9}{5} \) x -40

F – 32 = 9 x 8

F – 32 = -72

F = -72+32

F = -40°

10. Write the equations of the following mathematical statements.

1) A number A decreased by 5 is 14.

Solution: A – 5 = 14

2) Eight times of y plus 3 is -5.

Solution: Eight times of

y = 8 X y = 8y

Eight times of y plus 3 is – 5

8y + 3 = -5

3) If you add one fourth of z to 3 you get 7.

Solution:

One fourth of z = \( \frac{z}{4} \)

If we add one fourth of z to \( 3=\frac{z}{4}+3 \)

\( \frac{z}{4}+3=7 \)

4) If you take away 5 from 3 times of m, you get 11.

Solution:

3 times of m = 3 x m = 3m

If we take away 5 from 3 times of m

= 3m – 5

3m- 5 = 11

5) Sum of angles 2x (x – 30) is a right angle.

Solution: The sum of angles 2x (x – 30)

= 2x + x – 30

= 3x – 30

Right angle = 90°

3x -30 = 90″

3x = 120°

6) The perimeter of a square of side ‘a’ is 14 m

Solution:

The side of a square is ‘a’ m

Perimeter of a square = 4 x side

= 4 x a = 4a m.

The result is => 4a = 14 m.

11. Write the following equations in statement form.

1) m – 5 =12

Solution: ‘m’ is decreased by 5 is 12.

2) \( \frac{a}{3}=4 \)

Solution: One third of number ‘a’ is 4.

3) 4x + 7 = 15

Solution: 4 times of ‘x’ and 7 is 15

4) 2 – 3y = 11

Solution: 2 is decreased bv 3 time ‘y’ is 11

12. Write any three equivalent equations having the solution a = -5

Solution:

Write any three equivalent equations having the solution a = -5

Here a + 7 = 2; a-10 = -15 and 3a + 5 = -10 are equivalent equations because they have the same solution a = – 5

13. The difference between two numbers is 7. Six times the smallerplus the larger is 77. Find the numbers.

Solution:

Let the smaller number be ‘x’

The differencebetween two numbers =7

Larger number = x +7

Six times of smaller plus the largeris 77

=> 6x + x + 7 = 77

=> 7x +7 = 77

=> 7x = 77 -7

=> 7x = 70

\( \frac{70}{7} \)

=>x = 10

The smaller number = 10

Larger number = x + 7

= 10 + 7 = 17

14. In a class of 48 students, the number of girls is one third the number of boys. Find the number of girls and boys in the class.

Solution:

Let the number of boys be ‘x

then number of girls = \( \frac{x}{3} \)

Total students in the class is 48

\( \begin{aligned}
& \Rightarrow \quad x+\frac{x}{3}=48 \\
& \Rightarrow \quad \frac{3 x+x}{3}=48 \\
& \Rightarrow \quad \frac{4 x}{3}=48 \\
& \Rightarrow 4 x=48 \times 3 \\
& \Rightarrow 4 x=144 \\
& \Rightarrow x=\frac{144}{4} \\
& \Rightarrow x=36
\end{aligned} \)

Number of boys = 36

\( \begin{aligned}
\text { Number of girls } & =\frac{x}{3}=\frac{36}{3} \\
& =12
\end{aligned} \)

15. A sum of 500 is in the form of notes of denominations of 5 and 10. If the total number of notes is 90 then find the number of notes of each type.

Solution:

Total number of notes

Let the 5 notes be x then the 10 notes = 90 -x

5 denomination amount = 5x

10 denomination amount = 10 (90-x)

= 900 – 10x

Total amount = 500

=> 5x + 900 – 10 x =500

=> 900 – 5x = 500

=>-5x = 500-900

=>-5x = -400

=> 5x= 400

=> \( x=\frac{400}{5} \)

=> x = 80

Number of notes of 5 = 80

Number of notes of 10 = 90 – x

= 90 – 80 = 10

16. Suhana said, “multiplying my number by 5 and adding 8 to it gives the same answer as subtracting my number from 20. Find Suhana’s number.

Solution:

Let Suhana’s number be ‘x’

From the problem, we have

5x x + 8 = 20-x

5x + 8 = 20 – x

By transposing variables and constants,

5x + x = 20- 8

6x= 12

\( x=\frac{12}{6} \)

[Transposing 6 to R.H.S]

Suhana’s number is 2

Haryana Board Class 7 Maths Chapter 4 Long Answer Questions

17. Check whether the value given in the brackets is a solution to the given equation or not.

1) 5n – 7 = 23 (n = 6)

Solution:

When n = 6

LHS = 5n -7

= 5(6) – 7 = 30- 7

= 23

RHS = 23

LHS = RHS

So n = 6 is a solution.

2) \( \frac{p}{4}-7 \) = 5 (p = 8)

Solution: When p = 8

LHS = \( \frac{p}{4}-7 \) = \( \frac{8}{4} \) -7

= 2- 7 = -5

RHS = 5

LHS = RHS

So ‘p’ = 8 is not a solution

3) 5 – 2x = 19 (x = -7)

Solution: When x = -7

LHS = 5 – 2x = 5 – 2(-7)

= 5 + 14 = 19

RHS = 19

LHS = RHS

So, x = -7 is a solution

4) 2 + 3 (m – 1) = 5 (m = -2)

Solution:

When m = -2

LHS = 2 + 3 (m – 1)

= 2 + 3 (-2- 1) = 2 + 3(-3)

= 2-9 = -7

RHS = 5

LHS = RHS

m = -2

So, ‘m’ = -2 is not a solution

Haryana Board Solutions For Class 7 Maths Chapter 5 Understanding Elementary Shapes

Haryana Board Solutions For Class 7 Maths Chapter 5 Understanding Elementary Shapes

  • A line segment is a part of line with two end points.
  • A triangle is made of three line segments.
  • A quadrilateral is made of four line segments.
  • It is possible to measure a line segment. The measure of each line segment is its ‘length’.
  • We can compare the length of two line segments by a) Simple observation, b) Tracing on a paper and comparing, c) Using instruments.
  • The instruments used to compare and draw line segments are ruler (Scale) and divider in the Geometry box.
  • The unit of measuring length is 1 centimeter (1 cm.) 1 cm = 10 mm.
  • Angle is formed between two rays or two line segments.
  • A protractor is a semi circular curved model with 180 equal divisions used to measure and construct angles.
  • The unit of measuring an angle is a degree (1°). It is \(\frac{1}{360}\)th part of one revolution.
  • Kinds of Angles:
  • An angle whose measure is 90° is called. “Right angle”.
  • An angle whose measure is 180° is called “Straight angle”. (= 180°)
  • An angle is ‘acute’ if its measure is smaller than that of a right angle. (< 90°)
  • An angle is ‘obtuse’ if its measure is more than that of a right angle and less than a straight angle. (>90° and < 180°)
  • A ‘reflex angle’ is more than a straight angle. (>180° and < 360°)…
  • If the angle measure is ‘O’ then it is called ‘Zero angle’.
  • If the measure of the angle is 360° then it is called ‘Complete angle’. (= 360°)
  • Angles where the ray moves in the opposite direction of the hands of a clock are called “Anti clock – wise angles”.
  • Angles where the ray moves in the direction of the hands of a clock are called “clock- wise angles”.
  • Two distinct lines of a plane which have a common point are “intersecting lines”.

Class 6 Maths Chapter 5 Understanding Elementary shapes Intersecting Lines

  • Two intersecting lines are perpendicular if the angle between them is a right angle.

Class 6 Maths Chapter 5 Understanding Elementary shapes Perpendicular

Haryana Board Class 7 Maths Understanding Elementary Shapes solutions

  • If two lines on a plane do not intersect each other at any point, they are called parallel lines.
    l and m are parallel lines.
    We write it as l || m and read it as l is parallel to m.
  • Two parallel lines do not have any common point.
    Symbols:
    || – Parallel
    ⊥ – Perpendicular
  • A ‘polygon’ is a closed figure made up of line segments.
  • A polygon with all equal sides and all equal angles is called a regular polygon.
  • Eg Equilateral triangle, Square & etc.
  • Plane: A plane is the two-dimensional flat surface that extends infinitely far.
    Eg: Wall, blackboard, paper. …….. etc.
  • In ‘polygons’ poly means ‘many’, gons means ‘sides’. Polygons means ‘many sides’.
  • A polygon is a closed figure in the plane with at least three line segments and typically four or more.
    Eg: Triangle, quadrilateral, pentagon, hexagon, heptagon, octagon, nanogon and decagon are some of the polygons.
  • The simple closed figure formed by three line segments is called a triangle. The line segments are called sides.
    A triangle contains three sides, three angles and three vertices. A triangle is denoted by the symbol “A”.

Class 6 Maths Chapter 5 Understanding Elementary shapes Triangle

  • Triangular regions: A triangle divides the points on the plane into three parts.

Class 6 Maths Chapter 5 Understanding Elementary shapes Triangular Regions

    1. Interior points of the triangle – D, E, J
    2. Points on the triangle – G, F, A, B, C
    3. Exterior points of the triangle – H, I, K

Quadrilateral: A polygon with four sides is called a quadrilateral. Quadra means four, lateral means ‘side’. In quadrilateral ABCD, four line segments

Class 6 Maths Chapter 5 Understanding Elementary shapes Quadrilateral

\(\overline{\mathrm{AB}}, \overline{\mathrm{BC}}, \overline{\mathrm{CD}} \text { and } \overline{\mathrm{DA}}\) are called sides and ∠A, ∠B, ∠C and ∠D are its four angles. Points A,B,C and D are its vertices. The line segments AC and BD are called its diagonals

  • 1 mm = 0.1 cm.
  • A triangle having all the three unequal sides is called a scalene triangle.
  • A triangle having two equal sides is called an isosceles triangle.
  • A triangle having three equal sides is called an equilateral triangle.
  • A cuboid has 6 faces, each face has 4 edges. Each face has 4 corners.
  • A polygon with 6 sides is called Hexagon.
  • A polygon with 8 sides is called Octagon. A parallelogram with 4 sides of equal length is called Rhombus.
  • A Rhombus with 4 right angles is called square..
  • Study the table given below and observe the names of various types of polygons.

Class 6 Maths Chapter 5 Understanding Elementary shapes Types of Polygons

Types of angles and triangles Class 7 HBSE Maths

Question 1. Take any post card. Use the above technique to measure its two adjacent. sides.

Solution. Post card length – 14 cm.

Breadth – 9.5 cm.

Yes, all post cards have the same dimensions.

Question 2. Select any three objects having a flat top. Measure all sides of the top using a divider and a ruler.

Solution.

Three objects:

Class 6 Maths Chapter 5 Understanding Elementary shapes Sharpner

Length = 2.7 cm.

Breadth = 2 cm.

Class 6 Maths Chapter 5 Understanding Elementary shapes Pencil

Length = 5.2cm.

Breadth = 1 cm.

3) Eraser

Class 6 Maths Chapter 5 Understanding Elementary shapes Eraser

Length = 2.4 cm.

Breadth = 1 cm.

Haryana Board Solutions For Class 7 Maths Chapter 5 Understanding Elementary Shapes Exercise 5.1

Question 1. What is the disadvantage in comparing line segments by mere observation?

Solution. Sometimes the difference in lengths between two line segments is not obvious with our eyes. So, we are not always sure about the judgement.

Question 2. Why is it better to use a divider than a ruler, while measuring the length of a line segment?

Solution. The errors are due to the thickness of the ruler and angular viewing by using a ruler. These errors are eradicated by using a divider. So, it is better to use a divider, while measuring the length of a line segment.

Question 3. Draw any line segment, say \(\overline{\mathrm{AB}}\). Take any point C lying in between A and B. Measure the lengths of AB, BC and AC. Is AB = AC+ CB?

(Note: If A, B, C are any three points on a line such that AC+ CB= AB, then we can be sure that C lies between A and B.) A

Solution.

Class 6 Maths Chapter 5 Understanding Elementary shapes Exercise 5.1 Question 3

Length of AB = 5 cm

Length of BC = 3 cm

Length of AC = 2 cm.

AC + BC = 2 cm + 3 cm = 5cm = AB

∴ AC + BC = AB

Question 4. If A, B, C are three points on a line such that AB = 5 cm, BC = 3 cm and AC = 8 cm, which one of them lies between the other two?

Class 6 Maths Chapter 5 Understanding Elementary shapes Exercise 5.1 Question 4

Solution. The point B lies between the point A and C.

Question 5. Verify, whether D is the mid point of \(\overline{\mathrm{AG}}\)

Solution.

Class 6 Maths Chapter 5 Understanding Elementary shapes Exercise 5.1 Question 5

Yes! D is the mid point of \(\overline{\mathrm{AG}}\). (A to G = 7)

Question 6. If B is the mid point of \(\overline{\mathrm{AC}}\) and C is the mid point of \(\overline{\mathrm{AC}}\), where A, B, C, D lie on a straight line, say why AB = CD? C D

Solution.

Class 6 Maths Chapter 5 Understanding Elementary shapes Exercise 5.1 Question 6

∴ B is the mid point of \(\overline{\mathrm{AC}}\)

∴ AB = BC. ……(1)

∴ C is the mid point of \(\overline{\mathrm{AC}}\)

∴ BC = CD …….. (2)

From (1) &(2) AB = BC = CD ⇒ AB = CD

Question 7. Draw five triangles and measure their sides. Check in each case, if the sum of the lengths of any two sides is always less than the third side.

Solution.

1)

Class 6 Maths Chapter 5 Understanding Elementary shapes Exercise 5.1 Question 7.1

AB + BC = 6 + 4 = 10 cm is not less than AC = 3 cm

2)

Class 6 Maths Chapter 5 Understanding Elementary shapes Exercise 5.1 Question 7.2

DF + DE = 5 + 7 = 12 cm is not less than EF = 4 cm.

3)

Class 6 Maths Chapter 5 Understanding Elementary shapes Exercise 5.1 Question 7.3

XY + ZY = 4 + 5 = 9 cm is not less than XZ = 3 cm

4)

Class 6 Maths Chapter 5 Understanding Elementary shapes Exercise 5.1 Question 7.4

AB + AC = 3 + 6 = 9 cm is not less than BC = 8 cm.

5)

Class 6 Maths Chapter 5 Understanding Elementary shapes Exercise 5.1 Question 7.5

PQ + PR = 8 + 17 = 25 cm is not less than QR = 15 cm. It is not possible to construct a triangle with one side more than the total of the other two sides.

Question 8. What is the angle name for half a revolution?

Solution. 180°.

Question 9. What is the angle name for one-fourth revolution?

Solution. 90°.

Question 10. Draw five other situations of one-fourth, half and three-fourth revolution on a clock.

Solution.

Class 6 Maths Chapter 5 Understanding Elementary shapes Exercise 5.1 Question 8

Haryana Board Solutions For Class 7 Maths Chapter 5 Understanding Elementary Shapes Exercise – 5.2

Question 1. What fraction of a clock wise revolution does the hour hand of a clock turn through, when it goes from

a) 3 to 9

b) 4 to 7

c) 7 to 10

d) 12 to 9

e) 1 to 10

f) 6 to 3

Solution. a) 1/2 of a revolution.

b) 1/4 of a revolution

c) 1/4 of a revolution

d) 3/4 of a revolution

e) 3/4 of a revolution

f) 3/4 of a revolution

Question 2. Where will the hand of a clock stop if it

a) Starts at 12 and makes 1/2 of a revolution, clockwise?

Solution. 6

b) Starts at 2 and makes 1/2 of a revolution, clockwise?

Solution. 8

c) Starts at 5 and makes 1/4 of a revolution, clockwise?

Solution. 8

d) Starts at 5 and makes 3/4 of a revolution, clockwise?

Solution. 2

Question 3. Which direction will you face if you start facing

a) East and make 1/2 of a revolution

Solution. West

b) East and make 1%, of a revolution clockwise?

Solution. West

c) West and make 3/4 of revolution anti-clock wise?

Solution. North.

d) South and make one full revolution?

Solution. South

(Should we specify clockwise or anti-clockwise for this last question? Why not?)

No, since the result of both types of revolutions is the same.

Question 4. What part of a revolution have you turned through if you stand facing

a) East and turn clockwise to face north?

Solution. 3/4 of a revolution.

b) South and turn clockwise to face east?

Solution. 3/4 of a revolution

c) West and turn clockwise to face east?

Solution. 1/2 of a revolution.

Question 5. Find the number of right angles turned through by the hour hand of a clock when it goes from

a) 3 to 6

Solution. 1 right angle

b) 2 to 8

Solution. 2 right angles

c) 5 to 11

Solution. 2 right angles

d) 10 to 1

Solution. 1 right angle

e) 12 to 9

Solution. 3 right angles

f) 12 to 6

Solution. 2 right angles.

Question 6. How many right angles do you make if you start facing

a) south and turn clockwise to west?

Solution. 1 right angle.

b) north and turn anti-clockwise to east?

Solution. 3 right angles.

c) west and turn to west?

Solution. 4 right angles.

d) south and turn to north?

Solution. 2 right angles.

Question 7. Where will the hours hand of a clock stop if it starts.

a) from 6 and turns through 1 right angle?

Solution. 9

b) from 8 and turns through 2 right angles ?

Solution. 2

c) from 10 and turns through 3 right angles ?

Solution. 7

d) from 7 and turns through 2 straight angles ?

Solution. 1

Question 8. Are there angles smaller than a right angle?

Solution. Yes. Acute angle (<90°)

Question 9. Are there angles greater than a right angle?

Solution. Yes. Obtuse angle (>90° and <180°)

Question 10. The hour hand of a clock moves from 12 to 5.

Is the revolution of the hour hand more than 1 right angle?

Class 6 Maths Chapter 5 Understanding Elementary shapes Question 1

Solution. Yes.

Question 11. What does the angle made by the hour hand of the clock look like when it moves from 5 to 7. Is the angle moved more than 1 right angle?

Class 6 Maths Chapter 5 Understanding Elementary shapes Question 2

Solution. The angle made by the hour hand looks like on acute angle. No, the angle moved by the hour hand is less than 1 right Sol. Opening of a book, roof top, when the angle.

Question 12. Look around you and identify edges meeting at corners to produce angles. List such situations.

Solution. Edge of a book, edge of a wall, edge of a table.

Question 13. List three situations where the angles made are acute.

Solution. Opening of a book, roof top, when the time in a clock is 3 hours 5 min.

Question 14. List three situations where the angles made are right angles.

Solution. The angle between two edges of a table, corners of room, when a pole is placed vertically on the ground.

Question 15. Find three situations where obtuse angles are made.

Solution. House, book reading desk, the angle between two hands of a clock if the time is 2 hours 55 min.

Question 16. List three situations where reflex angles may be seen.

Solution.

1) The angle between two hands of a clock if the time is 12 hours 40 min.

2) The angle made by the sector in the alternate segment at the centre.

3) The angle between two hands of a clock if the time is 5 hours.

Haryana Board Solutions For Class 7 Maths Chapter 5 Understanding Elementary Shapes Exercise – 5.3

Question 1. Classify each one of the following angles as right, straight, acute, obtuse or reflex:

Solution.

Class 6 Maths Chapter 5 Understanding Elementary shapes Exercise 5.3 Question 2

a) Acute b) Obtuse c) Right angle d) Reflex e) Straight f) Acute

Question 2. How many degrees are there in half a revolution? In one right angle? In one straight angle?

Solution. Half a revolution = 180°; Right angle = 90°; Straight angle = 180°

Question 3. How many right angles make 180° ? 360° ?

Solution. 2 right angles make 180° [90° x 2]

4 right angles make 360° [90° 4]

Chapter 5 Understanding Elementary Shapes Exercise – 5.4

Question 1. What is the measure of

(1) a right angle?

(2) a straight angle?

Solution. 1) The measure of a right angle is 90°.

2) The measure of a straight angle is 180°.

Question 2. Say True or False:

a) The measure of an acute angle < 90°.

Solution. True.

b) The measure of an obtuse angle. < 90°.

Solution. False.

c) The measure of a reflex angle > 180°.

Solution. True.

d) The measure of one complete revolution = 360°.

Solution. True.

e) If m∠A = 53° and m∠B = 35° then m∠A > m∠B.

Solution. True.

Question 3. Write down the measures of (a) some acute angles. (b) some obtuse angles.

(give atleast two examples of each.)

Solution. a) 30°, 85°

b) 110°, 170°

Question 4. Measure the angles given below using the Protractor and write down the measure.

Class 6 Maths Chapter 5 Understanding Elementary shapes Exercise 5.4 Question 4

Solution. a) 45°

b) 1280

c).90°

d) 65°

Polygons and their properties Class 7 Haryana Board

Question 5. Which angle has a large measure? First estimate and then measure.

Measure of Angle A =

Class 6 Maths Chapter 5 Understanding Elementary shapes Exercise 5.4 Question 5.1

Measure of Angle B =

Class 6 Maths Chapter 5 Understanding Elementary shapes Exercise 5.4 Question 5.2

Solution. Measure of angle A = 40°

Measure of angle B = 70°

∴ The angle B is a larger measure.

Question 6. From these two angles which has larger measure? Estimate and then confirm by measuring them.

Class 6 Maths Chapter 5 Understanding Elementary shapes Exercise 5.4 Question 6

Solution. Measure of first angle = 45°

Measure of second angle = 60°

Second angle has larger measure.

Question 7. Fill in the blanks with acute, obtuse, right or straight.

a) An angle whose measure is less than that of a right angle is _____

Answer. acute

b) An angle whose measure is greater than that of a right angle is _____

Answer. obtuse

c) An angle whose measure is the sum of the measures of two right angles is _____

Answer. straight

d) When the sum of the measures of two angles is that of a right angle, then each one of them is.

Answer. acute

e) When the sum of the measures of two angles is that of a straight angle and if one of them is acute then the other should be ___________ or ___________

Answer. acute or obtuse

Question 8. Find the measure of the angle shown in each figure (first estimate with your eyes and then find the actual measure with a protractor).

Solution.

Class 6 Maths Chapter 5 Understanding Elementary shapes Exercise 5.4 Question 8

1) 40°

2) 130°

3) 65°

4) 140°

Question 9. Find the angle measure between the hands of the clock in each figure:

Class 6 Maths Chapter 5 Understanding Elementary shapes Exercise 5.4 Question 9

Solution.

(1) 90°

(2) 30°

(3) 180°.

Question 10. Investigate: In the given figure the angle measures 30°. Look at the same figure through a magnifying glass. Does the angle becomes larger? Does the size of the angle change?

Class 6 Maths Chapter 5 Understanding Elementary shapes Exercise 5.4 Question 10

Solution. No.

Question 11. Measure and classify each angle:

Class 6 Maths Chapter 5 Understanding Elementary shapes Exercise 5.4 Question 11

Class 6 Maths Chapter 5 Understanding Elementary shapes Exercise 5.4 Question 11.1

Solution.

Class 6 Maths Chapter 5 Understanding Elementary shapes Exercise 5.4 Question 11.2

Haryana Board Solutions For Class 7 Maths Chapter 5 Understanding Elementary Shapes Exercise – 5.5

Question 1. Which of the following are models for perpendicular lines:

a) The adjacent edges of a table top.

Solution. a) Perpendicular lines

b) The lines of a railway track.

Solution. b) Not perpendicular lines.

c) The line segments forming the letter “L”.

Solution. Perpendicular lines.

d) The letter V.

Solution. Not perpendicular.

Question 2. Let \(\overline{\mathrm{PQ}}\) be the perpendicular to the line segment \(\overline{\mathrm{XY}}\).Let \(\overline{\mathrm{PQ}}\) and \(\overline{\mathrm{XY}}\) intersect in the point A. What is the measure of ∠PAY?

Solution. The point A is placed in \(\overline{\mathrm{XY}}\) such that it is placed in \(\overline{\mathrm{PQ}}\) also.

Measure of ∠PAY is 90°.

Question 3. There are two set squares in your box. What are the measures of the angles that are formed at their corners? Do they have any angle measure that is common?

Solution. The angles are 90°, 30°, 60°, 90°, 45°, 45° Yes, they have angle measure 90° as common.

Question 4. Study the diagram. The line l is perpendicular to line m.

a) Is CE = EG?

Solution. CE = 5 -3 = 2

EG = 7 – 5 = 2

Yes. CE = EG

Class 6 Maths Chapter 5 Understanding Elementary shapes Exercise 5.5 Question 4

b) Does PE bisect CG?

Solution. Yes

c) Identify any two line segments for which PE is the perpendicular bisector.

Solution. \(\overline{P E}\) bisects \(\overline{D F}\)

\(\overline{P E}\) bisects \(\overline{B H}\)

d) Are these true?

1) AC > FG Solution. Yes

2) CD = GH Solution. Yes

3) BC < EH Solution. Yes

Haryana Board Solutions For Class 7 Maths Chapter 5  Understanding Elementary Shapes Exercise – 5.6

Question 1. Name the types of following triangles:

a) Triangle with lengths of sides 7 cm, 8 cm and 9 cm.

Solution. Scalene.

b) ∠ABC with AB = 8.7 cm, AC = 7 cm and BC = 6 cm.

Solution. Scalene.

c) ∠PQR such that PQ = QR = PR=5 cm.

Solution. Equilateral.

d) ΔDEF with m∠D = 90°

Solution. Right angled triangle.

e) ΔXYZ with m∠Y=90° and XY = YZ.

Solution. Isosceles right angled.

f) ALMN with m∠L = 30° and m∠m = 70° and m∠N = 80°.

Solution. Acute

Question 2. Match the following:

Class 6 Maths Chapter 5 Understanding Elementary shapes Exercise 5.6 Question 2

Solution. 1 – e, 2 – g, 3 – a, 4 – f, 5 – d, 6 – c, 7 – b

Question 3. Name each of the following triangles in two different ways: (You may judge the nature

Class 6 Maths Chapter 5 Understanding Elementary shapes Exercise 5.6 Question 3

Solution. a) Isosceles triangle, acute angled triangle.

b) Right angled triangle, scalene triangle.

c) Obtuse angled triangle, isosceles triangle.

d) Right angled isosceles triangle, isosceles triangle.

e) Equilateral triangle, acute angled triangle.

f) Obtuse angled triangle, scalene triangle.

Question 4. Try to construct triangles using match sticks. Some are shown here.

Class 6 Maths Chapter 5 Understanding Elementary shapes Exercise 5.6 Question 4

Can you make a triangle with

a) 3 match sticks?

Solution. Yes

b) 4 match sticks?

Solution. No

c) 5 match sticks?

Solution. Yes

d) 6 match sticks?

Solution. Yes.

  • With 3 match sticks, the triangle so formed is an equilateral triangle.
  • With 5. match sticks, the triangle so formed is an isosceles triangle.
  • With 6 match sticks, the triangle so formed is an equilateral triangle.

3D shapes faces edges and vertices Class 7 HBSE

Question 5. Place a pair of unequal sticks such that they have their end point joined at one end. Now place another such pair meeting the free ends of the first pair.

Class 6 Maths Chapter 5 Understanding Elementary shapes Exercise 5.6 Question 5

What is the figure enclosed

It is a quadrilateral, like the one you see here.

The sides of the quadrilateral are \(\overline{\mathrm{AB}}, \overline{\mathrm{BC}}\)__,__,.

There are 4 angles for this quadrilateral.

They are givenby ∠BAD, ∠ADC, ∠DCB and _____

BD is one diagonal. What is the other? Measure the length of the sides and the diagonals.

Measure all the angles also.

Solution. The enclosed figure is quadrilateral whose sides are \(\overline{\mathrm{AB}}, \overline{\mathrm{BC}}, \overline{\mathrm{CD}}, \overline{\mathrm{DA}}\).

The 4 angles of the quadrialteral are ∠BAD, ∠ADC, ∠DCB, ∠ABC.

\(\overline{\mathrm{BD}}\) is one diagonal. The other diagonal is \(\overline{\mathrm{AC}}\)

The measures of length of the side diagonals and angles are as follows

AB = 2.8 cm

BC = 2 cm

CD = 3.3 cm

DA = 3.9 cm

AC = 3.8 cm

BD = 4 cm

∠ABC = 110°

∠BCD = 100°

∠CAD = 60°

∠DAB = 90°

Question 6. Using four unequal sticks, as you did in the above activity, see if you can form a quadrilateral such that

a) all the four angles is acute.

Solution. No

b) one of the angles is obtuse

Solution. Yes.

c) one of the angles is right angled.

Solution. Yes.

d) two of the angles are obtuse.

Solution. Yes

e) two of the angles are right angled.

Solution. Yes

f) the diagonals are perpendicular to one another.

Solution. Yes.

Haryana Board Solutions For Class 7 Maths Chapter 5  Understanding Elementary Shapes Exercise – 5.7

Question 1. Say True or False:

a) Each angle of a rectangle is a right angle.

Solution. True

b) The opposite sides of a rectangle are equal in length.

Solution. True

c) The diagonals of a square are perpendicular to one another.

Solution. True

d) All the sides of a rhombus are of equal length.

Solution. True

e) All the sides of a parallelogram are of equal length.

Solution. False.

f) The opposite sides of a trapezium are parallel.

Solution. False.

Question 2. Give reasons for the following:

a) A square can be thought of as a special rectangle.

Solution. Yes, because all the sides are equal.

b) A rectangle can be thought of as a special parallelogram.

Solution. Yes, because each angle is 90°.

c) A square can be thought of as a special rhombus.

Solution. Yes, the diagonals are of equal length.

d) Squares, rectangles, parallelograms are all quadrilaterals.

Solution. Yes, because all are of four sides.

e) Square is also a parallelogram.

Solution. Yes, each angle is 90° and all the sides are of equal length.

Question 3. A figure is said to be regular if its sides are equal in length and angles are equal in measure. Can you identify the regular quadrilateral ?

Solution. Square is a regular quadrilateral

Haryana Board Solutions For Class 7 Maths Chapter 5 Understanding Elementary Shapes Exercise – 5.8

Question 1. Examine whether the following are polygons. If any one among them is not, say why?

Class 6 Maths Chapter 5 Understanding Elementary shapes Exercise 5.8 Question 1

Solution. a) Not a polygon.

b) Polygon.

c) Not a polygon.

d) Not a polygon.

Question 2. Name each polygon.

Class 6 Maths Chapter 5 Understanding Elementary shapes Exercise 5.8 Question 2

Solution. a) Quadrilateral.

b) Triangle.

c) Pentagon.

d) Octagon.

Question 3. Draw a rough sketch of a regular hexagon. Connecting any three of its vertices, draw a triangle. Identify the type of the triangle you have drawn.

Solution.

Class 6 Maths Chapter 5 Understanding Elementary shapes Exercise 5.8 Question 3

The triangle drawn is an obtuse angled triangle.

Question 4. Draw a rough sketch of a regular octagon. (Use squared paper if you wish). Draw a rectangle by joining exactly four of the vertices of the octagon.

Solution.

Class 6 Maths Chapter 5 Understanding Elementary shapes Exercise 5.8 Question 4

Question 5. A diagonal is a line segment that joins any two vertices of the polygon and is not a side of the polygon. Draw a rough sketch of a pentagon and draw its diagonals.

Solution.

Class 6 Maths Chapter 5 Understanding Elementary shapes Exercise 5.8 Question 5

Haryana Board Solutions For Class 7 Maths Chapter 5 Understanding Elementary Shapes Very Short Answer Questions

Question 1. In the adjacent figure ∠AOB and ∠AOC are given. Which angle is clock wise and which angle is anti-clock wise?

Think and discuss with your friends

Class 6 Maths Chapter 5 Understanding Elementary shapes Very Short Answer Question 1

Solution. ∠AOB → Clock wise angle.

∠AOC → Anti-clock wise angle.

Question 2. Which are obtuse angles?

Class 6 Maths Chapter 5 Understanding Elementary shapes Very Short Answer Question 2

Solution. Obtuse angles: (1), (3), (5)

Question 3. Draw any two acute and two obtuse

Solution.

Class 6 Maths Chapter 5 Understanding Elementary shapes Very Short Answer Question 3

Question 4. Draw any two separate lines in a plane. Do they intersect at more than one point?

Solution.

Class 6 Maths Chapter 5 Understanding Elementary shapes Very Short Answer Question 4

1, m are two separate lines in a plane. They intersect at only one point ‘P’.

Question 5. If 11m then we can say that ml?

Solution. Yes.

If l ⊥ m then m ⊥ l.

Class 6 Maths Chapter 5 Understanding Elementary shapes Very Short Answer Question 5

Question 6. How many perpendicular lines can be drawn to a given line?

Class 6 Maths Chapter 5 Understanding Elementary shapes Very Short Answer Question 6

Solution. Many lines can be drawn.

l is the given line.

p, q, r, s, t, are perpendicular lines. So we can draw many perpendicular lines to a given line.

Question 7. Which letters in English alphabets possess perpendicularity?

Solution. In English alphabet the letters E, L, H, F,T possess perpendicularity.

Question 8. Make a pair of parallel lines what is the angle formed between them?

Solution.

Class 6 Maths Chapter 5 Understanding Elementary shapes Very Short Answer Question 8

l and m are parallel lines.

Angle between them is 0°.

Question 9. How do you measure the length of a line segment by using a ruler.

Class 6 Maths Chapter 5 Understanding Elementary shapes Very Short Answer Question 9

Solution. Place the zero mark (cm) of the scale or ruler at A. Read the mark against B.

This gives the length of AB line segment.

Length of \(\overline{\mathrm{AB}}\) = 4 cm

Question 10. By observing your class room, write two examples for right angles.

Solution. 1) Angle between floor and walls.

2) Angle between floor and doors.

Question 11. Answer the following questions.

1) What is the measure of the angle made by the seconds hand in 1 minute?

Solution. 360°.

2) What is the measure of the angle made by the hour hand in 1 hour?

Solution. 30°.

Haryana Board Solutions For Class 7 Maths Chapter 5  Understanding Elementary Shapes Short Answer Questions

Question 1. Use the straw angle apparatus and identify the following angles.

Class 6 Maths Chapter 5 Understanding Elementary shapes Short Answer Question 1

Solution. 1) Obtuse angle

2) Right angle

3) Acute angle.

4) Obtuse angle

Question 2. List out five daily life situations where you observe acute angle and obtuse angles.

Solution. 1) Acute angle: Stem of a plant with the leaf, V shaped Grill of a window, ladder with the floor etc.

2) Obtuse angle: Edge of the Glass with the bottom, Edge of the bucket with the bottom, Table with legs, Flame of the Gas stove, Edges of a bowl with the bottom etc.

Question 3. Draw some angles of your choice. Test them by angle apparatus and write which are acute and which are obtuse.

Class 6 Maths Chapter 5 Understanding Elementary shapes Short Answer Question 3

Solution. 1) obtuse angle

2) → acute angle

3) → acute angle

4) → obtuse angle

Word problems on elementary shapes for Class 7 HBSE

Question 4. Which angles in the adjacent figure are acute. Which are obtuse? Check your estimation by measuring them. Write their measures too.

Class 6 Maths Chapter 5 Understanding Elementary shapes Short Answer Question 4

Solution. Acute angles : ∠1 and ∠3

Obtuse angles : ∠2 and ∠4

Measuring :

∠1 = 70°;

∠3=80°

∠2=105°;

∠4=105°

Question 5. Write the type of angle formed between the long hand and short hand of a clock at the given timings.

(Take the small hand as the base)

Solution. 1) At 9 o’clock in the morning – Right angle, 90°

2) At 6 o’ clock in the evening – Straight angle, 180°

3) At 12 noon – Complete angle, 360°

4) At 4 o’ clock in the afternoon – Obtuse angle, 120°

5) At 8 o’clock in the night – Reflex angle, 240°

Question 6. Which of the following are models for parallel lines, perpendicular lines and which are neither of them.

Solution. 1) The vertical window bars : Parallel lines

2) Railway lines : Parallel lines

3) The adjacent edges of doors : Perpendicular lines

4) The letter ‘v’ in English alphabet : Neither of them

5) The opposite edges of Black Board : Parallel lines

Question 7. State the angle between the two hands of a clock, when the time is

Solution. 1) 6 o’clock – 180°

2) 12 o’clock – 0°

3) 3 o’clock – 90°

4) 8 o’ clock – 240°

5) 9 o’clock – 270°

Question 8. Look at the given triangle and answer the following questions.

Class 6 Maths Chapter 5 Understanding Elementary shapes Short Answer Question 8

1) How many angles are there in the triangle? What are they?

Solution. 3 angles: They are ∠MNO, ∠NOM and ∠OMN.

2) What is the angle opposite to \(\overline{\mathrm{MN}}\)?

Solution. ∠NOM.

3) Where is the right angle in the given triangle ?

Solution. Right angle is at vertex ‘O’.

Question 9. Look at the adjacent figure, and answer the following questions.

1) What is the name of the triangle?

2) Write all sides, all angles and all vertices of the triangle.

Class 6 Maths Chapter 5 Understanding Elementary shapes Short Answer Question 9

Solution. 1) ΔPQR

2) Sides : \(\overline{\mathrm{PQ}}, \overline{\mathrm{QR}}, \overline{\mathrm{PR}}\)

Angles : ∠PQR, ∠QRS, ∠RPQ.

Vertices : P, Q, R.

Haryana Board Solutions For Class 7 Maths Chapter 5  Understanding Elementary Shapes Long Answer Questions

Question 1. Classify the following angles into acute, right, obtuse and straight angles : 40°, 140°, 90°, 210°, 44°, 215°, 345°, 125°, 10°, 120°, 89°, 270°, 30°, 115°, 180°

Solution. Acute angles: 40°, 44°, 10°, 89°, 30°

(between 0° and 90°)

Right angles: 90°

Obtuse angles: 140°, 125°, 120°, 115°

(between 90° and 180°)

Reflex angles : 210°, 215°, 345°, 270°

(between 180° and 360°)

Straight angle: 180°

Question 2. Write True or False. Correct all those that are False.

1) An angle smaller than right angle is an acute angle.

True

2) A right angle measures 180°

Correct: A right angle measures 90°

False

3) A straight angle measures 90°

Correct: A straight angle measure 180°

False

4) The measure greater than 180° is a reflex angle.

True

5) A complete angle measures 360°

True

Question 3. Match the following angles with measures their. Draw figures for these as well.

Class 6 Maths Chapter 5 Understanding Elementary shapes Long Answer Question 3

Figures for these:

Class 6 Maths Chapter 5 Understanding Elementary shapes Long Answer Question 3.1

Difference between quadrilaterals and polygons Class 7

Question 4. ABCD is a rectangle. \(\overline{\mathrm{AC}} \text { and } \overline{\mathrm{BD}}\) are diagonals. Write the pairs of parallel lines, perpendicular lines and intersecting lines from the figure in symbolic form.

Class 6 Maths Chapter 5 Understanding Elementary shapes Long Answer Question 4

Solution. ABCD is a rectangle.

\(\overline{\mathrm{AC}} \text { and } \overline{\mathrm{BD}}\) are diagonals.

a) Parallel lines : \(\overline{\mathrm{AB}} \text { and } \overline{\mathrm{CD}} ; \overline{\mathrm{AD}} \text { and } \overline{\mathrm{BC}}\)

Symbolic form : \(\overline{\mathrm{AB}}\|\overline{\mathrm{CD}} ; \overline{\mathrm{AD}}\| \overline{\mathrm{BC}}\)

b) Perpendicular lines : \(\overline{\mathrm{AD}}, \overline{\mathrm{AB}} ; \overline{\mathrm{AB}}, \overline{\mathrm{BC}} ; \overline{\mathrm{BC}}, \overline{\mathrm{CD}} ; \overline{\mathrm{CD}}, \overline{\mathrm{DA}}\)

Symbolic form : \(\overline{\mathrm{AD}} \perp \overline{\mathrm{AB}} ; \overline{\mathrm{AB}} \perp \overline{\mathrm{BC}} ; \overline{\mathrm{BC}} \perp \overline{\mathrm{CD}} ; \overline{\mathrm{CD}} \perp \overline{\mathrm{DA}}\)

c) Pair of intersecting line: \(\overline{A C} \text { and } \overline{B D}\)

Question 5. State to which type of angle do the following belong.

Solution.

Class 6 Maths Chapter 5 Understanding Elementary shapes Long Answer Question 5

Question 6. Some angle measures are given below. State which kind do each belong.

1) 45°

2) 210°

3) 100°

4) 90°

5) 135°

6) 350°

7) 0°

8) 360°

9) 180°

10) 50°

Solution. 1) 45° – Acute angle

2) 210° – Reflex angle

3) 100° – Obtuse angle

4) 90° – Right angle

5) 135° – Obtuse angle

6) 350° – Reflex angle

7) 0° – Zero angle

8) 360° – Complete angle

9) 180° – Straight angle

10) 50° – Acute angle

Question 7. Measure all the line segments in the figure given below and arrange them in the ascending order of their lengths.

Class 6 Maths Chapter 5 Understanding Elementary shapes Long Answer Question 7

Line segments: \(\overline{\mathrm{AB}}, \overline{\mathrm{AC}}, \overline{\mathrm{AD}}, \overline{\mathrm{AE}}, \overline{\mathrm{BC}}, \overline{\mathrm{BD}}, \overline{\mathrm{BE}}, \overline{\mathrm{CD}}, \overline{\mathrm{CE}}, \overline{\mathrm{DE}}\)

Lengths:

\(\overline{\mathrm{AB}}\) = 3 cm

\(\overline{\mathrm{BC}}\) = 4 cm

\(\overline{\mathrm{CD}}\) = 2 cm

\(\overline{\mathrm{AC}}\) = 7 cm

\(\overline{\mathrm{BD}}\) = 6 cm

\(\overline{\mathrm{CE}}\) = 7 cm

\(\overline{\mathrm{AD}}\) = 9 cm

\(\overline{\mathrm{BE}}\) = 11 cm

\(\overline{\mathrm{DE}}\) = 5 cm

\(\overline{\mathrm{AE}}\) = 14 cm

Descending order: \(\overline{\mathrm{AE}}, \overline{\mathrm{BE}}, \overline{\mathrm{AD}}, \overline{\mathrm{AC}}, \overline{\mathrm{CE}}, \overline{\mathrm{BD}}, \overline{\mathrm{DE}}, \overline{\mathrm{BC}}, \overline{\mathrm{AB}}, \overline{\mathrm{CD}}\)

Ascending order: \(\overline{\mathrm{CD}}, \overline{\mathrm{AB}}, \overline{\mathrm{BC}}, \overline{\mathrm{DE}}, \overline{\mathrm{BD}}, \overline{\mathrm{CE}}, \overline{\mathrm{AC}}, \overline{\mathrm{AD}}, \overline{\mathrm{BE}}, \overline{\mathrm{AE}}\)

Question 8. Each of these figures given has many line segments. For the almirah we have shown one line segment along the longer edge. Identify and mark all such line segments in these figures.

Class 6 Maths Chapter 5 Understanding Elementary shapes Long Answer Question 8

Solution. 1) Line segments: \(\overline{\mathrm{AB}}, \overline{\mathrm{BC}}, \overline{\mathrm{CD}}, \overline{\mathrm{DA}}, \overline{\mathrm{DB}}\)

2) Line segments: \(\overline{\mathrm{PQ}}, \overline{\mathrm{QR}}, \overline{\mathrm{RT}}, \overline{\mathrm{TU}}, \overline{\mathrm{UV}}, \overline{\mathrm{VW}}, \overline{\mathrm{SR}}, \overline{\mathrm{SP}}, \overline{\mathrm{SU}}, \overline{\mathrm{WP}}\)

3) Line segments: \(\overline{\mathrm{AB}}, \overline{\mathrm{BC}}, \overline{\mathrm{CD}}, \overline{\mathrm{BE}}, \overline{\mathrm{GH}}, \overline{\mathrm{EF}}, \overline{\mathrm{AF}}, \overline{\mathrm{DE}}\)

Question 9. Look at the adjacent figure. Answer the following questions.

Class 6 Maths Chapter 5 Understanding Elementary shapes Long Answer Question 9

1) How many sides are there in the triangle? What are they?

Solution. 3 sides; They are: \(\overline{\mathrm{PQ}}, \overline{\mathrm{QR}} \text { and } \overline{\mathrm{PR}}\)

2) How many vertices lie on the triangle? What are they?

Solution. 3 vertices: They are P, Q and R.

3) What is the side opposite to the vertex P?

Solution. \(\overline{\mathrm{QR}}\)

4) What is the vertex opposite to \(\overline{\mathrm{PR}}\)?

Solution. Q

Question 10. Look at the given quadrilateral and answer the following questions.

Class 6 Maths Chapter 5 Understanding Elementary shapes Long Answer Question 10

1) What are the sides of the given quadrilateral ?

Solution. Sides are: \(\overline{\mathrm{AB}}, \overline{\mathrm{BC}}, \overline{\mathrm{CD}} \text { and } \overline{\mathrm{DA}}\)

2) What is the opposite side of \(\overline{\mathrm{AB}}\)?

Solution. \(\overline{\mathrm{CD}}\)

3) What is the opposite vertex of B?

Solution. D.

4) How many pairs of adjacent angles are there? What are they?

Solution. Four pairs: They are ∠A and ∠B, ∠B and ∠C; ∠C and ∠D; ∠D and ∠A.

5) How many pairs of opposite angles are there? What are they?

Solution. Two pairs: They are ∠A and ∠C; ∠B and ∠D.

Question 11. Look at the adjacent figure and answer the following questions.

1) Write the name of this polygon.

Class 6 Maths Chapter 5 Understanding Elementary shapes Long Answer Question 11

Solution. Quadrilateral EFGH

2) Write the pairs of adjacent sides and adjacent angles.

Solution. \(\overline{\mathrm{EF}}\) and \(\overline{\mathrm{FG}} ; \overline{\mathrm{FG}}\) and \(\overline{\mathrm{GH}} ; \overline{\mathrm{GH}}\) and \(\overline{\mathrm{HE}}, \overline{\mathrm{HE}} \text { and } \overline{\mathrm{EF}}\) are adjacent sides.

∠E and ∠F; ∠F and ∠G; ∠G and ∠H; ∠H and ∠E are adjacent angles.

3) Write all vertices, pairs of opposite sides and pairs of opposite angles.

Solution. Vertices: E, F, G, H

Pairs of opposite sides: \(\overline{\mathrm{EF}} \text { and } \overline{\mathrm{GH}} ; \overline{\mathrm{EH}} \text { and } \overline{\mathrm{FG}}\)

Pairs of opposite angles: ∠E and ∠G; ∠F and ∠H

Haryana Board Solutions For Class 7 Maths Chapter 5 Understanding Elementary Shapes Multiple Choice Question And Answers

 

Question 1. The angle above 180° and below 360° are called

  1. right angle
  2. reflex angle
  3. obtuse angle
  4. acute angle

Answer. 2. reflex angle

Question 2. Angle made by the seconds hand in 1 minute in a clock

  1. 30°
  2. 60°
  3. 360°

Answer. 4. 360°

Question 3.

Class 6 Maths Chapter 5 Understanding Elementary shapes Choose the correct answer Question 3

∠AOB is a ……. angle

  1. right
  2. acute
  3. obtuse
  4. zero

Answer. 3. obtuse

Question 4. The lines do not intersect are called

  1. parallel lines
  2. intersecting lines
  3. like lines
  4. unlike lines

Answer. 1. parallel lines

Question 5.

Class 6 Maths Chapter 5 Understanding Elementary shapes Choose the correct answer Question 5

In the figure the right angle is

  1. ∠A
  2. ∠B
  3. ∠C
  4. BC

Answer. 1. ∠A

Question 6. Length of two line segments by

  1. simple observation
  2. tracing on a paper and comparing
  3. using instruments
  4. all the above

Answer. 4. all the above

Question 7. Each centimeter is divided into ………. parts in a scale.

  1. one
  2. ten
  3. hundred
  4. 15

Answer. 2. ten

Question 8. Angles are measured by using an instrument

  1. scale
  2. set squares
  3. divider
  4. protractor

Answer. 4. protractor

Question 9. ‘⊥’ symbol for

  1. perpendicular
  2. parallel
  3. intersecting
  4. none of these

Answer. 1. perpendicular

Question 10. 1 cm = ………… mm

  1. 1
  2. 10
  3. 100
  4. \(\frac{1}{10}\)

Answer. 2. 10

Question 11. Number of line segments of a triangle is ………..

  1. 4
  2. 3
  3. 2
  4. 9

Answer. 2. 3

Question 12. How many sides are three in a pentagon?

  1. 4
  2. 6
  3. 5
  4. 2

Answer. 3. 5

Question 13. Number of sides of a octagon is ……

  1. 3
  2. 6
  3. 8
  4. 7

Answer. 3. 8

Question 14. How many least sides are required to form a polygon?

  1. 1
  2. 2
  3. 3
  4. 4

Answer. 3. 3

Question 15. ………… is a fixed portion of a line

  1. line segment
  2. circle
  3. cone
  4. sphere

Answer. 1. line segment

Question 16. A reflex angle is larger than ………

  1. 180°
  2. 450°
  3. 390°
  4. 400°

Answer. 1. 180°

Question 17. Which of the following is an acute angle?

  1. 162°
  2. 75°
  3. 90°
  4. 110°

Answer. 2. 75°

Question 18. The measure of right angle is ………

  1. 90°
  2. 100°
  3. 190°
  4. 80°

Answer. 1. 90°

Question 19. A triangle having three unequal sides is called …… triangle.

  1. Equilateral
  2. Isosceles
  3. Scalene
  4. None

Answer. 3. Scalene

Question 20. Each angle in equilateral triangle is ……..

  1. 30°
  2. 40°
  3. 80°
  4. 60°

Answer. 4. 60°

Question 21. Polygon with 6 sides is called ………

  1. triangle
  2. hexagon
  3. pentagon
  4. octagon

Answer. 2. hexagon

Chapter 5 Understanding Elementary Shapes Fill in the blanks

Question 22. 175° is a ……… angle

Answer. obtuse

Question 23.

Class 6 Maths Chapter 5 Understanding Elementary shapes Fill in the blanks Question 24

place the 1 mark (cm) of the ruler at A. Then the mark against B is 6.5 cm. the length of \(\overline{\mathrm{AB}}\) ………………….

Answer. 5.5 cm

Question 24. The unit of measuring an angle is a …………

Answer. degree

Question 25. Two …………….lines do not have any common point.

Answer. parallel

Question 26. Among 90°, 180°, 360°, 100° which is different ………..

Answer. 100°

Question 27. Degree is a ………. part of one revolution.

Answer. \(\frac{1}{360}\)th

Question 28. A ……… angle is more than a straight angle.

Answer. reflex

Question 29. The angle between perpendicular lines is ……..

Answer. 90°

Question 30. A protractor is a semi circular curved model with ……… equal divisions.

Answer. 180

Question 31. 359° is a…….

Answer. reflex angle

Question 32. “l and m are parallel lines”. The mathematical form of this statement ……….

Answer. l ∥ m

Question 33. The mathematical notation of the statement “l is perpendicular to m” is …………

Answer. l ⊥ m

Question 34. A ……………. is divided into 15 big parts as marked along one of its edges.

Answer. ruler or scale

Question 35. “Edge of a black board” is an example for ……….

Answer. line segment

Question 36. Hexagon contains ………. sides.

Answer. 6

Question 37. Number of diagonals of rhombus is ……….

Answer. 2

Question 38. A polygon with 8 sides is called …………..

Answer. Octagon

Question 39. A rhombus with 4 right angles is ………….

Answer. Square

Question 40. Two lines are perpendicular if the angle between them is …………

Answer. 90°

Question 41. The angle name for half revolution is ……….

Answer. Straight angle

Question 42. 1 mm …… cm.

Answer. 0.1

Question 43. A triangle is made of ……… line segments.

Answer. three

Question 44. Complete angle means ……..

Answer. 360°

Question 45. 115° is an example of ……. angle.

Answer. obtuse

Question 46. A polygon with 4 sides is called ………

Answer. quadrialteral

Chapter 5 Understanding Elementary Shapes Match the following

Question 47.

Class 6 Maths Chapter 5 Understanding Elementary shapes Match the following Question 48

Answer. 1. 1-B, 2-E, 3-C, 4-A, 5-D

Question 48.

Class 6 Maths Chapter 5 Understanding Elementary shapes Match the following Question 49

Answer. 2. 6-B, 7-D, 8-A, 9-E, 10-C